Anda di halaman 1dari 143

NOTAS DE TRABAJO, 30

ARITMTICA Y COMBINATORIA
(I. SUCESIONES RECURRENTES)
Pascual Jara Martnez
Departamento de lgebra. Universidad de Granada
Granada, 20042013
Primera redaccin: Julio 2004.
Segunda redaccin: Marzo 2006.
Tercera redaccin: Febrero 2007.
Cuarta redaccin: Mayo 2009.
Quinta redaccin: Enero 2013.
Introduction
This text is a compilation of Discrete Mathematics.
.
ndice general
Introduction I
I Sucesiones recurrentes 1
Introduccin 3
I Sucesiones 5
1 Progresiones aritmticas . . . . . . . . . . . . . . . . . . . . . . . . . . . . . . 6
2 Progresiones aritmticas de order superior . . . . . . . . . . . . . . . . . . . . 7
3 Progresiones geomtricas . . . . . . . . . . . . . . . . . . . . . . . . . . . . . . 13
4 Ejercicios con solucin . . . . . . . . . . . . . . . . . . . . . . . . . . . . . . . 14
II Sucesiones recurrentes 19
5 Sucesiones recurrentes homogneas . . . . . . . . . . . . . . . . . . . . . . . 20
6 Sucesiones recurrentes no homogneas . . . . . . . . . . . . . . . . . . . . . . 31
7 Funciones generatrices . . . . . . . . . . . . . . . . . . . . . . . . . . . . . . . 35
8 Ejercicios con solucin . . . . . . . . . . . . . . . . . . . . . . . . . . . . . . . 41
III Ejercicios de repaso 47
9 Ejercicios de repaso . . . . . . . . . . . . . . . . . . . . . . . . . . . . . . . . . 47
IV Ejercicios avanzados 83
10 Ejercicios avanzados . . . . . . . . . . . . . . . . . . . . . . . . . . . . . . . . . 83
V Ejemplos 91
11 Ejemplos . . . . . . . . . . . . . . . . . . . . . . . . . . . . . . . . . . . . . . . 91
VI Miscelnea 111
12 Nmeros naturales. El principio de induccin . . . . . . . . . . . . . . . . . . 111
13 Progresiones aritmticas . . . . . . . . . . . . . . . . . . . . . . . . . . . . . . 114
14 Progresiones geomtricas . . . . . . . . . . . . . . . . . . . . . . . . . . . . . . 117
15 Sucesiones recurrentes . . . . . . . . . . . . . . . . . . . . . . . . . . . . . . . 118
16 Ejercicios resueltos. Seleccin . . . . . . . . . . . . . . . . . . . . . . . . . . . 121
17 Recurrencia en combinatoria . . . . . . . . . . . . . . . . . . . . . . . . . . . . 122
18 Ampliacin de nmeros combinatorios . . . . . . . . . . . . . . . . . . . . . . 131
Bibliografa 135
2 NDICE GENERAL
ndice alfabtico 137
28 de enero de 2013 Curso 20122013. NOTAS DE TRABAJO, 30
Parte I
Sucesiones recurrentes
1
Introduccin
Pretendemos en este pequeo paneto resumir algunas ideas elementales sobre el estudio
de las sucesiones recurrentes, esto es, sucesiones recurrentes, lineales o no, con coecientes
constantes. Ejemplos de esta teora abundan en mltiples aplicaciones y adems son una
buena excusa para tratar sobre la aritmtica de nmeros: enteros, reales y complejos, y tam-
bin de polinomios.
No hemos pretendido ser exhaustivos en el tratamiento de la teora, de hecho hemos adop-
tado una presentacin ms bien light, pero hemos procurado que los argumentos utilizados
queden claros y sirvan de ejemplo para posteriores desarrollos. Es evidente que quedan fuera
algunos temas que iremos incluyendo en sucesivas versiones de este texto.
En el desarrollo que hemos hecho comenzamos con una presentacin de las progresiones
aritmticas de orden uno y superior; de forma que tenemos una fcil introduccin a las su-
cesiones denidas por recurrencia lineal con coecientes constantes. Luego, para tratar de
hacer un estudio completo de estas ltimas sucesiones, introducimos las progresiones geo-
mtricas y el polinomio caracterstico que stas denen. En este punto un poco de lgebra
Lineal sera necesaria, sobre todo para ver las sucesiones denidas por recurrencia como ele-
mentos de un espacio vectorial, y calcular una base del mismo.
El resto del texto se centra en el estudio de ejemplos y ejercicios relacionados con la teora
expuesta. Tal vez se eche en falta una ordenacin ms racional de los ejercicios y problemas
que se presentan, pero dejamos esto al lector. Tambin hemos realizado una recopilacin de
problemas que presentamos sin solucin; la no inclusin de la solucin podemos achacarla
a la falta material de tiempo para completar el texto.
Vamos a hacer responsable a esta falta de tiempo de las numerosas erratas que sin duda
salpicarn el texto por aqu y por all. Por esto agradeceramos al lector interesado en esta
materia cualquier comentario o informacinsobre las mismas conobjeto de corregir algunas
de stas en prximas versiones del texto, aunque no somos muy optimistas sobre su total
eliminacin; ni cuartel sin ratas ni libro sin erratas" dice un viejo proverbio castellano.
Hemos preferido hacer uso de la red de internet y de los numerosos textos que en ella vi-
ven para documentarnos, no vamos a citar ninguno de ellos en particular, pero desde aqu
agradecemos a los autores su generosidad al compartir con todos su inestimable trabajo. Sin
embargo nos gustara citar dos textos publicados en papel por gente prxima a la organiza-
cin de la Olimpada Matemtica Espaola que nos han ayudado mucho en la elaboracin
de este trabajo.
4
[1] Manual de matemticas para preparacin olmpica de Cristobal Snchez Rubio y Manuel
Ripolls Amela, publicado por la Universita Jaume I de Castelln en 2000.
[2] Sessions de preparaci per a lolimpada matemtica. Publicado por la Societat Catalana
de Matematiques en el ao 1999.
28 de enero de 2013 Curso 20122013. NOTAS DE TRABAJO, 30
Captulo I
Sucesiones
1 Progresiones aritmticas . . . . . . . . . . . . . . . . . . . . . . . . . . . . . 6
2 Progresiones aritmticas de order superior . . . . . . . . . . . . . . . . . . 7
3 Progresiones geomtricas . . . . . . . . . . . . . . . . . . . . . . . . . . . . 13
4 Ejercicios con solucin . . . . . . . . . . . . . . . . . . . . . . . . . . . . . . 14
Introduccin
Sea a
0
, a
1
, a
2
, a
3
, . . . una sucesin de nmeros (enteros, racionales, reales o complejos). Va-
mos a representar esta sucesin por {a
n
}
nN
, {a
n
}
n
o simplemente por a

si necesitamos
abreviar.
Estamos interesados en aquellas sucesiones en las que cada trmino es construido en fun-
cin de los anteriores mediante frmulas o reglas, que ms adelante determinaremos.
Los primeros ejemplos de esta teora son las progresiones aritmticas y geomtricas.
6 CAP. I. SUCESIONES
1. Progresiones aritmticas
Un tipo especial de sucesiones son las progresiones aritmticas. stas son sucesiones {a
n
}
n
para las que existe un nmero d, la diferencia, tal que el trmino a
n+1
est denido como
a
n+1
= a
n
+ d, para cada n 0.
Es claro que en una progresin aritmtica {a
n
}
n
de diferencia d se tiene
a
n
= a
0
+ n d.
sta es la expresin del trmino general de la progresin.
Dada una sucesin {a
n
}
n
, llamamos msima suma parcial a
S
m
= a
0
+ + a
m
.
Podemos calcular los valores de las sumas parciales de forma sencilla; basta hacer induccin
sobre m; se obtiene
S
m
=
(a
0
+ a
m
)(m + 1)
2
.
DEMOSTRACIN. El resultado es cierto para m = 0. Suponemos t 1, que el resultado es
cierto para m = t y vamos a ver qu ocurre para m = t + 1. Tenemos
S
t+1
= S
t
+ a
t+1
=
(a
0
+a
t
)(t+1)
2
+ a
t+1
=
a
0
(t+1)+a
0
(t+1)+td(t+1)+2a
0
+2(t+1)d
2
=
a
0
2(t+2)+(t+1)(t+2)d
2
=
(a
0
+a
0
+(t+1)d)(t+2)
2
=
(a
0
+a
t+1
)(t+2)
2
.

Observa que tambin se tiene S


m
= (m + 1)a
0
+
m(m+1)
2
d.
Ver el ejercicio (4.1.).
Otros ejercicios relacionados: (4.1.).
28 de enero de 2013 Curso 20122013. NOTAS DE TRABAJO, 30
SEC. 2. PROGRESIONES ARITMTICAS DE ORDER SUPERIOR 7
2. Progresiones aritmticas de order superior
Diferencias
Dada una sucesin {a
n
}
n
llamamos diferencia de orden uno de {a
n
}
n
a la sucesin {a
n
}
n
,
en donde
a
n
= a
n+1
a
n
.
Podemos representar esta sucesin tambin por {a
n
}
n
.
Por recurrencia se dene la diferencia de orden k > 1 de {a
n
}
n
como la sucesin

k
{a
n
}
n
=
k1
{a
n
}
n
.
La denicin se completa tomando
0
{a
n
}
n
= {a
n
}
n
.
La expresin de los trminos de
k
{a
n
}, en funcin de los a
n
, se calcula fcilmente por in-
duccin sobre k. En efecto, se tiene

k
a
n
=
k

i=0
(1)
i
_
k
i
_
a
n+ti
.
DEMOSTRACIN. Hacemos la demostracin para el trmino a
0
; para otros valores basta su-
mar una constante a cada uno de los ndices.
Tenemos a
0
= a
1
a
0
, entonces
2
a
0
= a
1
a
0
= a
2
2a
1
+a
0
. Supongamos que t 1
y hacemos la hiptesis

t
a
0
=
t

i=0
(1)
i
_
t
i
_
a
ti
.
Entonces:

t+1
a
0
=
t
a
1

t
a
0
=

t
i=0
(1)
i
_
t
i
_
a
t+1i

t
i=0
(1)
i
_
t
i
_
a
ti
= a
t+1
+

t
i=1
(1)
i
_
t
i
_
a
t+1i

t1
i=0
(1)
i
_
t
i
_
a
ti
(1)
t
a
0
= a
t+1
+

t
i=1
(1)
i
_
t
i
_
a
t+1i

t
i=1
(1)
i1
_
t
i1
_
a
t+1i
(1)
t
a
0
= a
t+1
+

t
i=1
(1)
i
__
t
i
_
+
_
t
i1
_
a
t+1i
(1)
t
a
0
= a
t+1
+

t
i=1
(1)
i
_
t+1
i
_
a
t+1i
(1)
t
a
0
=

t+1
i=0
(1)
i
_
t+1
i
_
a
t+1i

La sucesin {a
n
}
n
es una progresin aritmtica de grado k 0 si la sucesin
k
{a
n
}
n
es
constante.
Las progresiones aritmticas de la seccin (1) son progresiones aritmticas de grado uno.
ARITMTICA Y COMBINATORIA
(I. Sucesiones recurrentes)
P. Jara
8 CAP. I. SUCESIONES
Si {a
n
}
n
es una progresin aritmtica de grado k, podemos escribir:
a
0
= a
1
a
0

2
a
0
= a
2
2a
1
+ a
0

3
a
0
= a
3
3a
2
+ 3a
1
a
0
.
.
.

k
a
0
= a
k

_
k
1
_
a
k1
+ + (1)
k1
_
k
k1
_
a
1
+ (1)
k
a
0
= cte.

k+1
a
0
= a
k+1

_
k+1
1
_
a
k
+ + (1)
k
_
k+1
k
_
a
1
+ (1)
k+1
a
0
= 0
En resumen:

k
a
0
=
k

i=0
(1)
i
_
k
i
_
a
ki
. (I.1)
Surge ahora la pregunta de si el valor de a
k+1
se puede calcular en funcin de los
j
a
0
, con
0 j k + 1. La respuesta es s.
DEMOSTRACIN. Consideramos las sucesiones de las diferencias por columnas:
a
0
a
0

2
a
0

3
a
0
. . .
a
1
a
1

2
a
1

3
a
1
. . .
a
2
a
2

2
a
2

3
a
2
. . .
a
3
a
3

2
a
3

3
a
3
. . .
.
.
.
.
.
.
.
.
.
.
.
.
Es claro que esta tabla se puede tambin formar, a partir de la primera la, haciendo sumas,
en vez de diferencias. Veamos como hacer esos clculos.
Se tiene a
1
= a
0
+
1
a
0
, y
_
_
_
a
2
= a
1
+
1
a
1

1
a
1
=
1
a
0
+
2
a
0
a
1
= a
0
+
1
a
0
_
_
_
, luego a
2
= a
0
+2
1
a
0
+
2
a
0
. Suponemos
que para t 1 se verica a
t
=

t
i=0
_
t
i
_

i
a
0
, y anlogamente
1
a
t
=

t
i=0
_
t
i
_

i
a
0
, y vamos
a probar que el mismo resultado se tiene para a
t+1
.
a
t+1
= a
t
+
1
a
t
=
_

t
i=0
_
n
i
_

i
a
0
_
+
_

t
i=0
_
t
i
_

i+1
a
0
_
=
_

t
i=0
_
n
i
_

i
a
0
_
+
_

t+1
i=1
_
t
i1
_

i
a
0
_
=
_
t
0
_

0
a
0
+

t
i=1
__
t
i
_
+
_
t
i1
__

i
+
_
t
t
_

t+1
a
0
=
_
t
0
_

0
a
0
+

t
i=1
_
t+1
i
_

i
+
_
t
t
_

t+1
a
0
=

t+1
i=0
_
t+1
i
_

i
a
0
.
28 de enero de 2013 Curso 20122013. NOTAS DE TRABAJO, 30
SEC. 2. PROGRESIONES ARITMTICAS DE ORDER SUPERIOR 9
En resumen
a
n
=
n

i=0
_
n
i
_

i
a
0
. (I.2)

Observa que la expresin (I.2) es un polinomio en n. Como consecuencia de este desarrollo


tenemos el siguiente resultado:
Teorema. 2.1.
Una sucesin {a
n
}
n
es una progresin aritmtica de grado k si, y slo si, existe un polinomio
F de grado k tal que a
n
= F(n) para cada entero natural n.
DEMOSTRACIN. La condicin suciente es consecuencia del hecho expresado en la rela-
cin (I.2) de la pgina 9. El trmino a
n
es: a
n
=

n
i=0
_
n
i
_

i
a
0
=

k
i=0
_
n
i
_

i
a
0
, que es un
polinomio de grado k en n, ya que
i
a
0
= 0, si i > k, y
_
n
i
_
= 0, si i > n.
Para la condicin necesaria basta comprobar que para cada ndice n, si F(n) es un polinomio
en n de grado k, entonces el polinomio F(n + 1) F(n) es de grado menor o igual que k 1
en n.
El trmino general de una progresin aritmtica
Sea {a
n
}
n
una sucesin de grado k. Como se tiene
k+1
a
n
= 0 para cada ndice n, la relacin
(I.3) de la pgina 9, escrita para el ndice k + 1 es:
0 =
k+1

i=0
(1)
i
_
k + 1
i
_
a
k+1i
,
y por tanto el trmino a
k+1
se escribe:
a
k+1
=
k+1

i=1
(1)
i+1
_
k + 1
i
_
a
k+1i
. (I.3)
La misma relacin es vlida para ndices mayores que k + 1.
Por otro lado, de la relacin (I.2) se tiene que el trmino a
n
se expresa
a
n
=
n

i=0
_
n
i
_

i
a
0
, (I.4)
que como ya conocemos es un polinomio en n de grado como mximo k.
ARITMTICA Y COMBINATORIA
(I. Sucesiones recurrentes)
P. Jara
10 CAP. I. SUCESIONES
Ejemplo. 2.2.
Consideramos la sucesin aritmtica denida por
a
0
= 1, con diferencia d = 3,
esto es, la sucesin: 1, 4, 7, 10, 13, . . .
Se verica:
n
0
a
n

1
a
n

2
a
n
0 1 3 0
1 4 3 0
2 7 3 0
3 10 3
4 13
Tenemos una progresin aritmtica de grado uno, esto es, k = 1, por lo que la frmula (I.3)
se escribe
a
2
=
_
2
1
_
a
1

_
2
2
_
a
0
= 2a
1
a
0
,
y podemos cambiar los ndices obteniendo a
n+1
= 2a
n+1
a
n
.
La frmula que se obtiene de (I.4) es:
a
n
=
_
n
0
_

0
a
0
+
_
n
1
_

1
a
0
= 1 + 3n.
Sucesiones de potencias
Uncasoespecial de progresinaritmtica de gradosuperior loproporcionanlas progresiones
de potencias: {n
k
}
n
. Vamos a estudiar en este caso como calcular las sumas parciales.
Llamamos S
k
n
a la suma 0 + 1 + 2
k
+ 3
k
+ + n
k
.
Si k = 0 este valor es conocido. Veamos el caso general k 1.
Se verica, por el desarrollo del binomio:
(m + 1)
k
m
k
=
k1

i=0
_
k
i
_
m
i
.
Sumando en m, desde 1 hasta n, resulta:
(n + 1)
k
1 =

n
m=1
[(m + 1)
k
m
k
] =

n
m=1

k1
i=0
_
k
i
_
m
i
=

k1
i=0
_
k
i
_
n
m=1
m
i
=

k1
i=0
_
k
i
_
S
i
n
= n +

k1
i=1
_
k
i
_
S
i
n
.
28 de enero de 2013 Curso 20122013. NOTAS DE TRABAJO, 30
SEC. 2. PROGRESIONES ARITMTICAS DE ORDER SUPERIOR 11
Se obtiene entonces la relacin:
_
k
k 1
_
S
k1
n
= (n + 1)
k
(n + 1)
k2

i=1
_
k
i
_
S
i
n
. (I.5)
Los primeros ejemplos son:
S
1
n
=
1
2
((n + 1)
2
(n + 1)) =
n
2
+n
2
S
2
n
=
1
3
((n + 1)
3
(n + 1)

1
i=1
_
3
i
_
S
i
n
)
=
1
3
(n
3
+ 3n
2
+ 3n + 1 n 1 3S
1
n
)
=
1
3
(n
3
+ 3n
2
+ 2n
3
2
(n
2
+ n))
=
1
6
(2n
3
+ 3n
2
+ n)
=
n(n+1)(2n+1)
6
S
3
n
=
1
4
((n + 1)
4
(n + 1)

2
i=1
_
4
i
_
S
i
n
)
=
1
4
(n
4
+ 4n
3
+ 6n
2
+ 3n (4S
1
n
+ 6S
2
n
))
=
1
4
(n
4
+ 2n
3
+ n
2
) =
_
n(n+1)
2
_
2
(I.6)
El clculo explcito de S
i
n
se hace fcilmente a partir de aqu de forma recursiva.
Ver el problema (9.14.).
Aplicacin al clculo del trmino general de una progresin aritmtica
Consideramos una progresin aritmtica {a
n
}
n
de grado k > 1.
Supongamos que no conocemos el trmino general; vamos a dar unalgoritmo para calcularlo
determinando previamente los trminos generales de las sucesiones de menor grado que
aparecen asociadas a {a
n
}
n
.
Construimos la tabla siguiente:
a
0

1
a
0

2
a
0
. . .
k2
a
0

k1
a
0

k
a
0
a
1

1
a
1

2
a
1
. . .
k2
a
1

k1
a
1

k
a
1
a
2

1
a
2

2
a
2
. . .
k2
a
2

k1
a
2

k
a
2
a
3

1
a
3

2
a
3
. . .
k2
a
3

k1
a
3

k
a
3
a
4

1
a
4

2
a
4
. . .
k2
a
4

k1
a
4

k
a
4
Por hiptesis la sucesin
k
{a
n
}
n
es constante, por lo tanto su trmino general es sencillo de
calcular.
La sucesin
k1
{a
n
}
n
es de grado uno, su primer trmino es
k1
a
0
y su diferencia es exac-
tamente la constante
k
a
0
, luego su trmino general es:

k1
a
n
=
k1
a
0
+ n
k
a
0
.
ARITMTICA Y COMBINATORIA
(I. Sucesiones recurrentes)
P. Jara
12 CAP. I. SUCESIONES
Para calcular el trmino general de la sucesin
k2
{a
n
}
n
consideramos la suma de los tr-
minos de la sucesin
k1
{a
n
}
n
, sea sta
S
k1
m
=
k1
a
0
+ +
k1
a
m
, para m 0.
Observa que se tiene
S
k1
m

k1
a
m
=
k1
a
0
+ +
k1
a
m1
=
k2
a
m

k2
a
0
.
De aqu, cambiando el ndice de m a n, se tiene

k2
a
n
= S
k1
n
+
k2
a
0

k1
a
n
,
que nos da el trmino general de la sucesin
k2
{a
n
}
n
. Adems el clculo de S
k1
n
se realiza
sumando las potencias de n; en este caso el trmino general de
k1
{a
n
}
n
est dado por un
polinomio de grado uno,
k1
a
n
=
k1
a
0
+ n
k
a
0
, y por tanto
S
k1
n
=

n
i=0

k1
a
i
=

n
i=0
_

k1
a
0
+ i
k
a
0
_
= (n + 1)
k1
a
0
+
_
n
i=0
i
_

k
a
0
= (n + 1)
k1
a
0
+
n(n+1)
2

k
a
0
.
Una vez conocidos los trminos generales de las sucesiones
k
{a
n
}
n
,
k1
{a
n
}
n
, . . . ,
ki
{a
n
}
n
,
podemos calcular el trmino general de
ki1
{a
n
}
n
siguiendo el mismo procedimiento, as
se obtendr

ki1
a
n
= S
ki
n
+
ki1
a
0

ki1
a
n
,
y el clculo de S
ki
n
se realiza conociendo las sumas de las primeras potencias de los primeros
n + 1 enteros naturales.
Ver el problema (4.2.).
28 de enero de 2013 Curso 20122013. NOTAS DE TRABAJO, 30
SEC. 3. PROGRESIONES GEOMTRICAS 13
3. Progresiones geomtricas
Un segundo tipo de sucesin son las progresiones geomtricas; stas son sucesiones {a
n
}
n
para las que existe un nmero r, al que vamos a llamar la razn tal que el trmino a
n+1
est
denido como
a
n+1
= a
n
r,
para cada n N. Podemos probar por induccin que a
n
= a
0
r
n
.
Dada una progresin geomtrica {a
n
}
n
, consideramos la msima suma parcial a
S
m
= a
0
+ + a
m
.
Los valores de las sumas parciales podemos calcularlos de forma sencilla: En el caso en que
r = 1 se tiene que a
n
= a
0
para cada ndice n, luego S
m
= (m + 1)a
0
. En el caso en que r = 1
tenemos:
S
m
= a
0
+ + a
m
= a
0
(1 + r + + r
m
)
= a
0
(1+r++r
m
)(1r)
1r
= a
0
1r
m+1
1r
.
ARITMTICA Y COMBINATORIA
(I. Sucesiones recurrentes)
P. Jara
14 CAP. I. SUCESIONES
4. Ejercicios con solucin
Sucesiones
Ejercicio. 4.1.
Se considera un cuadrado 5 5 con algunas casillas marcadas.
74
186
103
0
Es posible completar el cuadrado sabiendo que cada la y cada columna forma una progre-
sin aritmtica (de grado uno)?
[From Erds to Kiev. Problems of Olympiad Caliber. MAA, 1996, pag. 93]
SOLUCIN
SOLUCIN. Ejercicio (4.1.)
Consideramos la primera columna, sea 4d, 3d, 2d, d, 0, entonces podemos completar con x
la cuarta la, comenzando sta por d, x, 103, y la tercera la que es: 2d, y, , , 186. Tenemos
entonces parte de la segunda columna, 74, y, x.
4d
3d
74
2d
y
186
d
x
103
0
Escribiendo las relaciones tenemos:
Cuarta la. x d = 103 x, equivalentemente x =
103+d
2
.
Segunda columna. 74 y = y x, o equivalentemente y =
74+x
2
=
251+d
4
.
Tercera la. 186 = 2d + 4d

= y + 3d

.
La ltima relacin da las siguientes:
d

= y 2d =
251 7d
4
y 2d + 4d

= 186.
Se obtiene entonces d = 13, x =
103+13
2
= 58 e y =
251+13
4
= 66. Entonces la diferencia en la
progresin de la segunda columna es: 8 y esto permite completar el cuadrado.
El resultado es:
28 de enero de 2013 Curso 20122013. NOTAS DE TRABAJO, 30
SEC. 4. EJERCICIOS CON SOLUCIN 15
30
52 82 112 142 172
35
39
74
109 144 179
40
26 66 106 146
186
45
13 58
103
148 193
50 0
50 100 150 200
13 8 3 2 7

Ejercicio. 4.2.
Determinar el trmino general de la sucesin 1, -7, -45, -23, 389, 1881, 5623, 13385, 27657,
51769.
SOLUCIN
SOLUCIN. Ejercicio (4.2.)
Primero comprobamos si son trminos de una progresin aritmtica calculando las diferen-
cias sucesivas:
n a
n
a
n

2
a
n

3
a
n

4
a
n

5
a
n
0 1 8 30 90 240 120
1 7 38 60 330 360 120
2 45 22 390 690 480 120
3 23 412 1080 1170 600 120
4 389 1492 2250 1770 720 120
5 1881 3742 4020 2490 840
6 5623 7762 6510 3330
7 13385 14272 9840
8 27657 24112
9 51769
Es pues una progresin aritmtica de orden cinco. Se trata ahora de averiguar cul es su tr-
mino general, ver pgina 9 y siguiente.
El trmino general de
5
{a
n
}
n
es
5
a
n
= 120.
El trmino general de
4
{a
n
}
n
es:
4
a
n
=
4
a
0
+ n
5
a
0
= 240 + 120n = 120(n + 2).
Para calcular el trmino general de
3
{a
n
}
n
tenemos que calcular
S
4
n
=
n

i=0

4
a
i
=
n

i=0
120(i + 2) = 120
n

i=0
(i + 2) = 120
(2 + n + 2)(n + 1)
2
= 60(n + 4)(n + 1).
A partir de aqu calculamos
3
a
n
.

3
a
n
= S
4
n
+
3
a
0

4
a
n
= 60(n + 4)(n + 1) + 90 120(n + 2) = 30(2n
2
+ 6n + 3).
ARITMTICA Y COMBINATORIA
(I. Sucesiones recurrentes)
P. Jara
16 CAP. I. SUCESIONES
Para calcular el trmino general de
2
{a
n
}
n
tenemos que calcular
S
3
n
=
n

i=0

3
a
i
=
n

i=0
(30(2i
2
+ 6i + 3)) = 30
_
2
n

i=0
i
2
+ 6
n

i=0
i + 3
n

i=0
1
_
= 30
_
2
n(n + 1)(2n + 1)
6
+ 6
n(n + 1)
2
+ 3(n + 1)
_
= 10(2n
3
+ 12n
2
+ 19n + 9)
Ahora calculamos
2
a
n
.

2
a
n
= S
3
n
+
2
a
0

3
a
n
= 10(2n
3
+12n
2
+19n+9)3030(2n
2
+6n+3) = 10(2n
3
+6n
2
+n3).
Para calcular el trmino general de {a
n
}
n
tenemos que calcular
S
2
n
=
n

i=0

2
a
i
=
n

i=0
(10(2i
3
+ 6i
2
+ i 3))
= 10
_
2
n

i=0
i
3
+ 6
n

i=0
i
2
+
n

i=0
i 3
n

i=0
1
_
= 10
_
2
_
n(n + 1)
2
_
2
+ 6
n(n + 1)(2n + 1)
6
+
n(n + 1)
2
3(n + 1)
_
= 5(n
4
+ 6n
3
+ 8n
2
3n 6).
Ahora calculamos a
n
.
a
n
= S
2
n
+ a
0

2
a
n
= 5(n
4
+ 6n
3
+ 8n
2
3n 6) 8 10(2n
3
+ 6n
2
+ n 3)
= 5n
4
+ 10n
3
20n
2
25n 8.
Para calcular el trmino general de {a
n
}
n
tenemos que calcular
S
1
n
=
n

i=0
a
i
=
n

i=0
(5i
4
+ 10i
3
20i
2
25i 8)
= 5
n

i=0
i
4
+ 10
n

i=0
i
3
20
n

i=0
i
2
25
n

i=0
i 8
n

i=0
1
= 5
_
n(n + 1)(2n + 1)(3n
2
+ 3n 1)
30
_
+ 10
_
n(n + 1)
2
_
2
20
n(n + 1)(2n + 1)
6
25
n(n + 1)
2
8(n + 1) = n
5
+ 5n
4
20n
2
24n 8.
En donde hemos utilizado que

n
i=0
i
4
=
n(n+1)(2n+1)(3n
2
+3n1)
30
.
28 de enero de 2013 Curso 20122013. NOTAS DE TRABAJO, 30
SEC. 4. EJERCICIOS CON SOLUCIN 17
Ahora calculamos a
n
.
a
n
=
0
a
n
= S
1
n
+
0
a
0

1
a
n
= n
5
+ 5n
4
20n
2
24n 8 + 1 (5n
4
+ 10n
3
20n
2
25n 8)
= n
5
10n
3
+ n + 1.
En consecuencia el trmino general de la sucesin {a
n
}
n
es:
a
n
= n
5
10n
3
+ n + 1.
Otra solucin.
De la ecuacin (I.2) se deduce que el trmino general de la sucesin {a
n
}
n
es:
a
n
=
n

i=0
=
_
n
i
_

i
a
0
,
por lo tanto en este caso resulta:
a
n
=
_
n
0
_
a
0
+
_
n
1
_

1
a
0
+
_
n
2
_

2
a
0
+
_
n
3
_

3
a
0
+
_
n
4
_

4
a
0
+
_
n
5
_

5
a
0
= a
0
+ na
0
+
n(n 1)
2

2
a
0
+
n(n 1)(n 2)
6

3
a
0
+
n(n 1)(n 2)(n 3)
24

4
a
0
+
n(n 1)(n 2)(n 3)(n 4)
120

5
a
0
= 18n15n(n1)+15n(n1)(n2)+10n(n1)(n2)(n3)+n(n1)(n2)(n3)(n4)
= n
5
10n
3
+ n + 1.
El trmino general de la sucesin es:
a
n
= n
5
10n
3
+ n + 1.

ARITMTICA Y COMBINATORIA
(I. Sucesiones recurrentes)
P. Jara
18 CAP. I. SUCESIONES
28 de enero de 2013 Curso 20122013. NOTAS DE TRABAJO, 30
Captulo II
Sucesiones recurrentes
5 Sucesiones recurrentes homogneas . . . . . . . . . . . . . . . . . . . . . . 20
6 Sucesiones recurrentes no homogneas . . . . . . . . . . . . . . . . . . . . 31
7 Funciones generatrices . . . . . . . . . . . . . . . . . . . . . . . . . . . . . . 35
8 Ejercicios con solucin . . . . . . . . . . . . . . . . . . . . . . . . . . . . . . 41
Introduction
La frmula (I.3) de la pgina 9
a
k+1
=
_
k + 1
1
_
a
k
+ + (1)
k+1
_
k + 1
k
_
a
1
+ (1)
k+2
a
0
nos indica que los trminos de una sucesin aritmtica de orden superior se pueden cons-
truir de forma recurrente. En este captulo vamos a adentrarnos en el estudio de este tipo
ms general de sucesiones.
20 CAP. II. SUCESIONES RECURRENTES
5. Sucesiones recurrentes homogneas
Una sucesin {a
n
}
n
se llama recurrente de orden t si existen nmeros c
1
, . . . , c
t
, con c
t
= 0, y
un ndice m 0 tal que para cada n m se verica:
a
n+t
= a
n+t1
c
1
+ + a
n
c
t
. (II.1)
Podemos suponer, prescindiendo de los trminos necesarios, que en la denicin se verica
m = 0.
Una sucesin recurrente de orden t est denida por dos t-uplas,
- una los elementos c
1
, . . . , c
t
, con c
t
= 0, y
- otra los t primeros trminos de la sucesin a
0
, . . . , a
t1
, a los que llamaremos condicio-
nes iniciales de la sucesin. El resto de los elementos se dene por:
a
n+t
= a
n+t1
c
1
+ + a
n
c
t
, (n 0). (II.2)
Llamamos a (II.1) a (II.2) la ecuacin de recurrencia de la sucesin.
Ejemplo. 5.1.
Cada progresin aritmtica de orden k es una sucesin recurrente de orden k + 1.
Ver pgina 9.
Ejemplo. 5.2.
Cada progresin geomtrica es una sucesin recurrente de orden 1.
La sucesin
a
0
= a, a
1
= ar, a
2
= ar
2
, . . . , a
n
= ar
n
. . .
cumple con la ecuacin recurrente a
n+1
= a
n
r, para n 0.
Ejemplo. 5.3. (Sucesin de Fibonacci.)
El prototipo de sucesin recurrente es la sucesin de Fibonacci denida:
a
0
= 0,
a
1
= 1,
a
n+2
= a
n+1
+ a
n
, la ecuacin de recurrencia para n 0.
El problema que estudia Fibonacci es la evolucin de una poblacin de conejos sometida a
las siguientes reglas:
(1) Se inicia el proceso con una pareja recin nacida.
(2) Cada pareja es frtil al cabo de un mes, y se reproduce cada mes dando lugar a una nueva
pareja.
(3) En el momento cero no hay ninguna pareja.
28 de enero de 2013 Curso 20122013. NOTAS DE TRABAJO, 30
SEC. 5. SUCESIONES RECURRENTES HOMOGNEAS 21
La evolucin es la siguiente: Al inicio del primer periodo se introduce una pareja. Al inicio
del segundo periodo solamente tenemos una pareja. Al inicio del tercer periodo tenemos dos
parejas. Al inicio del cuarto periodo tenemos tres parejas, y as sucesivamente.
La siguiente es una representacin grca de este proceso:
Periodo
Parejas Nm. Parejas
0
(0
1 1
$$
J
J
J
J
J
J
J
J
J
J
J
J
J
J
J
J
J
J
J
J
J
J
J
J
J
J
J

(1
2 1

A
A
A
A
A
A
A
A
A
A
A
A
A
A
A
A
A
A
A
A
A

(1
3 1

3
3
3
3
3
3
3
3
3
3
3
3
3
3
3
3
3

-
-
-
-
-
-
-
-
-
-
-
-
-
-
-
-

(2
4 1

(3
5 1 1 1 1 1
(5
Eneste caso a
n
es el nmero de parejas, de nuestra poblacinde conejos, al inicio del periodo
n.
La sucesin es: 0, 1, 1, 2, 3, 5, 8, 13, 21, 34, 55, . . .
El espacio vectorial
Para cada t nmeros: c
1
, . . . , c
t
, c
t
= 0, el conjunto de las sucesiones recurrentes con ecuacin
recurrente
a
n+t
= a
n+t1
c
1
+ + a
n
c
t
. (n 0) (II.3)
tiene estructura de espacio vectorial de dimensin t, ya que tenemos en principio t +1 grados
de libertad, uno para cada a
i
, i = n, n + 1, . . . , n + t, y una restriccin, justo la que da la
ecuacin (II.3).
Podemos calcular una base de este espacio, pero vamos primero a ver qu progresiones geo-
mtricas podemos encontrar en este espacio vectorial.
Consideramos una progresin geomtrica, por ejemplo a
n
= a
n1
r = a
0
r
n
, con a
0
= 0 y r = 0
para evitar casos triviales.
Se verican entonces las igualdades
a
0
r
n+t
= a
0
r
n+t1
c
1
+ + a
0
r
n
c
t
. (II.4)
ARITMTICA Y COMBINATORIA
(I. Sucesiones recurrentes)
P. Jara
22 CAP. II. SUCESIONES RECURRENTES
r
t
= r
t1
c
1
+ + rc
t1
+ c
t
. (II.5)
Llamamos a sta la ecuacin caracterstica de la sucesin.
Cada solucin de esta ecuacin determina una progresin geomtrica {1, ,
2
, . . .} que
pertenece al espacio vectorial.
Cuando tiene multiplicidad al menos dos tenemos en este mismo espacio otra sucesin
recurrente, ahora no geomtrica, la denida por 0, , 2
2
, 3
3
, . . .. Para comprobarlo, como
es tambin raz de la derivada de r
t
r
t1
c
1
c
t
, entonces es raz del polinomio:
tr
t1
(t 1)r
t2
c
1
2rc
t2
c
t1
, (II.6)
y al multiplicar por r, tambin es raz de
tr
t
(t 1)r
t1
c
1
2r
2
c
t2
rc
t1
. (II.7)
De esta forma la sucesin 0, , 2
2
, 3
3
, . . . tambin verica la ecuacin (II.4) y pertenece el
espacio de soluciones..
Cuando la multiplicidad de es tres o superior podemos seguir un proceso similar para en-
contrar nuevas sucesiones. Veamos el caso de tres: Tras derivar tenemos que es raz de
t(t 1)r
t2
(t 1)(t 2)r
t3
c
1
3 2rc
t3
2c
t2
, (II.8)
multiplicando por r
2
se obtiene que es raz de:
t(t 1)r
t
(t 1)(t 2)r
t1
c
1
3 2r
3
c
t3
2r
2
c
t2
. (II.9)
Ahora es fcil comprobar que (II.7)+(II.9) nos da
t
2
r
t
(t 1)
2
r
t1
c
1
3
2
r
3
c
t3
2r
2
c
t2
r
2
c
t1
, (II.10)
y que es raz de este polinomio, tenemos entonces la solucin: 0, , 2
2

2
, 3
2

3
, . . .
Para multiplicidades superiores se obtienen resultados anlogos, ya que basta desarrollar los
productos t(t 1)(t 2), t(t 1)(t 2)(t 3), etc., y obtener la mxima potencia de t en
trminos de las restantes.
De esta forma podemos obtener nuevas sucesiones recurrentes en el mismo espacio vecto-
rial. De hecho si la multiplicidad de es s, obtenemos la siguientes sucesiones recurrentes,
que son linealmente independientes:
1 : 1, ,
2
,
3
, . . .
2 : 0, , 2
2
, 3
3
, . . .
3 : 0, , 2
2

2
, 3
2

3
, . . .
.
.
.
s : 0, , 2
s1

2
, 3
s1

3
, . . .
28 de enero de 2013 Curso 20122013. NOTAS DE TRABAJO, 30
SEC. 5. SUCESIONES RECURRENTES HOMOGNEAS 23
Si las races del polinomio r
t
r
t1
c
1
c
t
son
1
, . . . ,
h
con multiplicidades s
1
, . . . , s
h
,
respectivamente, uniendo las construcciones para cada una de ellas obtenemos una base del
espacio vectorial formada por s
1
+ + s
h
sucesiones recurrentes.
Ejemplo. 5.4. (Sucesin de Fibonacci.)
Vamos a calcular el trmino general de la sucesin de Fibonacci dada por la ecuacin de
recurrencia a
n+2
= a
n+1
+ a
n
, para n 0 con las condiciones iniciales a
0
= 0 y a
1
= 1. Ver
ejemplo (5.3.).
En este caso t = 2, y c
1
= 1 = c
2
.
La ecuacin caracterstica es: r
2
r 1 = 0.
Las races son:
1
=
1+

5
2
y
2
=
1

5
2
, y son races simples.
Una base del espacio de las sucesiones recurrentes est formado por las dos progresiones
geomtricas: {
n
1
}
n
y {
n
2
}
n
. Entonces el trmino general a
n
se escribe
a
n
=
1

n
1
+
2

n
2
, para cada entero no negativo n.
En donde
1
y
2
son nmeros que vamos a determinar utilizando las condiciones iniciales.
Para n = 0 se verica: 0 =
1
+
2
, y para n = 1 se tiene: 1 =
1

1
+
2

2
.
Tenemos pues un sistema lineal

1
+
2
= 0

1
+
2

2
= 1
_
cuyas soluciones son:

1
=

5
5
,
2
=

5
5
.
El trmino general de la sucesin de Fibonacci es:
a
n
=

5
5
_
1 +

5
2
_
n

5
5
_
1

5
2
_
n
.
Esta expresin se conoce como la Frmula de Binet,y para cada entero no negativo da siem-
pre valores enteros.
Ver tambin otro ejemplo en el problema (8.5.).
ARITMTICA Y COMBINATORIA
(I. Sucesiones recurrentes)
P. Jara
24 CAP. II. SUCESIONES RECURRENTES
Construccin de sucesiones recurrentes. Producto
Supongamos que tenemos dos sucesiones recurrentes, {a
n
}
n
y {b
n
}
n
dadas por las ecuacio-
nes de recurrencia
a
n+t
= a
n+t1
c
1
+ + a
n
c
t
b
m+s
= b
m+s1
d
1
+ + b
m
d
s
podemos suponer que tenemos trminos generales dados por expresiones del tipo siguiente:
a
n
=

h
i=1
p
i

n
i
,
b
n
=

k
j=1
q
j

n
j
en donde p
i
y q
j
son polinomios en n, cuyo grado depende de la multiplicidad de la raz
i

j
, respectivamente en la ecuacin caracterstica.
Denimos una nueva sucesin mediante:
x
n
= a
n
b
n
, para cada ndice n.
El trmino general de la nueva sucesin {x
n
}
n
es el producto de los trminos de las sucesio-
nes factores.
x
n
=
h

i=1
p
i

n
i
k

j=1
q
j

n
j
=

i,j
p
i
q
j
(
i

j
)
n
,
entonces el problema es el clculo de la ecuacin caracterstica de {x
n
}
n
; sta va a depender
de los polinomios p
i
y q
j
que aparezcan. Vamos a estudiar dos ejemplos.
Proposicin. 5.5. (Caso cuadrtico)
Consideramos las sucesiones recurrentes
a
n+2
= a
n+1
c
1
+ a
n
c
2
y condiciones iniciales a
0
, a
1
;
b
n+2
= b
n+1
d
1
+ b
n
d
2
y condiciones iniciales b
0
, b
1
.
Si
1
,
2
son las races de la ecuacin caracterstica r
2
= rc
1
+ c
2
y si
1
,
2
son las races de
la ecuacin caracterstica r
2
= rd
1
+ d
2
, entonces tenemos:
a
n
=
1

n
1
+
2

n
2
,
b
n
=
1

n
1
+
2

n
2
,
x
n
=
1

1
(
1

1
)
n
+
1

2
(
1

2
)
n
+
2

1
(
2

1
)
n
+
2

2
(
2

2
)
n
(Los casos en que existen races mltiples se tratan de la misma manera.)
Como consecuencia el polinomio caracterstico de {x
n
}
n
es:
(r
1

1
)(r
1

2
)(r
2

1
)(r
2

2
),
la cual se puede expresar en funcin de los polinomios caractersticos originales ya que sus
coecientes son polinomios simtricos en los
i
y polinomios simtricos en los
j
.
28 de enero de 2013 Curso 20122013. NOTAS DE TRABAJO, 30
SEC. 5. SUCESIONES RECURRENTES HOMOGNEAS 25
En nuestro caso tenemos la siguiente expresin:
(r
1

1
)(r
1

2
)(r
2

1
)(r
2

2
)
=
2
1

2
2

2
1

2
2

_

2
1

2
1

2
+
1

2
2

2
1

2
+
2
1

2
2
+
1

2
2

2
2
_
r
+
_

2
1
+
2
1

2
+ 2
1

2
+
2
2

2
+
1

2
2
_
r
2
(
1

1
+
2

1
+
1

2
+
2

2
) r
3
+ r
4
= c
2
2
d
2
2
c
1
c
2
d
1
d
2
r (c
2
1
d
2
+ c
2
d
2
1
+ 2c
2
d
2
)r
2
c
1
d
1
r
3
+ r
4
Ejercicio. 5.6.
Se consideran las sucesiones recurrentes denidas por
a
0
= 0, a
1
= 1 y la frmula a
n+2
= 2a
n+1
a
n
para n 0 y
b
0
= 1, b
1
= 2 y la frmula b
n+2
= 3a
n+1
2a
n
para n 2.
Se dene la nueva sucesin recurrente con x
n
= a
n
b
n
. Vamos a calcular el trmino general
de x
n
y su ecuacin caracterstica.
Primero el polinomio caracterstico es:
c
2
2
d
2
2
c
1
c
2
d
1
d
2
r (c
2
1
d
2
+ c
2
d
2
1
+ 2c
2
d
2
)r
2
c
1
d
1
r
3
+ r
4
,
y como c
1
= 2, c
2
= 1, d
1
= 3, d
2
= 2, tenemos el valor:
r
4
6r
3
+ 13r
2
12r + 4.
Las races de este polinomio son
1
= 1,
2
= 1,
3
= 2 y
4
= 2. Los valores iniciales son:
x
0
= a
0
b
0
= 0 1 = 0,
x
1
= a
1
b
1
= 1 2 = 2,
x
2
= a
2
b
2
= 2 4 = 8,
x
3
= a
3
b
3
= 3 8 = 24.
La solucin general es:
x
n
= (
1
+ n
2
) + (
3
+ n
4
)2
n
.
Para averiguar el trmino general de la solucin al problema, basta resolver el sistema de
ecuaciones lineales:

1
+
3
= 0,

1
+
2
+2
3
+2
4
= 2,

1
+2
2
+4
3
+8
4
= 8,

1
+3
2
+8
3
+24
4
= 24
_

_
que tiene la solucin
1
=
2
=
3
= 0,
4
= 1.
ARITMTICA Y COMBINATORIA
(I. Sucesiones recurrentes)
P. Jara
26 CAP. II. SUCESIONES RECURRENTES
El trmino general de la solucin es:
x
n
= n2
n
.
La comprobacin de que el resultado es correcto es fcil, ya que el trmino general de {a
n
}
n
es a
n
= n y el de {b
n
}
b
es b
n
= 2
n
, y por lo tanto el trmino general del producto es n2
n
.
Ejercicio. 5.7.
Se considera las sucesiones recurrentes denidas por
a
0
= 1, a
1
= 1 y la frmula a
n+2
= 2a
n+1
a
n
para n 0 y
b
0
= 1, b
1
= 0 y la frmula b
n+2
= 5b
n+1
6b
n
para n 0.
Se dene una nueva sucesin recurrente con x
n
= a
n
b
n
. Vamos a averiguar el trmino
general de x
n
y su ecuacin caracterstica.
Primero el polinomio caracterstico es: c
2
2
d
2
2
c
1
c
2
d
1
d
2
r (c
2
1
d
2
+c
2
d
2
1
+2c
2
d
2
)r
2
c
1
d
1
r
3
+r
4
,
y como c
1
= 2, c
2
= 1, d
1
= 5, d
2
= 6, tenemos el valor:
r
4
+ 10r
3
+ 37r
2
+ 60r + 36.
Las races de este polinomio son
1
= 3,
2
= 3,
3
= 2 y
4
= 2. Los valores iniciales
son:
x
0
= a
0
b
0
= 1 1 = 1,
x
1
= a
1
b
1
= 1 0 = 0,
x
2
= a
2
b
2
= 1 (6) = 6,
x
3
= a
3
b
3
= 1 (30) = 30.
Para calcular el trmino general de la solucin al problema calcular una solucin general,
x
n
= (
1
+ n
2
)(3)
n
+ (
3
+ n
4
)(2)
n
,
e imponemos que verique las condiciones iniciales. Tenemos que resolver el sistema de
ecuaciones lineales:

1
+
3
= 1,
(
1
+
2
)(3) (
3
+
4
)(2) = 0,
(
1
+ 2
2
)(3)
2
(
3
+ 2
4
)(2)
2
= 6,
(
1
+ 3
2
)(3)
3
(
3
+ 3
4
)(2)
2
= 30
_

_
que tiene la solucin
1
= 2,
2
= 0,
3
= 3,
4
= 0. As pues tenemos el trmino general
x
n
= 2 (3)
n
+ 3 (2)
n
.
La comprobacin es fcil, ya que el trmino general de {a
n
}
n
es a
n
= (1)
n
y el de {b
n
}
b
es
b
n
= 3 2
n
2 3
n
.
28 de enero de 2013 Curso 20122013. NOTAS DE TRABAJO, 30
SEC. 5. SUCESIONES RECURRENTES HOMOGNEAS 27
Ejercicio. 5.8.
Se considera las sucesiones recurrentes denidas por
a
0
= 1, a
1
= 3 y la frmula a
n+2
= 2a
n+1
+ a
n
para n 0 y
b
0
= 1, b
1
= 0 y la frmula b
n+2
= 5b
n+1
6b
n
para n 0.
Se dene la nueva sucesinrecurrente conx
n
= a
n
b
n
. Vamos a averiguar el trmino general
de x
n
y su ecuacin caracterstica.
Primero el polinomio caracterstico es: c
2
2
d
2
2
c
1
c
2
d
1
d
2
r (c
2
1
d
2
+c
2
d
2
1
+2c
2
d
2
)r
2
c
1
d
1
r
3
+r
4
,
y como c
1
= 2, c
2
= 1, d
1
= 5, d
2
= 6, tenemos el valor:
r
4
+ 10r
3
+ 11r
2
60r + 36.
Las polinomio races de este son
1
= 2(1

2),
2
= 3(1

2),
3
= 2(1 +

2) y

4
= 3(1 +

2). Los valores iniciales son:


x
0
= a
0
b
0
= 1 1 = 1,
x
1
= a
1
b
1
= 3 0 = 0,
x
2
= a
2
b
2
= 5 (6) = 30,
x
3
= a
3
b
3
= 13 (30) = 390.
Para calcular el trmino general, x
n
=
1
(2(1

2))
n
+
2
(3(1

2))
n
+
3
(2(1+

2))
n
+

4
(3(1 +

2))
n
, basta resolver el sistema de ecuaciones lineales:

1
+
2
+
3
+
4
= 1

1
(2(1

2)) +
2
(3(1

2)) +
3
(2(1 +

2)) +
4
(3(1 +

2)) = 0

1
(2(1

2))
2
+
2
(3(1

2))
2
+
3
(2(1 +

2))
2
+
4
(3(1 +

2))
2
= 30

1
(2(1

2))
3
+
2
(3(1

2))
3
+
3
(2(1 +

2))
3
+
4
(3(1 +

2))
3
= 390
_

_
que tiene la solucin

1
=
3
2
(1 2

2),
2
= 1 + 2

2,
3
=
3
2
(1 + 2

2),
4
= 1 2

2.
As pues tenemos el trmino general
x
n
=
3
2
(1 2

2)(2(1

2))
n
+ (1 + 2

2)(3(1

2))
n
+
3
2
(1 + 2

2)(2(1 +

2))
n
+ (1 2

2)(3(1 +

2))
n
.
La comprobacin es fcil, ya que el trmino general de {a
n
}
n
es
a
n
=
1
2
(1

2)(1 2

2)
n
+
1
2
(1 + 2

2)(1 +

2)
n
y el de {b
n
}
b
es
b
n
= 3 2
n
2 3
n
.
ARITMTICA Y COMBINATORIA
(I. Sucesiones recurrentes)
P. Jara
28 CAP. II. SUCESIONES RECURRENTES
Proposicin. 5.9. (Caso cbico)
Consideramos las sucesiones recurrentes
a
n+3
= a
n+2
c
1
+ a
n+1
c
2
+ a
n
c
3
y condiciones iniciales a
0
, a
1
, a
2
;
b
n+2
= b
n+1
d
1
+ b
n
d
2
y condiciones iniciales b
0
, b
1
.
Y se dene la nueva sucesin recurrente con x
n
= a
n
b
n
. Si
1
,
2
,
3
son las races de la
ecuacin caracterstica r
3
= r
2
c
1
+ rc
2
+ c
3
y si
1
,
2
son las races de la ecuacin caracte-
rstica r
2
= rd
1
+ d
2
, entonces tenemos:
a
n
=
1

n
1
+
2

n
2
+
3

n
3
,
b
n
=
1

n
1
+
2

n
2
,
x
n
=
1

1
(
1

1
)
n
+
1

2
(
1

2
)
n
+
2

1
(
2

1
)
n
+
2

2
(
2

2
)
n
+
3

1
(
3

1
)
n
+
3

2
(
3

2
)
n
(Los casos enque existenraces mltiples se tratande la misma manera.) Comoconsecuencia
el polinomio caracterstico de {x
n
}
n
es:
(r
1

1
)(r
1

2
)(r
2

1
)(r
2

2
)(r
3

1
)(r
3

2
),
la cual se puede expresar en funcin de los polinomios caractersticos originales ya que sus
coecientes son polinomios simtricos en los
i
y polinomios simtricos en los
j
.
28 de enero de 2013 Curso 20122013. NOTAS DE TRABAJO, 30
SEC. 5. SUCESIONES RECURRENTES HOMOGNEAS 29
En nuestro caso tenemos la siguiente expresin:
(r
1

1
)(r
1

2
)(r
2

1
)(r
2

2
)(r
3

1
)(r
3

2
)
=
2
1

2
2

2
3

3
1

3
2

2
1

2
2

3
1

2
2
+
2
1

2
3

3
1

2
2
+
1

2
2

2
3

3
1

2
2
+
2
1

2
2

2
1

3
2
+
2
1

2
3

2
1

3
2
+
1

2
2

2
3

2
1

3
2
_
r
+
_

2
1

3
1

2
+
1

2
2

3
1

2
+
1

2
3

3
1

2
+
2
1

2
2

2
1

2
2
+ 2
2
1

2
1

2
2
+ 2
1

2
2

2
1

2
2
+
2
1

2
3

2
1

2
2
+ 2
1

2
3

2
1

2
2
+
2
2

2
3

2
1

2
2
+
2
1

3
2
+
1

2
2

3
2
+
1

2
3

3
2
_
r
2

3
1
+
2
1

2
1

2
+
1

2
2

2
1

2
+
2
1

2
1

2
+ 3
1

2
1

2
+
2
2

2
1

2
+
1

2
3

2
1

2
+
2

2
3

2
1

2
+
2
1

2
2
+
1

2
2

2
2
+
2
1

2
2
+ 3
1

2
2
+
2
2

2
2
+
1

2
3

2
2
+
2

2
3

2
2
+
1

3
2
_
r
3
+
_

2
1
+
1

2
1
+
2

2
1
+
2
1

2
+ 2
1

2
+
2
2

2
+ 2
1

2
+ 2
2

2
+
2
3

2
+
1

2
2
+
1

2
2
+
2

2
2
_
r
4
(
1

1
+
2

1
+
3

1
+
1

2
+
2

2
+
3

2
) r
5
+ r
6
= c
2
3
d
3
2
+ c
2
c
3
d
1
d
2
2
r (c
1
c
3
(d
2
1
d
2
+ 2d
2
2
) c
2
2
d
2
2
)r
2
(c
2
c
1
(d
1
d
2
) + c
3
(d
3
1
+ 3d
1
d
2
))r
3
(c
2
(d
2
1
+ 2d
2
) + c
2
1
d
2
)r
4
c
1
d
1
r
5
+ r
6
(II.11)
As pues el polinomio caracterstico es:
c
2
3
d
3
2
+c
2
c
3
d
1
d
2
2
r

_
c
1
c
3
(d
2
1
d
2
+ 2d
2
2
) c
2
2
d
2
2
_
r
2

_
c
2
c
1
(d
1
d
2
) + c
3
(d
3
1
+ 3d
1
d
2
)
_
r
3

_
c
2
(d
2
1
+ 2d
2
) + c
2
1
d
2
_
r
4
c
1
d
1
r
5
+r
6
ARITMTICA Y COMBINATORIA
(I. Sucesiones recurrentes)
P. Jara
30 CAP. II. SUCESIONES RECURRENTES
Ejercicio. 5.10.
Se considera las sucesiones recurrentes denidas por
a
0
= 6, a
1
= 5, a
2
= 11 y la frmula a
n+3
= 3a
n+1
+ 2a
n
para n 0 y
b
0
= 1, b
1
= 0 y la frmula b
n+2
= 5b
n+1
6b
n
para n 0.
Se dene la nueva sucesinrecurrente conx
n
= a
n
b
n
. Vamos a averiguar el trmino general
de x
n
y su ecuacin caracterstica.
Primero el polinomio caracterstico es el dado en la frmula (II.11). Como c
1
= 0, c
2
= 3,
c
3
= 2, d
1
= 5, d
2
= 6, tenemos el valor:
r
6
39r
4
70r
3
+ 324r
2
+ 1080r + 864.
Cuyas races son
1
= 3,
2
= 3,
3
= 2,
4
= 2,
5
= 4 y
6
= 6. Los valores iniciales
son:
x
0
= a
0
b
0
= 6 1 = 6,
x
1
= a
1
b
1
= 5 0 = 0,
x
2
= a
2
b
2
= 11 (6) = 66,
x
3
= a
3
b
3
= 27 (30) = 810
x
4
= a
4
b
4
= 43 (114) = 4902
x
5
= a
5
b
5
= 103 (390) = 40170.
Para calcular el trmino general, x
n
= (
1
+n
2
)(3)
n
+ (
3
+n
4
)(2)
n
+
5
4
n
+
6
6
n
, basta
resolver el sistema de ecuaciones lineales:
(
1
+ 0
2
)(3)
0
+ (
3
+ 0
4
)(2)
0
+
5
4
0
+
6
6
0
= 6,
(
1
+ 1
2
)(3)
1
+ (
3
+ 1
4
)(2)
1
+
5
4
1
+
6
6
1
= 0,
(
1
+ 2
2
)(3)
2
+ (
3
+ 2
4
)(2)
2
+
5
4
2
+
6
6
2
= 66,
(
1
+ 3
2
)(3)
3
+ (
3
+ 3
4
)(2)
3
+
5
4
3
+
6
6
3
= 810,
(
1
+ 4
2
)(3)
4
+ (
3
+ 4
4
)(2)
4
+
5
4
4
+
6
6
4
= 4902,
(
1
+ 5
2
)(3)
5
+ (
3
+ 5
4
)(2)
5
+
5
4
5
+
6
6
5
= 40170,
_

_
que tiene la solucin
1
= 6,
2
= 4,
3
= 9,
4
= 6,
5
= 9 y
6
= 6. As pues tenemos el
trmino general
x
n
= (6 + 4n)(3)
n
+ (9 6n)(2)
n
+ 9 4
n
6 6
n
.
La comprobacin es fcil, ya que el trmino general de {a
n
}
n
es
a
n
= (3 2n)(1)
n
+ 3 2
n
,
y el de {b
n
}
n
es
b
n
= 3 2
n
2 3
n
.
28 de enero de 2013 Curso 20122013. NOTAS DE TRABAJO, 30
SEC. 6. SUCESIONES RECURRENTES NO HOMOGNEAS 31
6. Sucesiones recurrentes no homogneas
Las sucesiones recurrentes no homogneas son las soluciones a ecuaciones del tipo
a
n
= a
n1
c
1
+ + a
nt
c
t
+ f (n), (n t), (II.12)
en donde f (n) es una funcin de n.
Para distinguir esta sucesin recurrente de las sucesiones antes introducidas, las vamos a
llamar sucesiones recurrentes no homogneas.
Las sucesiones recurrentes no homogneas no constituyen un espacio vectorial, como fcil-
mente se puede observar. Pero s podemos considerar el espacio vectorial de las sucesiones
recurrentes (homogneas) asociado; esto es, el denido por la ecuacin (homognea)
a
n
= a
n1
c
1
+ + a
nt
c
t
, (n t), (II.13)
de forma que cada sucesin recurrente que verica la ecuacin (II.12) se va a obtener como
una combinacin lineal de una solucin particular de (II.12) y una combinacin lineal de
sucesiones en el espacio vectorial denido por la ecuacin (II.13).
Una construccin de sucesiones recurrentes. La suma
Supongamos que tenemos dos sucesiones recurrentes, {a
n
}
n
y {b
n
}
n
dadas por las ecuacio-
nes de recurrencia
a
n
= a
n1
c
1
+ + a
nt
c
t
b
m
= b
m1
d
1
+ + b
ms
d
s
podemos suponer que tenemos trminos generales dados por expresiones del tipo siguiente:
a
n
=

h
i=1
p
i

n
i
,
b
n
=

k
j=1
q
j

n
j
en donde p
i
y q
j
son polinomios en n, cuyo grado depende de la multiplicidad de la raz
i

j
, respectivamente en el polinomio (ecuacin) caracterstica.
Denimos una nueva sucesin mediante:
x
n
= a
n
+ b
n
, para cada ndice n.
El trmino general de la nueva sucesin {x
n
}
n
es la suma de los trminos de las sucesiones.
x
n
=

i
p
i

n
i
+

j
q
j

n
j
,
entonces el problema es el clculo de la ecuacin caracterstica de {x
n
}
n
; sta depende de los
polinomios p
i
y q
j
que aparezcan y de las races
i
y
j
.
ARITMTICA Y COMBINATORIA
(I. Sucesiones recurrentes)
P. Jara
32 CAP. II. SUCESIONES RECURRENTES
Cuando las races
i
y
j
son distintas, resulta que el polinomio caracterstico de la sucesin
{x
n
}
n
, es exactamente el producto de los polinomios caractersticos de {a
n
}
n
y {b
n
}
b
.
Cuando existe una raz comn, entonces el polinomio caracterstico se obtiene a partir del
producto de los polinomios caractersticos, pero considerando las races comunes con la m-
xima multiplicidad.
En cualquier caso podemos siempre suponer que las races son distintas y tomar como po-
linomio caracterstico el producto de los dos polinomios caractersticos; la razn es que en
cualquier caso la sucesin {x
n
}
n
pertenece al espacio vectorial determinado por el producto
de los dos polinomios.
Ejercicio. 6.1.
Consideramos las sucesiones
{a
n
}
n
denida por a
0
= 6, a
1
= 5, a
2
= 11 y a
n
= 3a
n2
+ 2a
n3
, n 3 y
{b
n
}
n
denida por b
0
= 1, b
1
= 0 y b
n
= 5b
n1
6b
n2
, n 2.
Se dene la nueva sucesin recurrente con x
n
= a
n
+ b
n
. Vamos a averiguar el trmino
general de x
n
y su ecuacin caracterstica.
Primero calculamos el trmino general de {a
n
}
n
, que es: a
n
= (3 2n)(1)
n
+ 3 2
n
; su
polinomio caracterstico es: r
3
3r 2 = (r + 1)
2
(r 2).
Ahora calculamos el trmino general de {b
n
}
n
, que es: b
n
= 3 2
n
3 3
n
; su polinomio
caracterstico es: r
2
5r + 6 = (r 2)(r 3).
El trmino general de {x
n
}
n
es:
x
n
= (3 2n)(1)
n
+ 3 2
n
+ 3 2
n
3 3
n
= (3 2n)(1)
n
+ 6 2
n
3 3
n
.
Su polinomio caracterstico es (r + 1)
2
(r 2)(r 3), pero es tambin una sucesin asociada
a (r + 1)
2
(r 2)
2
(r 3), y por tanto el trmino general de {x
n
}
n
podemos obtenerlo a partir
de cualquiera de estos dos polinomios caractersticos.
Clculo del trmino general
Veamos una aplicacin de la suma de sucesiones recurrentes. Se considera una sucesin re-
currente no homognea
a
n
= a
n1
c
1
+ + a
nt
c
t
+ f (n), (n t), (II.14)
si f (n) es el trmino general de una sucesin recurrente con polinomio caracterstico Q(r),
entonces podemos proceder como sigue: consideramos la frmula de recurrencia homog-
nea
a
n
= a
n1
c
1
+ + a
nt
c
t
, (n 0), (II.15)
el polinomio caracterstico que dene, sea P(r) y tomamos una sucesin recurrente del es-
pacio vectorial asociado, sea {h
n
}
n
. Entonces a
n
se puede escribir
a
n
= h
n
+ f (n),
28 de enero de 2013 Curso 20122013. NOTAS DE TRABAJO, 30
SEC. 6. SUCESIONES RECURRENTES NO HOMOGNEAS 33
esto es, {a
n
}
n
es la suma de dos sucesiones: {h
n
}
n
y {f (n)}
n
; por lo tanto, cuando {f (n)}
n
es una sucesin recurrente, conocemos que la sucesin {a
n
}
n
es una sucesin recurrente
asociada al polinomio P(r)Q(r).
Este simple argumento nos permite calcular fcilmente una solucin particular de la recu-
rrencia no lineal (II.14), pues sabemos calcular soluciones de recurrencias homogneas. En
este clculo, por comodidad, los sumandos que corresponden a P(r) podemos eliminarlos
al obtener la solucin particular, ya que estos aparecern de nuevo al considerar la solucin
general.
Veamos un ejemplo.
Ejemplo. 6.2.
Se considera la relacin de recurrencia no homognea
a
n
= 3a
n2
+ 2a
n3
+ 2n + 5n
4
,
sujeta a las condiciones iniciales a
0
= 1, a
1
= 2 y a
2
= 3.
Vamos a calcular una solucinparticular. Consideramos la ecuacincaracterstica que dene
una sucesin con 2n + 5n
4
como trmino general; en este caso (r 1)
5
.
Tenemos que {a
n
}
n
es la suma de dos sucesiones, una denida por la relacin h
n
= 3h
n2
+
2h
n3
, con polinomio caracterstico r
3
3r 2 = (r + 1)
2
(r 2), y otra por los trminos
2n + 5n
4
. Entonces un polinomio caracterstico para {a
n
}
n
es:
(r 1)
5
(r + 1)
2
(r 2)
Una solucin genrica es:
(
1
+
2
n +
3
n
2
+
4
n
3
+
5
n
4
) + (
6
+
7
n)(1)
n
+
8
2
n
Como antes hemos sealado, la parte (
6
+
7
n)(1)
n
+
8
2
n
podemos eliminarla, ya que la
encontraremos de nuevo al obtener la solucin general; as pues trabajamos con
1
+
2
n +

3
n
2
+
4
n
3
+
5
n
4
.
Al imponer que verica la ecuacin de denicin se tiene:

1
+
2
n +
3
n
2
+
4
n
3
+
5
n
4
= 3(
1
+
2
(n 2) +
3
(n 2)
2
+
4
(n 2)
3
+
5
(2)n
4
)
+2(
1
+
2
(n 3) +
3
(n 3)
2
+
4
(n 3)
3
+
5
(n 3)
4
)
+2n + 5n
4
.
Al desarrollar se obtiene
4
1
+ 12
2
30
3
+ 78
4
210
5
+(2 4
2
+ 24
3
90
4
+ 312
5
)n
+(4
3
+ 36
4
180
5
)n
2
+(4
4
+ 48
5
)n
3
+(5 4
5
)n
4
= 0
ARITMTICA Y COMBINATORIA
(I. Sucesiones recurrentes)
P. Jara
34 CAP. II. SUCESIONES RECURRENTES
De aqu se deducen las relaciones:
4
1
+12
2
30
3
+78
4
210
5
= 0
2 4
2
+24
3
90
4
+312
5
= 0
4
3
+36
4
180
5
= 0
4
4
+48
5
= 0
5 4
5
= 0
_

_
que tiene como solucin:

1
=
1341
4
,
2
= 233,
3
=
315
4
,
4
= 15,
5
=
5
4
.
Entonces una solucin particular es:

1341
4
233n
315
4
n
2
15n
3

5
4
n
4
La solucin general es la que se obtiene al sumar a la solucin particular una solucin general
de h
n
= 3h
n2
+ 2h
n3
, cuya ecuacin caracterstica, como ya sabemos, es: r
3
3r 2 =
(r + 1)
2
(r 2). Por tanto la solucin general de la homognea es:
(
1
+
2
n)(1)
n
+
3
2
n
y una solucin general de la recurrencia es:

1341
4
233n
315
4
n
2
15n
3

5
4
n
4
+ (
6
+
7
n)(1)
n
+
8
2
n
Para calcular
6
,
7
y
8
tenemos considerar las condiciones iniciales:

1341
4
+
6
+
8
= 1

2653
4

6

7
+2
8
= 2

5025
4
+
6
+2
7
+4
8
= 3
_

_
La solucin es:
6
=
401
36
,
7
=
157
6
,
8
=
2926
9
.
La solucin general de la recurrencia no homognea es:

1341
4
233n
315
4
n
2
15n
3

5
4
n
4
+
_
401
36

157
6
n
_
(1)
n
+
2926
9
2
n
28 de enero de 2013 Curso 20122013. NOTAS DE TRABAJO, 30
SEC. 7. FUNCIONES GENERATRICES 35
7. Funciones generatrices
Sea {a
n
}
n
una sucesin de nmeros reales o complejos, podemos denir entonces una fun-
cin formal mediante
f (x) = a
0
+ a
1
x + a
2
x
2
+ a
3
x
3
+
Recordar que esta funcinformal no dene una funcinde Ra R de Ca C, ya que engeneral
no est denida la suma innita. Casos particulares se presentan cuando la sucesin {a
n
}
n
es nula a partir de un ndice; en este caso f (x) est denido por un polinomio. Otro caso es
cuando la serie que dene f (x) es convergente, entonces f (x) estar denida en una parte de
R C, segn el caso.
Nosotros estamos interesados en los coecientes, o los trminos de la sucesin que denen
a f (x), por lo que llamamos a f (x) la funcin generatriz de la sucesin {a
n
}
n
.
Veamos un ejemplo.
Ejemplo. 7.1.
Si consideramos la sucesin constante igual a 1 la funcin generatriz es: f (x) = 1 +x +x
2
+
x
3
+ , que corresponde al desarrollo en serie de la funcin x
1
1x
.
Casos ms sencillos se plantean cuando consideramos otras funciones.
Ejemplo. 7.2.
La funcin
x (1 + x)
n
=
_
n
0
_
+
_
n
1
_
x
+
+
_
n
n 1
_
x
n1
+
_
n
n
_
x
n
es la funcin generatriz cuya sucesin es:
_
n
0
_
,
_
n
1
_
, . . . ,
_
n
n 1
_
,
_
n
n
_
, 0, 0, . . .
ARITMTICA Y COMBINATORIA
(I. Sucesiones recurrentes)
P. Jara
36 CAP. II. SUCESIONES RECURRENTES
Ejemplo. 7.3.
La funcin
x (1 x)
n
=
_
n
0
_

_
n
1
_
x + + (1)
n1
_
n
n 1
_
x
n1
+ (1)
n
_
n
n
_
x
n
es la funcin generatriz cuya sucesin es:
_
n
0
_
,
_
n
1
_
, . . . , (1)
n1
_
n
n 1
_
, (1)
n
_
n
n
_
, 0, 0, . . .
Como se verica 1 x
n
= (1 x)(1 + x + + x
n1
), entonces la funcin
x
1 x
n
1 x
= 1 + x + + x
n1
,
resulta que es la funcin generatriz de la sucesin 1, 1, . . . , 1, 0, 0 ,. . . , en donde aparecen
exactamente n unos.
Llevado el proceso al lmite se obtiene que la funcin x
1
1x
es la funcin generatriz de
la sucesin innita en la que los coecientes de todos los trminos son iguales a uno, como
hemos sealado anteriormente.
Realizando modicaciones a estas funciones obtenemos nuevas funciones generatrices.
Ejemplo. 7.4.
Si derivamos
1
1x
con respecto a x se obtiene:
d
dx
1
1 x
=
1
(1 x)
2
= 1 + 2x + 3x
2
+ 4x
3
+
La sucesin asociada es: 1, 2, 3, 4, 5, . . . Al multiplicar por x se tiene
x
(1x)
2
, que es la funcin
generatriz de la sucesin: 0, 1, 2, 3, . . .
Ejemplo. 7.5.
Si derivamos esta nueva funcin, obtenemos:
d
dx
x
(1 x)
2
=
1 + x
(1 x)
3
= 1 + 2
2
x + 3
2
x
2
+ 4
2
x
3
+
La sucesin asociada es: 1, 2
2
, 3
2
, 4
2
, 5
2
, . . .
28 de enero de 2013 Curso 20122013. NOTAS DE TRABAJO, 30
SEC. 7. FUNCIONES GENERATRICES 37
Un problema domstico
Ejemplo. 7.6.
Mara tiene un montn de 12 monedas que quiere repartir entre tres huchas, sean A, B y C.
De cuntas formas lo puede hacer si en la hucha A tiene que introducir al menos 4 mone-
das, en las huchas B y C tiene que introducir al menos 2 monedas y en la hucha C no puede
introducir ms de 5 monedas?
SOLUCIN. Podramos hacer una tabla y considerar todos los casos de forma exhaustiva, a
modo de ejemplo tenemos:
A B C A B C A B C A B C A B C
4 3 5 5 2 5 6 2 4 7 2 3 8 2 2
4 4 4 5 3 4 6 3 3 7 3 2
4 5 3 5 4 3 6 4 2
4 6 2 5 5 2
Existe otra forma alternativa, que consiste en considerar:
para la hucha A el polinomio x
4
+ x
5
+ x
6
+ x
7
+ x
8
; sus coecientes son todos iguales
a uno, que corresponde a la condicin de que en la hucha A tiene que haber al menos 4
monedas.
para la hucha B el polinomio x
2
+x
3
+x
4
+x
5
+x
6
; sus coecientes son todos iguales a
uno. Como entre las huchas A y C tiene que haber al menos 6 monedas, quedan, como
mximo, 6 monedas para la hucha B, y como al menos tiene que haber 2, de ah el
polinomio que se considera.
para la hucha C el polinomio x
2
+x
3
+x
4
+x
5
; sus coecientes son todos iguales a uno
que corresponde a la condicin de que en la hucha Ctiene que haber al menos 2 y como
mximo 5 monedas.
Al realizar el producto
(x
4
+ x
5
+ x
6
+ x
7
+ x
8
)(x
2
+ x
3
+ x
4
+ x
5
+ x
6
)(x
2
+ x
3
+ x
4
+ x
5
)
= x
19
+ 3x
18
+ 6x
17
+ 10x
16
+ 14x
15
+ 16x
14
+ 16x
13
+ 14x
12
+ 10x
11
+ 6x
10
+ 3x
9
+ x
8
.
El coeciente de x
12
es justamente el nmero que andamos buscado, en este caso 14. La
ventaja es que es ms sencillo realizar el producto de los tres polinomios para determinar
este coeciente que hacer un tratamiento combinatorio de este problema.
Ejemplo. 7.7.
De cuntas formas es posible repartir 10 monedas entre tres personas si cada una debe
recibir al menos 2 monedas?
ARITMTICA Y COMBINATORIA
(I. Sucesiones recurrentes)
P. Jara
38 CAP. II. SUCESIONES RECURRENTES
SOLUCIN. Siguiendo el anterior mtodo de los polinomios, basta determinar el coeciente
de x
10
en el desarrollo de (x
2
+ x
3
+ x
4
+ x
5
+ x
6
)
3
. Como el desarrollo es:
x
6
+ 3x
7
+ 6x
8
+ 10x
9
+ 15x
10
+ 18x
11
+ 19x
12
+ 18x
13
+ 15x
14
+ 10x
15
+ 6x
16
+ 3x
17
+ x
18
el valor pedido es 15.
Series de Maclaurin
Se trata de calcular el desarrollo de 1/(1 + x)
n
, o equivalentemente de (1 + x)
n
, con n N.
Para esto introducimos los nmeros combinatorios
_
n
r
_
, para n, r N.
Se dene
_
n
r
_
=
(n)(n1)(nr+1)
r!
= (1)
r
_
n+r1
r
_
.
Entonces la serie de Maclaurin de (1 + x)
n
es:
(1 + x)
n
= 1 + (n)x +
(n)(n1)
2!
x
2
+
(n)(n1)(n2)
3!
x
3
+
= 1 +

r=1

(n)(n1)(nr+1)
r!
x
r
=

r=0
(1)
r
_
n+r1
r
_
x
r
=

r=0
_
n
r
_
x
r
,
que es la expresin anloga al desarrollo del binomio (1 + x)
n
.
Por extensin se dene la expansin en serie de Maclaurin de (1+x)
r
para cualquier nmero
real r.
Series de Fibonacci
Consideramos la sucesin de Fibonacci:
a
0
= 0, a
1
= 1, a
n+2
= a
n+1
+ a
n
, si n 0.
La funcin generatriz denida por esta sucesin es:
f (x) = a
0
+ a
1
x + a
2
x
2
+ = 0 + x + x
2
+ 2x
3
+ 3x
4
+
La denicin de la sucesin de Fibonacci nos proporciona la siguiente relacin entre las fun-
ciones generatrices:
f (x)xf (x)x
2
f (x) = a
0
+(a
1
a
0
)x+(a
2
a
1
a
0
)x
2
+ +(a
n+2
a
n+1
a
n
)x
n+2
+ = x.
En consecuencia se tiene la serie de Fibonacci:
f (x) =
x
1 x x
2
.
28 de enero de 2013 Curso 20122013. NOTAS DE TRABAJO, 30
SEC. 7. FUNCIONES GENERATRICES 39
Los coecientes de f (x) se puede determinar ahora atendiendo a esta fraccin.
Mtodo 1. Descomponiendo en fracciones simples:
x
1 x x
2
=
A
x
+
B
x
=
A( x) + B( x)
( x)( x)
.
Se tiene x
2
+ x 1 = (x )(x ), donde =
1+

5
2
y =
1

5
2
, con relaciones:
= 1, + = 1 y =

5.
Adems se tiene
x = A( x) + B( x) = (A + B) (A + B)x,
de donde A + B = 1 y A( x) + B( x) = 0, entonces A =

y B =

. En consecuencia
se tiene:
x
1xx
2
=

()(x)
+

()(x)
=
1
()(1
x

)

1
()(1
x

)
=
1

5(1
x

)

1

5(1
x

)
.
Basta entonces desarrollar en serie la fraccin
1
(1
x

)
para obtener los trminos de la sucesin
de Fibonacci.
1
1
x

= 1 +
x

+
_
x

_
2
+ ,
x
1xx
2
=
1

5(1
x

)

1

5(1
x

)
=
1

5
_
1 +
x

+
_
x

_
2
+
_

5
_
1 +
x

+
_
x

_
2
+
_
=
1

5
_
1

_
x +
1

5
_
_
1

_
2

_
1

_
2
_
x
2
+
Finalmente observar que = y = , ver pgina 23. Por lo tanto se tiene
1

= y
1

= ,
y obtenemos la frmula de Binet, ver pgina 23.
a
n
=
1

5
_
1

n

1

n
_
=

5
5
(
n

n
) .
Mtodo 2. Escribimos
x
1 x x
2
=
x
1 x(1 + x)
.
Consideramos el desarrollo
1
1x
= 1 + x + x
2
+ , y aplicando a este caso se tiene:
x
1x(1+x)
= x(1 + x(1 + x) + x
2
(1 + x)
2
+ x
3
(1 + x)
3
+ )
ARITMTICA Y COMBINATORIA
(I. Sucesiones recurrentes)
P. Jara
40 CAP. II. SUCESIONES RECURRENTES
= x
x
2
+ x
3
x
3
+2x
4
+ x
5
x
4
+3x
5
+3x
6
+ x
7
x
5
+4x
6
+6x
7
+4x
8
+x
9

= x +x
2
+2x
3
+3x
4
+5x
5
+
28 de enero de 2013 Curso 20122013. NOTAS DE TRABAJO, 30
SEC. 8. EJERCICIOS CON SOLUCIN 41
8. Ejercicios con solucin
Sucesiones recurrentes
Ejercicio. 8.1.
Se considera la sucesin recurrente denida por
a
0
= 3, a
1
= 3, a
n+2
= 6a
n+1
9a
n
, para n 0.
Calcular el trmino general.
SOLUCIN
SOLUCIN. Ejercicio (8.1.)
La ecuacin caracterstica es r
2
6r +9 = 0, que tiene una raz = 3 demultiplicidad dos. El
trmino de una solucin general es:
a
n
=
1
3
n
+
2
n3
n
,
Las condiciones iniciales son: a
0
= 3 y a
1
= 3.
Al resolver el sistema
3 =
1
,
3 =
1
3 +
2
3
_
,
llegamos a las soluciones
1
= 3,
2
= 2, y por tanto la sucesin que es solucin es de este
problema es:
a
n
= 3 3
n
2 n3
n
= (3 2n)3
n
.

Ejercicio. 8.2.
Se considera una sucesin de Fibonacci, esto es, a
n+2
= a
n+1
+a
n
, siendo a
0
y a
1
arbitrarios.
Si todos los a
n
son no nulos, demuestra que se verica:
1
a
n1
a
n

1
a
m
a
m+1
=
m

j=n
1
a
j1
a
j+1
, n m.
SOLUCIN
SOLUCIN. Ejercicio (8.2.)Llamamos t = m n, y hacemos induccin sobre t. Si t = 0, se
tiene:
1
a
n1
a
n

1
a
n
a
n+1
=
a
n+1
a
n1
a
n1
a
n
a
n+1
=
a
n
+ a
n1
a
n1
a
n1
a
n
a
n+1
=
a
n
a
n1
a
n
a
n+1
=
1
a
n1
a
n+1
ARITMTICA Y COMBINATORIA
(I. Sucesiones recurrentes)
P. Jara
42 CAP. II. SUCESIONES RECURRENTES
Supongamos que el resultado es cierto para t y vamos a ver qu ocurre para mn = t + 1.
1
a
n1
a
n

1
a
m
a
m+1
=
1
a
n1
a
n

1
a
n+t+1
a
n+t+2
=
1
a
n1
a
n

1
a
n+t
a
n+t+1
+
1
a
n+t
a
n+t+1

1
a
n+t+1
a
n+t+2
=

n+t
j=n
1
a
j1
a
j+1
+
1
a
n+t
a
n+t+2
=

n+t+1
j=n
1
a
j1
a
j+1
=

m
j=n
1
a
j1
a
j+1
.
Tomar el ejemplo: a
0
= 1, a
1
= 1, entonces
n 0 1 2 3 4 5 6 7 8 9 10 11
a
n
1 1 2 3 5 8 13 21 34 55 89 144
a
n1
a
n
1 2 6 15 40 104 273 714 1870 4895 12816
a
n1
a
n+1
2 3 10 24 65 168 442 1155 3026 7920
Tomando n = 5 y m = 9 se obtiene
1
a
4
a
5
=
1
a
9
a
10
+
9

j=5
1
a
j1
a
j+1
.
1
a
4
a
5
=
1
a
9
a
10
+
1
a
4
a
6
+
1
a
5
a
7
+
1
a
6
a
8
+
1
a
7
a
9
+
1
a
8
a
10
1
40
=
1
4895
+
1
65
+
1
168
+
1
442
+
1
1155
+
1
3026

Ejercicio. 8.3.
REDACTAR.
SOLUCIN
SOLUCIN. Ejercicio (8.3.)
HACER.
Ejercicio. 8.4. (Espiral de Fibonacci)
Se considera la siguiente construccin de cuadrados:
28 de enero de 2013 Curso 20122013. NOTAS DE TRABAJO, 30
SEC. 8. EJERCICIOS CON SOLUCIN 43
Observa que el lado de cada uno de los cuadrados vara segn la sucesin de Fibonacci: 1, 1,
2, 3, 5, 8, 13, 21, 34, 55, . . .
La curva obtenida se conoce como la espiral de Durero y recuerda al desarrollo del Nautilus.
SOLUCIN
SOLUCIN. Ejercicio (8.4.)
HACER.
ARITMTICA Y COMBINATORIA
(I. Sucesiones recurrentes)
P. Jara
44 CAP. II. SUCESIONES RECURRENTES
Ejercicio. 8.5. (Nmeros de Perrin)
La sucesin a
n
denida por
a
0
= 3, a
1
= 0, a
2
= 2, a
n+3
= a
n+1
+ a
n
fue introducida por Edouard Lucas (Amer. J. Math., vol. 1 (1978)). Aos despus fue utilizada
por R. Perrin. En1982 DanShanks y Bill Adams enMath. of Computation, vol. 39 (1982) hacen
un estudio de esta sucesin a sus trminos le dan el nombre de nmeros de Perrin.
Determina la funcin generatriz de la sucesin de los nmeros de Perrin.
De forma anloga a la espiral de cuadrados de Fibonacci se tiene una espiral de tringulos
equilteros; vamos a ver cmo asociada a la misma tenemos una sucesin de nmeros de
Perrin.
SOLUCIN
SOLUCIN. Ejercicio (8.5.)
Si f (x) es la funcin generatriz, se verica:
f (x) a
0
+a
1
x+a
2
x
2
+a
3
x
3
+a
4
x
4
+
x
2
f (x) a
0
x
2
+a
1
x
3
+a
2
x
4
+
x
3
f (x) a
0
x
3
+a
1
x
4
+
sumando se tiene
_
1 x
2
x
3
_
f (x) = a
0
+ a
1
x + (a
2
a
0
)x
2
= 3 x
2
En consecuencia
f (x) =
1 x
2
1 x
2
x
3
es la funcin generatriz.
Observa la siguiente construccin con tringulos equilteros:
28 de enero de 2013 Curso 20122013. NOTAS DE TRABAJO, 30
SEC. 8. EJERCICIOS CON SOLUCIN 45
A
Se tiene h
0
+ h
1
= h
3
, h
1
+ h
2
= h
4
, etc. Por lo tanto las alturas de los tringulos estn en la
misma progresin que los nmeros de Perrin. Esto nos debera permitir calcular la relacin
entre h
n
y h
n+1
. Primero observa que la ecuacin caracterstica de la sucesin de Perrin es:
X
3
= X + 1.
Calcula el rea de la regin marcada con la letra A.
Determina la expresin general del trmino nsimo de la sucesin de nmeros de Perrin.

ARITMTICA Y COMBINATORIA
(I. Sucesiones recurrentes)
P. Jara
46 CAP. II. SUCESIONES RECURRENTES
28 de enero de 2013 Curso 20122013. NOTAS DE TRABAJO, 30
Captulo III
Ejercicios de repaso
9. Ejercicios de repaso
Ejercicios de repaso
Problema. 9.1.
Dados enteros positivos a
0
, a
1
, . . . , a
100
vericando:
_

_
a
1
> a
0
,
a
2
= 3a
1
2a
0
,
. . .
a
100
= 3a
99
2a
98
.
_

_
Probar que a
100
> 2
99
.
[The problems of the All-Soviet-Unionmathematical competitions 1961-1986. Problem015]
SOLUCIN. Supongamos que a
1
= a
0
+ h para algn entero positivo h. Entonces
a
2
= 3a
1
2a
0
= 3(a
0
+ h) 2a
0
= a
0
+ 3h.
En general se tiene entonces a
n
= a
0
+ f
n
h, para algn entero positivo f
n
. La relacin de
denicin de la sucesin f
n
es la misma que para a
n
, siendo f
1
= 1 y f
2
= 3, f
n+2
= 3f
n1
2f
n
.
La sucesin f
n
tiene por ecuacin caracterstica r
2
= 3r 2. Las races son
1
= 1 y
2
= 2.
Entonces una base del espacio de las sucesiones recurrentes es: {1}
n
y {2
n1
}
n
. El trmino
general f
n
se expresa f
n
=
1
1 +
2
2
n1
. Tenemos entonces
1 =
1
+
2
,
3 =
1
+
2
2
_
La solucin es
1
= 1 y
1
= 2, y tenemos f
n
= 1 + 2 2
n1
= 2
n
1.
Se tiene entonces a
n
= a
0
+ f
n
h = a
0
+ (2
100
1)h > 2
99
.

47
48 CAP. III. EJERCICIOS DE REPASO
Problema. 9.2.
Se considera la sucesin {a
n
}
n
denida por:
a
1
= 1, a
2
= 2, a
n+2
= 2a
n+1
+ a
n
, para n 2.
Esta sucesin comienza:
1, 2, 5, 12, 29, 70, 169, 408, 985, 2378, 5741, 13860, . . .
Se observa fcilmente que los trminos a
2h
son los nicos que son mltiplos de 2, que los
trminos a
4h
son los nicos que son mltiplos de 4, y que los trminos a
8h
son los nicos
que son mltiplos de 8.
Se pide probar que para cada entero positivo t se tiene que a
n
es mltiplo de 2
t
si y slo si n
es mltiplo de 2
t
.
[Crux Mathematicorum, 1988, 225. FromErds to Kiev. Problems of Olympiad Caliber. MAA,
1996, pag. 79]
SOLUCIN. La ecuacin de recurrencia es a
n+2
= 2a
n+1
+a
n
, luego la ecuacin caracterstica
es: r
2
= 2r + 1. Las races de la ecuacin r
2
r 1 = 0 son:
1
= 1 +

2 y
2
= 1

2.
El trmino general de la sucesin a
n
es: a
n
=
1

n1
1
+
2

n1
2
. De donde resulta:
1 =
1
+
2
,
2 =
1

1
+
2

2
Los valores son:
1
=

2(1+

2)
4
=

2
4

1
y
2
=

2(1

2)
4
=

2
4

2
. Y el trmino general se
expresa:
a
n
=

2
4

n
1

2
4

n
2
=

2
4
(1 +

2)
n

2
4
(1

2)
n
=

2
4
_
(1 +

2)
n
(1

2)
n
_
(n 1).
Cuando n es par, por ejemplo n = 2h tenemos:
a
2h
=

2
4
_
(1 +

2)
2h
(1

2)
2h
_
=

2
4
_
(1 +

2)
h
+ (1

2)
h
__
(1 +

2)
h
(1

2)
h
_
La expresin (1 +

2)
h
+ (1

2)
h
es mltiplo de 2 y no de 4.
(1 +

2)
h
+ (1

2)
h
= 2

h
i=0;par
_
h
i
_

2
i
= 2(1 +
_
h
2
_
2 + )
28 de enero de 2013 Curso 20122013. NOTAS DE TRABAJO, 30
SEC. 9. EJERCICIOS DE REPASO 49
Como consecuencia la expresin de a
2h
es:
a
2h
= 2 X a
h
, siendo X un nmero impar
Como a
n
es impar si y solo si n es impar, entonces tenemos el resultado haciendo induccin
sobre n.
Problema. 9.3.
Se considera una bandera, la cual se puede pintar, por franjas horizontales, de cuatro colo-
res, sean A, B, C y D. Queremos averiguar cul es el nmero total de banderas que podemos
pintar con n franjas atendiendo a las siguientes condiciones:
(1) Cada franja est pintada de un color.
(2) Cada franja est pintada de un color y dos franjas contiguas lo estn de colores distintos.
(3) Cada franja est pintada de un color, dos franjas contiguas estn pintadas de colores
distintos y la franja superior y la franja inferior estn pintadas de distinto color.
SOLUCIN.
(1). Es claro que cada franja puede ser pintada de cualquiera de los cuatro colores, luego el
nmero total de banderas de n franjas es: 4
n
.
(2). La primera franja se puede pintar con cualquiera de los cuatro colores, en cambio la
segunda, tercera, etc. slo pueden pintarse de tres de los cuatro colores, luego el nmero
total de banderas es: 4 3
n1
.
(3). Vamos a averiguar el nmero de banderas a
n
que podemos pintar con n franjas, supo-
niendo que conocemos las banderas que se pueden pintar con menos franjas.
De cualquier bandera de n 1 franjas podemos obtener otra de n franjas sin ms que pintar
una n-sima franja de un color distinto a la primera franja y a la ltima franja; el nmero total
de banderas as obtenido es 2a
n1
.
De cualquier bandera de n 2 franjas podemos obtener otra de n franjas sin ms que pintar
la franja n 1 del mismo color que la primera y la franja n-sima de un color diferente; el
nmero total de banderas as obtenido es 3a
n2
.
Es claro que cada bandera de n franjas vericando las condiciones del enunciado est entre
las consideradas, ya que si la franja primera y penltima son distintas, se obtiene a partir de
la primera construccin, y si son iguales a partir de la segunda.
Tenemos entonces a
n
= 2a
n1
+ 3a
n2
para n 4. Los valores iniciales son: a
2
= 12; las
banderas son: AB, AC, AD, BC, BD, CD y las que se obtienen cambiando el orden, en total 12.
El valor de a
3
es 24, ya que tenemos las banderas ABC, ABD, ACD y BCD y todas las que se
obtienen permutando los colores, en total 46 = 24. Los valores de a
0
y a
1
son iguales a cero;
podemos no considerarlos en nuestra sucesin.
La ecuacin caracterstica es: r
2
= 2r + 3, sus races son:

1
= 1 y
2
= 3.
El trmino general es:
a
n
=
1
(1)
n
+
2
(3)
n
,
ARITMTICA Y COMBINATORIA
(I. Sucesiones recurrentes)
P. Jara
50 CAP. III. EJERCICIOS DE REPASO
que da las ecuaciones:
_
12 =
1
(1)
2
+
2
(3)
2

1
= 3
24 =
1
(1)
3
+
2
(3)
3

2
= 1
a
n
= 3(1)
n
+ 3
n
, para n 2.

Problema. 9.4.
Andrs tiene n euros para gastar. Puede gastarlos:
- en caramelos; una bolsa de caramelos le cuesta un euro, o
- en pasteles; hay dos tipos de pasteles y cada uno cuesta dos euros.
De cuntas formas distintas puede Andrs gastar los n euros? (Nota. Importa el orden en
que Andrs hace el gasto.)
SOLUCIN. Llamamos a
n
al nmero de formas en que Andrs puede gastar n monedas. Vea-
mos qu relaciones verica a
n
.
(1) Si el ltimo gasto de Andrs fue de un euro, slo lo puede gastar de una forma, luego te-
nemos a
n1
formas de gastar n euros.
(2) Si el ltimo gasto de Andrs fue de dos euros, puede gastarlo en uno de los dos tipos de
pasteles, luego tenemos 2a
n2
formas de gastar n euros.
La suma de estas posibilidades ser el nmero que andamos buscando:
a
n
= a
n1
+ 2a
n2
Tenemos adems los siguientes casos: a
0
= 1, a
1
= 1 y a
2
= 3. As pues se trata de averiguar
el trmino general de una sucesin recurrente de orden 2.
La ecuacin caracterstica es: r
2
= r + 2, y sus races son:
1
= 1,
2
= 2.
La sucesin recurrente general es: a
n
=
1
(1)
n
+
2
2
n
. Que verica:
1 =
1
+
2
,
1
=
1
3
1 =
1
(1) +
2
2,
2
=
2
3
a
n
=
1
3
(1)
n
+
2
3
2
n
=
1
3
(1)
n
+
1
3
2
n+1
.

Problema. 9.5.
Calcular el nmero de listas de longitud n, formadas por los elementos 0, 1, y 2, en las que
no aparezcan dos ceros seguidos.
SOLUCIN. Llamamos a
n
el nmero de listas de longitud n vericando las propiedades del
enunciado.
(1) Si a una lista, en las condiciones del enunciado, de longitud n 1 le aadimos 1 2,
entonces tenemos una lista vlida de longitud n; el nmero total es 2a
n1
,
28 de enero de 2013 Curso 20122013. NOTAS DE TRABAJO, 30
SEC. 9. EJERCICIOS DE REPASO 51
(2) Si a una lista, en las condiciones del enunciado, de longitud n 2 le aadimos 10 20,
entonces tenemos una lista vlida de longitud n.
Como cualquier lista vlida de longitud n se obtiene de una de estas formas, resulta la igual-
dad:
a
n
= 2a
n1
+ 2a
n2
,
Se tienen adems las siguiente condiciones iniciales: a
0
= 1, a
1
= 3, a
2
= 9 1 = 8, ya que
las listas 11, 12, 22, 10, 01, 20, 02, 21 son vlidas, todas menos la 00.
La ecuacin caracterstica es: r
2
= 2r + 2, y las races son:
1
= 1 +

3,
2
= 1

3.
La sucesin recurrente general es: a
n
=
1
(1 +

3)
n1
+
2
(1

3)
n1
. Que verica adems:
1 =
1
+
2
,
1
=
3+2

3
6
3 =
1
(1 +

3) +
2
(1

3),
2
=
32

3
6
La expresin del trmino general es:
a
n
=
3 + 2

3
6
(1 +

3)
n
+
3 2

3
6
(1

3)
n
.

Ejercicio. 9.6.
Se considera el conjunto de todas las listas de longitud n en el alfabeto {0, 1, 2}.
(1) Cuntas listas verican que cada elemento es menor o igual que anterior?
(2) Cuntas son capicas?
(3) Cuantas contienen dos elementos consecutivos iguales?
(4) Cuntas hay que que no tienen ni dos unos ni dos doses consecutivos?
SOLUCIN.
(1) Llamamos a
n
al nmero de listas en las que cada elemento es menor o igual que el an-
terior. Observa que de cada lista de este tipo de n elementos podemos formar una con
n + 1, basta aadir un elemento 0 al nal. Observa que solo hay una lista con 2 al nal, y
que sta est formada solo por elementos iguales a 2. Veamos que hay exactamente n+1
listas de este tipo con n + 1 elementos que acaben en l; hacemos la demostracin por
induccin sobre n. Para n = 0 el resultado es cierto, ya que solo hay una lista con un ele-
mento que acabe en 1. Supongamos que hay t listas de longitud t de este tipo que acaben
en uno; de cada una de ellas formamos una de t + 1 elementos simplemente aadiendo
un 1 al nal; por otro lado como hay una sola lista que acaba en 2, si le agregamos un 1 al
nal tenemos una lista de longitud t + 1 que acaba en 1. Es decir en total tenemos t + 1
listas de longitud t + 1 de este tipo que acaban en 1.
ARITMTICA Y COMBINATORIA
(I. Sucesiones recurrentes)
P. Jara
52 CAP. III. EJERCICIOS DE REPASO
As pues a
n+1
= a
n
+ n + 2.
Buscamos el polinomio caracterstica de una sucesin con trmino general del tipo n+2;
ste es: (r 1)
2
. Por otro lado la recurrencia a
n+1
= a
n
tiene el polinomio caracterstico
r 1. La sucesin general determinada por el polinomio caracterstico (r 1)
3
es
1
+

2
n +
3
n
2
. Si esta sucesin verica la relacin a
n+1
= a
n
+ n + 2 tenemos:

1
+
2
(n + 1) +
3
(n + 1)
2
=
1
+
2
n +
3
n
2
+ n + 2

1
+
2
+
3
+ (
2
+ 2
3
)n +
3
n
2
= (
1
+ 2) + (
2
+ 1)n +
3
n
2

1
+
2
+
3
=
1
+ 2

2
+ 2
3
=
2
+ 1

3
=
3
_
_
_

3
=
1
2
,
2
=
3
2
.
Al imponer que verique las condiciones iniciales resulta:
a
1
= 3 =
1
+
3
2
+
1
2
Esto es,
1
= 1. As pues el trmino general de la solucin es: a
n
= 1 +
3
2
n +
1
2
n
2
.
(2) Recordar que una lista es capica si leda en el orden habitual o en el orden inverso el
resultado es el mismo. Observar que una lista capica de 2n + 1 elementos se construye
a partir de una lista capica de 2n elementos agregando uno en la posicin central, y que
toda lista de 2n +2 elementos tambin se obtiene de una de 2n elementos agregando, en
este caso, dos iguales enlas posiciones centrales. Por lotantosi llamamos a
n
al nmero de
listas capicas de n elementos, se tiene a
2n+1
= a
2n+2
= 3a
2n
. Basta pues considerar los
elementos a
2
, a
4
, a
6
, . . .. renombramos estos nmeros en la forma b
n
= a
2n
. La relacin
para los a
i
escrita para los b
j
es: b
n+1
= 3b
n
para n 1, ya que no estamos consideramos
listas vacas, con la condicin inicial b
1
= 3. Tenemos pues ua sucesin geomtrica de
trmino general b
n
= 3
n
, con n 1. Por lo tanto el valor que andamos buscando es:
a
2n1
= a
2n
= b
n
= 3
n
, n 1.
(3) Llamamos a
n
al nmero de listas con dos elementos consecutivos iguales; tiene sentido
esta denicin para n 2. Se verica a
2
= 3. Si llamamos b
n
al nmero de listas en las
que no hay elementos consecutivos iguales, entonces b
2
= 6 y se verica a
2
+b
2
= 3+6 =
9 = 3
2
, que es el nmero total de listas de dos elementos. Tal vez sea ms sencillo calcular
b
n
que a
n
. Se tiene b
3
= 2b2, ya que cada lista de dos elementos distintos proporciona
dos listas de tres elementos en las que no hay dos elementos consecutivos iguales. Este
argumento es vlido para cualquier longitud mayor que dos, y por tanto se tiene b
n+1
=
2b
n
, si n 2. Como b
2
= 6 = 2 3, resulta b
n
= 3 2
n1
.
Como consecuencia a
n
= 3
n
b
n
= 3
n
3 2
n1
= 3(3
n1
2
n1
).
28 de enero de 2013 Curso 20122013. NOTAS DE TRABAJO, 30
SEC. 9. EJERCICIOS DE REPASO 53
(4) Llamamos a
n
al nmero de listas de longitud n en las que no hay dos unos o dos doses
consecutivos. Es claro que a
1
= 3, a
2
= 7 y a
3
= 17. Vamos a ver una recurrencia entre
estos nmeros. Dada una lista de n elementos vericando las condiciones, si agregamos
un 0 al nal tendremos una lista de n+1 elementos vericando las condiciones. Adems,
a cada lista de n elementos podemos agregar al nal 1 o 2 segn si naliza en 2 o en 1, de
esta forma tenemos una lista de n+1 elementos que verica las condiciones; nos quedan
las listas de n elementos que acaben un 0; en este caso agregamos al nal 1. Observar que
de esta forma no tenemos todas las listas de n+1 elementos que vericanlas condiciones;
nos faltan justamente las que nalizan en ..,02; pero de stas hay exactamente a
n1
, que
son los posibles inicios de n 1 elementos. Si contamos ahora todas las listas obtenidas,
resulta la igualdad
a
n+1
= 2a
n
+ a
n1
.
sta es la ecuacin de recurrencia que verica la sucesin {a
n
}
n
. Su polinomio caracte-
rstico es: r
2
2r1, y sus races son1+

2 y 1

2. La sucesingeneral tiene la expresin

1
(1 +

2)
n
+
2
(1

2)
n
. Al imponer las condiciones iniciales tenemos:
a
1
= 3 =
1
(1 +

2) +
2
(1

2)
a
2
= 7 =
1
(1 +

2)
2
+
2
(1

2)
2
_
La solucin es:
1
=
1+

2
2
y
2
=
1

2
2
, de forma que el trmino general de la sucesin es:
a
n
=
1 +

2
2
(1 +

2)
n
+
1

2
2
(1

2)
n
=
1
2
_
(1 +

2)
n
+ (1

2)
n
_
.

Problema. 9.7.
Dado un entero positivo n un desorden de n es una ordenacin del conjunto {1, 2, 3, . . . , n}
en la que ningn elemento ocupa su lugar original.
Calcular el nmero de desrdenes posibles de un conjunto de n elementos.
SOLUCIN. Llamamos a
n
al nmero de desrdenes del conjunto de {1, 2, 3, . . . , n} Veamos
como podemos construir un estado de desorden de estos n elementos a partir un conjunto
menor de elementos.
(1) Dado un estado de desorden de los n 1 elementos {1, 2, 3, . . . , n 1}, conseguimos otro
estado de desorden intercambiando n con cada uno de los otros elementos. En total tenemos
(n 1)a
n1
estados de desorden;
(2) Otra forma de obtener un estado de desorden de n elementos es considerar una permuta-
cin de los n1 primeros elementos en la que un solo elemento est en su posicin original,
sea i. Ahora en el lugar de i ponemos n y ponemos i al nal (derecha); se obtiene as un estado
de desorden. En total tenemos (n 1)a
n2
estados de desorden.
El total de estados de desorden es a
n
= (n 1)(a
n1
+ a
n2
). Se verica adems: a
1
= 0 y
a
2
= 1; como consecuencia a
3
= 2(1 + 0) = 2; estos son los dos ciclos 231 y 312.
ARITMTICA Y COMBINATORIA
(I. Sucesiones recurrentes)
P. Jara
54 CAP. III. EJERCICIOS DE REPASO
Vamos a calcular el trmino general de esta sucesin. Para ello introducimos dos sucesiones
auxiliares:
b
n
= n 1, si n 1,
c
n
= c
n1
+ c
n2
, si n 3, c
1
= 1, c
2
= 1
Es claro que la sucesin {c
n
}
n
es la sucesin de Fibonacci, cuyo trmino general es: c
n
=

5
5
_
1+

5
2
_
n

5
5
_
1

5
2
_
n
. Tenemos entonces que el trmino general de {a
n
}
n
es:
a
n
=
(n 1)

5
5
__
1 +

5
2
_
n

_
1

5
2
_
n
_

Problema. 9.8.
De cuntas formas se puede cubrir un tablero rectangular de dimensin 2 n con piezas
de dimensiones 1 2 y 2 2.
SOLUCIN. Llamamos a
n
al nmero de formas en que podemos cubrir un tal tablero 2 n,
y observamos cmo es la ltima o ltimas piezas que ponemos.
(1) La ltima pieza es una 1 2, en esta caso slo se puede poner de una forma, supuesto
que hemos completado en tablero de dimensin 2 (n 1), luego el nmero de formas que
aporta es a
n1
.
(2) La ltima pieza es una pieza 22, podemos hacerlo de dos formas: bien una pieza 22, o
bien dos piezas 1 2 horizontales, de ambas formas podramos acabar el tablero, el nmero
total de formas de completarlo que aporta es 2a
n2
.
Como consecuencia el nmero a
n
est determinado por las relaciones:
a
n
= a
n1
+ 2a
n2
.
Los valores iniciales son: a
0
= 1, a
1
= 1, a
2
= 3, esto es, dos piezas 1 2 verticales, dos
piezas 1 2 horizontales y una pieza 2 2. Veamos el caso de a
3
, el valor por la frmula es:
a
3
= 3 + 2 1 = 5, las posibles distribuciones son:
Para calcular el trmino general primero determinamos la ecuacin caracterstica: r
2
= r +2;
las races son:
1
= 1 y
2
= 2.
El trmino general de la sucesin es: A
n
=
1

n
1
+
2

n
2
=
1
(1)
n
+
2
2
n
. Al imponer las
condiciones iniciales se tiene
a
0
= 1 =
1
+
2
a
1
= 1 =
1
+ 2
2
_
La solucin es:
1
=
2
3
y
2
=
1
3
. El trmino general de la sucesin buscada es: a
n
=
1
3
(1)
n
+
2
3
2
n
.
28 de enero de 2013 Curso 20122013. NOTAS DE TRABAJO, 30
SEC. 9. EJERCICIOS DE REPASO 55
Nota: Semejante al problema de la pgina 50.
Problema. 9.9.
Bernardo sube la escalera de su casa saltando en cada paso uno o dos escalones. Si la es-
calera de la casa de Bernardo tiene treinta escalones, de cuntas formas diferentes puede
subir Bernardo la escalera.
SOLUCIN. Llamamos a
n
al nmero de formas distintas en que puede Bernardo subir una
escalera de n escalones.
(1) Si el ltimo paso de Bernardo es de un escaln, entonces Bernardo sube la escalera de
a
n1
formas distintas,
(2) Si el ltimo paso de Bernardo es de dos escalones, entonces Bernardo cube la escalera de
a
n2
formas distintas.
El nmero total de formas de subir una escalera de n escalones es: a
n
= a
n1
+ a
n2
. Los
casos iniciales son: a
0
= 0, a
1
= 1 y a
2
= 2; esto es, estamos en presencia de la sucesin de
Fibonacci.
Problema. 9.10.
Determinar el nmero de cadenas binarias (con ceros y unos) de longitud n que contienen
dos ceros consecutivos.
SOLUCIN. Llamamos a
n
al nmero de cadenas vlidas de longitud n. Dada una cadena
vlida de longitud n, estudiamos las siguientes posibilidades:
(1) La cadena acaba en 1; en este caso la cadena proviene de una cadena vlida de longitud
n 1; tenemos en total a
n1
,
(2) La cadena acaba en 0; en este caso la cadena puede haber sido obtenida a partir de una
cadena vlida de longitud n2 agregando bien 00 bien 10, tenemos en total 2a
n2
de stas;
o biena partir de una cadena no vlida de longitud n2 a la que hemos agregado 00, tenemos
2
n2
a
n2
de stas.
El nmero total de cadenas vlidas de longitud n es: a
n
= a
n1
+ a
n2
+ 2
n2
. Los valores
iniciales son: a
1
= 0, a
2
= 1
La ecuacin caracterstica es: r
2
= r + 1, cuyas races son:
1
=
1+

5
2
,
2
=
1

5
2
, que fue
estudiada con la sucesin de Fibonacci
Falta ahora encontrar una solucin particular de la ecuacin de recurrencia no homognea
a
n
= a
n1
+ a
n2
+ 2
n2
. Supongamos que sta sea 2
n2
, entonces se verica:
2
n2
= 2
n3
+ 2
n4
+ 2
n2
,
= 4.
Una solucin particular es 2
n
. La solucin general es
a
n
= 2
n
+
1
_
1 +

5
2
_
n1
+
2
_
1

5
2
_
n1
.
ARITMTICA Y COMBINATORIA
(I. Sucesiones recurrentes)
P. Jara
56 CAP. III. EJERCICIOS DE REPASO
Al introducir las condiciones iniciales tenemos:
0 = 2 +
1
+
2
,
1
=
2

5+5
5
,
1 = 2
2
+
1
_
1+

5
2
_
+
2
_
1

5
2
_
,
2
=
2

55
5
La solucin es:
a
n
= 2
n

5 + 5
5
_
1 +

5
2
_
+
2

5 5
5
_
1

5
2
_
.

Problema. 9.11.
Calcular el trmino general de la sucesin denida por
a
n
= a
n1
+ n
2
+ 3n 5, si n 1, y a
1
= 2
SOLUCIN. Tenemos que al calcular las diferencias, stas son:
a

2 1 6 19 42 77
a

1 5 13 23 35

2
a

6 8 10 12

3
a

2 2 2
Entonces la sucesin es una progresin aritmtica de orden 3. La frmula para calcular el
trmino general es:
a
n
=
n

i=0
_
n
i
_

i
a
1
.
Ver la frmula (I.2) en la pgina 9.
a
n
=
_
n
0
_
a
1
+
_
n
1
_
a
1
+
_
n
2
_

2
a
1
+
_
n
3
_

3
a
1
=
_
n
0
_
2 +
_
n
1
_
(1) +
_
n
2
_
6 +
_
n
3
_
2
=
n
3
+6n
2
10n+6
3
.

Problema. 9.12.
De cuntas formas se puede cubrir un tablero de tamao 32n con piezas de tamao 12?
SOLUCIN. Llamamos a
n
al nmero de formas de cubrir el tablero 3 2n. En funcin de
tableros completos este tablero se puede terminar de llenar:
(1) con tres piezas horizontales,
28 de enero de 2013 Curso 20122013. NOTAS DE TRABAJO, 30
SEC. 9. EJERCICIOS DE REPASO 57
(2) con una pieza horizontal y dos verticales
(2.1) con las dos piezas verticales en la parte superior
(2.2) con las dos piezas verticales en la parte inferior
En resumen un tablero de tamao 3 2(n 1)4 se puede rellenar de tres formas diferentes.
Si el tablero tiene tamao 3 2(n 2), entonces se puede rellenar de dos formas, segn el
siguiente esquema
El nmero total de formas en que podemos completar el tablero es: a
n
= 3a
n1
+ 2a
n2
.

Problema. 9.13.
Se considera la sucesin recurrente denida por:
a
0
= 2 = a
1
y a
n
= 2a
n1
2a
n2
.
Hallar el trmino general.
SOLUCIN. Es claro que la ecuacin caracterstica es: r
2
= 2r 2, por lo que las races son:

1
= 1 + i y
2
= 1 i.
Entonces el trmino general es:
a
n
=
1
(1 + i)
n
+
2
(1 i)
n
,
sujeto a las condiciones iniciales a
0
= 2 = a
1
. Tenemos que resolver el siguiente sistema de
ecuaciones lineales:
_
2 =
1
+
2
,
1
= 1
2 =
1
(1 + i) +
2
(1 i),
2
= 1
El trmino general es:
a
n
= (1 + i)
n
+ (1 i)
n
= cos
_
n
2
_
+ i sen
_
n
2
_
+ cos
_
n
2
_
i sen
_
n
2
_
= 2 cos
_
n
2
_

Problema. 9.14.
Calcular la suma de las potencias cuartas y quintas de los nmeros enteros positivos
1, 2, . . . , n.
ARITMTICA Y COMBINATORIA
(I. Sucesiones recurrentes)
P. Jara
58 CAP. III. EJERCICIOS DE REPASO
SOLUCIN. Potencias cuartas. De la frmula de la pgina 11,
_
k
k 1
_
C
k1
n
= (n + 1)
k
(n + 1)
k2

i=1
_
k
i
_
C
i
n
se obtiene para k = 5 las siguientes expresiones:
5C
4
n
= (n + 1)
5
(n + 1)

3
i=1
_
5
i
_
C
i
n
= n
5
+ 5n
4
+ 10n
3
+ 10n
2
+ 5n + 1 n 1 (5C
1
n
+ 10C
2
n
+ 10C
3
n
)
= n
5
+ 5n
4
+ 10n
3
+ 10n
2
+ 4n (5
n
2
+n
2
+ 10
2n
3
+3n
2
+n
6
+ 10
(n(n+1))
2
4
)
= n
5
+
5n
4
2
+
5n
3
3

n
6
=
6n
5
+15n
4
+10n
3
n
6
.
Potencias quintas. Utilizando la misma frmula, para k = 6 se obtiene:
6C
5
n
= (n + 1)
6
(n + 1)

4
i=1
_
6
i
_
C
i
n
= n
6
+ 6n
5
+ 15n
4
+ 20n
3
+ 15n
2
+ 6n + 1 n 1
(6C
1
n
+ 15C
2
n
+ 20C
3
n + 15C
4
n
)
=
1
2
n
2
(n + 1)
2
(2n
2
+ 2n 1)

Problema. 9.15.
Se considera la sucesin de Fibonacci: a
0
= 0, a
1
= 1 y a
n
= a
n1
+ a
n2
para n 2.
(1) Probar que a
n+k
= a
n
a
k+1
+ a
n1
a
k
.
(2) Probar que a
nk
mltiplo de a
k
.
(3) Se puede extender la denicin de la sucesin de Fibonacci para incluir a ndices nega-
tivos deniendo a
k
= a
k+2
ak + 1 cuando k es un entero negativo. Probar que se verica:
a
n
= (1)
n+1
a
n
para cada entero positivo n.
(4) m. c. d.{a
n
, a
k
} = a
m.c.d.{n,k}
.
SOLUCIN. (1). Hacemos induccin sobre k.
Para k = 0 tenemos: a
n
= a
n
a
1
+ a
n1
a
0
= a
n
.
Para k = 1 tenemos: a
n+1
= a
n
a
2
+ a
n1
a
1
= a
n
+ a
n1
= a
n+1
.
Suponemos que el resultado es cierto para todo m k, entonces se verica:
a
n+k+1
= a
n+k
+ a
n+k1
= (a
n
a
k+1
+ a
n1
a
k
) + (a
n
a
k
+ a
n1
a
k1
)
= a
n
(a
k+1
+ a
k
) + a
n1
(a
k
+ a
k1
)
= a
n
a
k+2
+ a
n1
a
k+1
.
(2). Hacemos induccin sobre k.
28 de enero de 2013 Curso 20122013. NOTAS DE TRABAJO, 30
SEC. 9. EJERCICIOS DE REPASO 59
Para k = 0 tenemos a
0
= 0 y el resultado es cierto. Suponemos que el resultado es cierto para
todo m k, entonces se verica:
a
n(k+1)
= a
nk+n
= a
nk
a
k+1
+ a
nk1
a
k
Por la hiptesis a
k
divide a a
nk
y a a
k
, luego divide a a
n(k+1)
.
(3). Hacemos induccin sobre n. Comprobamos que si n = 1 el resultado es correcto:
a
1
= a
1+2
a
1+1
= a
1
a
0
= 1 = (1)
2
a
1
Suponemos que el resultado es cierto para todo m n, entonces se verica:
a
(n+1)
= a
n+1
a
n
= a
(n1)
a
n
= (1)
n
a
n1
(1)
n+1
a
n
= (1)
n+1
(a
n1
+ a
n
)
= (1)
(n+1)+1
a
n+1
(4). Sea d = m. c. d.{n, k} y sea X = m. c. d.{a
n
, a
k
}. Como n = dn

y k = dk

para ciertos
enteros positivos o nulos n

y k

, resulta
a
n
= a
dn
es un mltiplo de a
d
.
El mismo argumento vale para a
k
, luego a
d
es un divisor de a
n
y de a
k
, por tanto es un divisor
de X.
Por otro lado, la identidad de Bezout expresa d en funcin de n y k, en la forma d = n + k,
entonces resulta
a
d
= a
n+k
= a
n
a
k+1
+ a
n1
a
k
.
Como a
n
divide a a
n
, entonces X divide a a
n
, y como a
k
divide a a
k
, entonces X divide a
a
k
. En consecuencia X divide a a
d
, y tenemos el resultado.
Problema. 9.16.
Se considera la sucesin de Fibonacci {a
n
}
n
, y se dene una nueva sucesin F
n
=

n
i=0
a
i
.
Encontrar una expresin recurrente para esta nueva sucesin y determinar su trmino ge-
neral.
SOLUCIN. Se tiene F
n
= F
n1
+ a
n
, entonces
F
n
F
n1
= a
n
= a
n1
+ a
n2
= (F
n1
F
n2
) + (F
n2
F
n3
,
de donde resulta que F
n
= 2F
n1
F
n3
. Los valores iniciales son: F
0
= 0, F
1
= 1 y F
2
= 2, y
la ecuacin caracterstica es:
r
3
= 2r
2
1
Sus races son:
1
= 1,
2
=
1+

5
2
y
3
=
1

5
2
.
ARITMTICA Y COMBINATORIA
(I. Sucesiones recurrentes)
P. Jara
60 CAP. III. EJERCICIOS DE REPASO
El trmino general de F
n
es:
F
n
=
1
1
n
+
2
_
1 +

5
2
_
n
+
3
_
1

5
2
_
n
que el imponer las condiciones iniciales nos da el sistema de ecuaciones lineales:
0 =
1
+
2
+
3
1 =
1
+
2
1+

5
2
+
3
1

5
2
2 =
1
+
2
_
1+

5
2
_
2
+
3
_
1

5
2
_
2
_

_
cuya solucin es:
1
= 1,
2
=
1+

5
5

5
y
3
=
1

5
5+

5
. De forma que el trmino general es:
F
n
= 1 +
1 +

5
5

5
_
1 +

5
2
_
n
+
1

5
5 +

5
_
1

5
2
_
n
que toma los valores: 0, 1, 2, 4, 7, 12, 20, 33, 54, 88, 143, 232, 376, 609, 986, 1596, 2583, . . .
Problema. 9.17.
Se considera la sucesin de Fibonacci: a
0
= 0, a
1
= 1 y a
n
= a
n1
+ a
n2
para n 2. Si
denimos P
n
= a
2n
y denimos I
n
= a
2n+1
, encontrar una relacin de recurrencia lineal
para P
n
e I
n
.
SOLUCIN. Tenemos:
P
n
= a
2n
= a
2n1
+ a
2n2
= a
2n1
+ P
n1
.
De aqu se obtiene s
2n1
= P
n
P
n1
, y por tanto podemos proceder como sigue:
P
n
= a
2n1
+ P
n1
= a
2n2
+ a
2n3
+ P
n1
= P
n1
+ a
2n3
+ P
n1
= P
n1
+ a
2(n1)1
+ P
n1
= P
n1
+ (P
n1
P
n2
) + P
n1
= 3P
n1
P
n2
Las condiciones iniciales son: P
0
= 0, P
1
= 1.
Para el clculo del trmino general de P
n
primero calculamos la ecuacin caracterstica: r
2
=
3r 1, cuyas races son:
:
1 =
3+

5
2
y
2
=
3

5
2
, entonces el trmino general es: P
n
=

1
_
3+

5
2
_
n
+
2
_
3

5
2
_
n
; al imponer las condiciones iniciales se tiene el sistema de ecuacio-
nes lineales:
0 =
1
+
2
1 =
1
_
3+

5
2
_
+
2
_
3

5
2
_
_
28 de enero de 2013 Curso 20122013. NOTAS DE TRABAJO, 30
SEC. 9. EJERCICIOS DE REPASO 61
que tiene solucin
1
=
1

5
y
2
=
1

5
. El trmino general es:
P
n
=
1

5
_
3 +

5
2
_
n

5
_
3

5
2
_
n
.
Para I
n
procedemos en la misma forma y obtenemos I
n
= 3I
n1
I
n2
con las condiciones
iniciales I
0
= 1, I
1
= 2.
Para el clculo del trmino general de I
n
procedemos en la misma forma. En este caso tene-
mos que resolver el sistema de ecuaciones lineales:
1 =
1
+
2
2 =
1
_
3+

5
2
_
+
2
_
3

5
2
_
_
que tiene solucin
1
=
5+

5
10
y
2
=
5

5
10
. El trmino general es:
I
n
=
5 +

5
10
_
3 +

5
2
_
n
+
5

5
10
_
3

5
2
_
n
.

Problema. 9.18. (Los nmeros de Lucas)


Consideramos la sucesin recurrente denida por: l
0
= 2. l
1
= 1 y la frmula l
n
= l
n1
+l
n2
para n 2. Los nmeros l
n
se llaman los nmeros de Lucas. Determinar el trmino general
de la sucesin de los nmeros de Lucas, y probar que si {f
n
}
n
es la sucesin de Fibonacci,
entonces se verica:
l
n
= f
n1
+ f
n+1
;
l
n
= f
n+2
f
n2
;
f
h
l
n
= f
n+h
+ (1)
h+1
f
nh
.
Probar la siguiente relacin entre nmeros de Fibonacci y nmeros de Lucas f
2n
= f
n
l
n
.
SOLUCIN. La ecuacin caracterstica es r
2
= r +1, y sus races son:
1
=
1+

5
2
y
2
=
1

5
2
.
El trmino general es: l
n
=
1
_
1+

5
2
_
n
+
2
_
1

5
2
_
n
. Al imponer las condiciones iniciales se
obtiene el siguiente sistema de ecuaciones lineales:
2 =
1
+
2
1 =
1
_
1+

5
2
_
+
2
_
1

5
2
_
_
La solucin es:
1
= 1 y
2
= 1. De forma que el trmino general es: l
n
=
_
1+

5
2
_
n
+
_
1

5
2
_
n
ARITMTICA Y COMBINATORIA
(I. Sucesiones recurrentes)
P. Jara
62 CAP. III. EJERCICIOS DE REPASO
Podemos hacer uso de la expresin del trmino general de la sucesin de Fibonacci: f
0
= 0,
f
1
= 1 y f
n
=

5
2
_
1+

5
2
_
n

5
2
_
1

5
2
_
n
, entonces se tiene:
f
n1
+ f
n+1
=

5
5
_
1+

5
2
_
n1

5
5
_
1

5
2
_
n1
+

5
5
_
1+

5
2
_
n+1

5
5
_
1

5
2
_
n+1
=

5
5
_
1+

5
2
_
n1
_
1 +
_
1+

5
2
_
2
_

5
5
_
1

5
2
_
n1
_
1 +
_
1

5
2
_
2
_
=
_
1+

5
2
_
n1

5
5
_
5+

5
2
_

_
1

5
2
_
n1

5
5
_
5

5
2
_
=
_
1+

5
2
_
n
+
_
1

5
2
_
n
= l
n
La misma relacin para f
n+2
se puede probar ahora como sigue:
l
n
= f
n+1
+ f
n1
= f
n+1
+ f
n
f
n
+ f
n1
= f
n+2
f
n2
,
ya que de la relacin f
n
= f
n1
+ f
n2
se obtiene f
n2
= f
n
f
n1
.
Podemos extender estas relaciones a nmeros de Fibonacci y nmeros de Lucas con ndices
no necesariamente positivos. Al estudiar diferentes casos, observamos la relacin:
f
h
l
n
= f
n+h
+ (1)
h+1
f
nh
.
Cuya demostracin es sencilla si hacemos induccin sobre h:
f
h+1
l
n
= f
h
l
n
+ f
h1
l
n
= f
n+h
+ (1)
h+1
f
nh
+ f
n+h1
+ (1)
h
f
nh+1
= (f
n+h
+ f
n+h1
) + (1)
h+2
(f
nh
+ f
nh+1
)
= f
n+h+1
+ (1)
h+2
f
nh1
La relacin nal del enunciado es evidente, basta tomar h = n.
Problema. 9.19.
Consideramos {f
n
}
n
la sucesin de Fibonacci, probar que se verican las siguientes identi-
dades:
(1) Identidad de Cassini. f
2
n
f
n+1
f
n1
= (1)
n1
f
2
1
.
(2) Identidad de Catalan. f
2
n
f
n+r
f
nr
= (1)
nr
f
2
r
, r 1.
(3) Identidad de dOcagne. f
m
f
n+1
f
n
f
m+1
= (1)
n
f
mn
.
(4) Identidad de GelinCesro. f
4
n
f
n2
f
n1
f
n+1
f
n+2
= 1.
28 de enero de 2013 Curso 20122013. NOTAS DE TRABAJO, 30
SEC. 9. EJERCICIOS DE REPASO 63
SOLUCIN.

Problema. 9.20.
Considerar las sumas parciales de la sucesin de los nmeros de Lucas.
SOLUCIN. .
Problema. 9.21.
Determinar un polinomio p(x) de grado cuatro, sin trmino independiente, que verique
p(x) p(x 1) = x
3
. Demuestra que la suma de los cubos de los n primeros nmeros
enteros positivos es p(n).
XXVIII Olimpada Matemtica Dist. Univ. Valencia. Problema nm. 5
SOLUCIN. Tenemos p(0) = 0, y de p(x) p(x 1) = x
3
se deduce que p(0) p(1) = 0,
luego p(1) = 0. Entonces se tiene la factorizacin p(x) = x(x + 1)(ax
2
+ bx + c).
Vamos a dar nuevos valores:
n = 1 p(1) p(0) = 1 p(1) = 1 = 1 2(a + b + c)
n = 2 p(2) p(1) = 2
3
p(2) = 2
3
1 = 7 = (2 3(4a + 2b + c)
n = 3 p(3) p(2) = 3
3
p(3) = 3
3
7 = 20 = 3 4(9a + 3b + c)
Tenemos que resolver el sistema de ecuaciones lineales:
a + b + c =
1
2
4a + 2b + c =
7
6
9a + 3b + c =
20
12
=
5
3
_
_
_
La solucin es: a =
1
4
= b, c = 0. El polinomio es: p(x) = x(x + 1)(
1
4
x
2
+
1
4
x) =
_
x(x+1)
2
_
2
.
Por induccin sobre n se prueba que la suma de los n primeros enteros positivos es p(n).

Problema. 9.22.
Se considera la sucesin de los nmeros triangulares, esto es, los n-
meros que representan los puntos que formas los tringulos siguientes:
i
i
i i
i
i i
i i i
Esta sucesin tiene los siguientes elementos: 1, 3, 6, 10, 15, . . .
Se trata de encontrar la frmula del trmino general de esta sucesin.
ARITMTICA Y COMBINATORIA
(I. Sucesiones recurrentes)
P. Jara
64 CAP. III. EJERCICIOS DE REPASO
SOLUCIN. Al hacer las diferencias sucesivas de los elementos de esta sucesin tenemos:
a
n
1 3 6 10 15
a
n
2 3 4 5

2
a
n
1 1 1
Para construir el trmino general procedemos al igual que en el problema (4.2.).
a
n1
= a
0
+ S
2
n

2
a
n1
= 2 + n 1 1
= n + 1
a
n1
= a
0
+ S
1
n
a
n1
= 1 +
(2+(n+1))n
2
(n + 1)
=
n(n+1)
2

Problema. 9.23.
Se considera la sucesin de los nmeros tetradricos, esto es, los n-
meros que representan los puntos que formas los tetraedros siguientes:
i
i
i i
i

A
A

@
Esta sucesin tiene los siguientes elementos: 1, 4, 10, 20, 35, . . .
Se trata de encontrar la frmula del trmino general de esta sucesin.
SOLUCIN. Al hacer las diferencias sucesivas de los elementos de esta sucesin tenemos:
a
n
1 4 10 20 35
a
n
3 6 10 15

2
a
n
3 4 5

3
a
n
1 1
Para construir el trmino general procedemos al igual que en el problema (4.2.).

2
a
n1
=
2
a
0
+ S
2
n

3
a
n1
= 3 + n 1 1
= n + 2
a
n1
= a
0
+ S
2
n

2
a
n1
= 3 +
(3+(n+2))n
2
(n + 2)
=
n
2
+3n+2
2
28 de enero de 2013 Curso 20122013. NOTAS DE TRABAJO, 30
SEC. 9. EJERCICIOS DE REPASO 65
a
n1
= a
0
+ S
1
n
a
n1
= 1 +
1
2
C
2
n
+
3
2
C
1
n
+ C
0
n

n
2
+3n+2
2
=
1
2
2n
3
+3n
2
+n
6
+
3
2
n(n+1)
2
+ n
n
2
+3n+2
2
=
2n
3
+6n
2
+4n
12
=
n(n+1)(n+2)
6
Nota. Los nmeros tetradricos son la cuarta la oblicua del tringulo de Tartaglia:
1
1 1
1 2 1
1 3 3 1
1 4 6 4 1
1 5 10 10 5 1
1 6 15 20 15 6 1
1 7 21 35 35 21 7 1
La tercera son los nmeros triangulares, por lo tanto las frmulas para el trmino general son
_
n
3
_
y
_
n
2
_
respectivamente.
Problema. 9.24.
Determinar el trmino general de la sucesin {a
n
}
n
vericando la ecuacin de recurrencia
a
n
= 2a
n1
a
n2
+ k,
vericando las condiciones iniciales a
0
= 0 y a
1
= h.
SOLUCIN. Calculamos una solucin particular. Sea
1
+
2
n +
3
n
2
, entonces se tiene:

1
+
2
n +
3
n
2
2(
1
+
2
(n 1) +
3
(n 1)
2
) + (
1
+
2
(n 2) +
3
(n 2)
2
) k
= 2
3
k,
entonces si tomamos
3
=
k
2
,
1
= 0 =
2
, tenemos la solucin particular
k
2
n
2
.
La parte homognea tiene por ecuacin caracterstica r
2
= 2r 1; la solucin es
1
= 1 =
2
,
entonces la solucin general de la parte homognea es:
4
+
5
n, y la solucin general de la
ecuacin de recurrencia no homognea es:
k
2
n
2
+
4
+
5
n.
Al hacer intervenir ahora a las condiciones iniciales, se obtiene el sistema de ecuaciones li-
neales

4
= 0
k
2
+
4
+
5
= h
_
cuya solucin es:
4
= 0 y
5
= h
k
2
. La solucin general es:
a
n
=
k
2
n
2
+ (h
k
2
)n =
k
2
(n
2
n) + hn.
ARITMTICA Y COMBINATORIA
(I. Sucesiones recurrentes)
P. Jara
66 CAP. III. EJERCICIOS DE REPASO

Problema. 9.25.
Sean a y b enteros positivos. Hallar el trmino general de la ecuacin recurrente
a
n
= a
n1
+ a +
_
b
2
+ 4aa
n1
, si n 1, a
0
= 0.
[Generalizacin de un problema propuesto en IMO 1981]
SOLUCIN. Vamos a tratar de eliminar los radicales.
Tenemos
a
n
a
n1
= a +
_
b
2
+ 4aa
n1
, (III.1)
y se verica
b
2
+ 4aa
n
= (b
2
+ 4aa
n1
) + 4a
2
+ 4a
_
b
2
+ 4aa
n1
= (2a +
_
b
2
+ 4aa
n1
)
2
.
Luego
_
b
2
+ 4aa
n
= 2a +
_
b
2
+ 4aa
n1
, lo que da lugar a la siguiente relacin:
a
n1
= a
n2
+ a +
_
b
2
+ 4aa
n2
= a
n2
+ a 2a +
_
b
2
+ 4aa
n1
.
De aqu se obtiene:
a
n1
a
n2
= a +
_
b
2
+ 4aa
n1
. (III.2)
Al realizar (III.1)(III.2) se tiene:
a
n
2a
n1
+ a
n2
= 2a.
Adems:
a
1
= a
0
+ a +
_
b
2
+ 4aa
0
= a +

b
2
= a + b,
luego tenemos una sucesin vericando:
a
n
= 2a
n1
a
n2
+ 2a, si n 2, a
0
= 0, a
1
= a + b,
entonces aplicando el problema (9.24.) el trmino general es:
a
n
=
2a
a
n
2
+ (a + b
2a
a
)n = an
2
+ bn = (an + b)n, n 2.

Problema. 9.26.
Demostrar que en la sucesin denida por los siguientes datos:
a
0
= 0 y
a
n+1
= (a
n
+ 1)k + (k + 1)a
n
+ 2
_
k(k + 1)a
n
(a
n
+ 1), n 0, k un entero positivo.
todos sus trminos a
n
, n 1, son enteros positivos.
[No elegido en la IMO 1983]
28 de enero de 2013 Curso 20122013. NOTAS DE TRABAJO, 30
SEC. 9. EJERCICIOS DE REPASO 67
SOLUCIN. Como la sucesin es estrictamente creciente, y como a
0
= 0, entonces todos los
trminos a
n
, n 1 son positivos. Vamos a ver que son enteros.
Al tratar de eliminar los radicales se tiene:
a
n+1
(a
n
+ 1)k (k + 1)a
n
= 2
_
k(k + 1)a
n
(a
n
+ 1)
Elevando al cuadrado y agrupando en el primer miembro se obtiene:
a
2
n
2a
n
(a
n+1
+ k + 2a
n+1
k) + (a
n+1
k)
2
= 0.
Resolvemos esta ecuacin en a
n
y se obtiene:
a
n
=
2(a
n+1
+k+2a
n+1
k)

4(a
n+1
+k+2a
n+1
k)
2
4(a
n+1
k)
2
2
= a
n+1
(2k + 1) + k 2
_
a
n+1
(a
n+1
+ 1)(k + 1)k
Ahora, como la sucesin es estrictamente creciente, en el radical debemos tomar el signo
menos":
a
n
= a
n+1
(2k + 1) + k 2
_
a
n+1
(a
n+1
+ 1)(k + 1)k
entonces, utilizando la denicin para a
n+2
se tiene:
a
n
= a
n+1
(2k + 1) + k 2
_
a
n+1
(a
n+1
+ 1)(k + 1)k
= a
n+1
(2k + 1) + k (a
n+2
(a
n+1
+ 1)k (k + 1)a
n+1
)
= a
n+1
(3k + 2) + k + (a
n+1
+ 1)k a
n+2
= a
n+1
(4k + 2) + 2k a
n+2
.
a
n+2
= a
n+1
(4k + 2) a
n
+ 2k,
Tenemos entonces que todos los valores son enteros, ya que
a
1
= (a
0
+ 1)k + (k + 1)a
0
+ 2
_
k(k + 1)a
0
(a
0
+ 1) = k
es tambin entero.
Problema. 9.27.
Sean m, n y r enteros positivos vericando 1 + m + n

3 = (2 +

3)
2r1
. Demostrar que
entonces m es un cuadrado perfecto.
[II Olimpada Iberoamericana Uruguay 1987 (Felipe Fritz Braga, mencin especial del jura-
do)]
SOLUCIN. Al conjugar por

3 tenemos las relaciones:


1 + m + n

3 = (2 +

3)
2r1
1 + mn

3 = (2

3)
2r1
,
que al sumar dan: 2 + 2m = (2 +

3)
2r1
+ (2

3)
2r1
.
ARITMTICA Y COMBINATORIA
(I. Sucesiones recurrentes)
P. Jara
68 CAP. III. EJERCICIOS DE REPASO
Si consideramos
1
= 2 +

3 y
2
= 2

3, stas son races del polinomio (x (2 +

3))(x
(2

3)) = x
2
4x+1. Podemos considerar la sucesinrecurrente conecuacincaracterstica
x
2
4x + 1, cuyo trmino general es:
a
r
=
1
(2 +

3)
r
+
2
(2

3)
r
.
Si tomamos las condiciones iniciales a
0
= 1 y a
1
= 1, entonces tenemos que resolver el
sistema de ecuaciones lineales
1 =
1
+
2
1 =
1
(2 +

3) +
2
(2

3)
_
La solucin es:
1
=

31
2
y
2
=

31
2
. El trmino general es:
a
r
=

3 1
2
(2 +

3)
r

3 + 1
2
(2

3)
r
Se verica ahora:
(a
r
)
2
=
_

31
2
_
2
(2 +

3)
2r
+
_

3+1
2
_
2
(2

3)
2r
2
_

31
2
__

3+1
2
_
=
2

3
2
(2 +

3)
2r
+
2+

3
2
(2

3)
2r
1
=
1
2
(2 +

3)
2r1
+
1
2
(2

3)
2r1
1
= m

Problema. 9.28.
Sea m un entero positivo y denamos la sucesin siguiente:
a
1
= 2(m + 1),
a
n+1
=
1
4m
(ma
n
1 +

1 + 2ma
n
), si n 1.
Obtener una expresin explcita del trmino a
n
en funcin de n y m.
[Gazeta Matematica. 1988. Rumania]
SOLUCIN. Observamos el valor de a
2
.
a
2
=
1
4m
(ma
n
1 +

1 + 2ma
n
)
=
1
4m
(m2(m + 1) 1 +
_
1 + 2m2(m + 1))
=
1
4m
(2m
2
2m1 + 2m + 1)
=
m+2
2
.
28 de enero de 2013 Curso 20122013. NOTAS DE TRABAJO, 30
SEC. 9. EJERCICIOS DE REPASO 69
Tenemos que a
1
> a
2
, ya que si a
1
< a
2
, entonces
2(m + 1) <
m+2
2
;
4m + 4 < m + 2;
3m + 2 < 0;
m <
2
3
,
lo que es una contradiccin. Adems a
1
, a
2
> 0.
Veamos ahora la diferencia a
n+1
a
n
.
a
n+1
a
n
=
1
4m
(ma
n
1 +

1 + 2ma
n
) a
n
=
1
4m
(3ma
n
1 +

1 + 2ma
n
).
Si 3ma
n
1 +

1 + 2ma
n
> 0, entonces

1 + 2ma
n
> 3ma
n
+ 1. Vamos a suponer que
a
n
> 0, entonces podemos tomar cuadrados manteniendo la desigualdad:
1 + 2ma
n
> 9m
2
a
2
n
+ 1 + 6ma
n
0 > 9m
2
a
2
n
+ 4ma
n
,
lo cual es imposible. Tenemos pues que si a
n
> 0, entonces a
n
> a
n+1
.
Vamos a ver que tambin a
n+1
> 0 en este caso. Si a
n+1
0, entonces
ma
n
1 +

1 + 2ma
n
0

1 + 2ma
n
1 ma
n
0

1 + 2ma
n
1 ma
n
( ma
n
1)
1 + 2ma
n
1 + m
2
a
2
n
2ma
n
4ma
n
m
2
a
2
n
4m m
2
a
n
4 ma
n
Llegamos as a una contradiccin: ma
n
1 < 4 ma
n
. Como consecuencia tenemos una
sucesin decreciente de nmeros positivos (no necesariamente enteros).
Vamos ahora a eliminar radicales.
a
n+1
=
1
4m
(ma
n
1 +

1 + 2ma
n
)
4ma
n+1
= ma
n
1 +

1 + 2ma
n
4ma
n+1
ma
n
+ 1 =

1 + 2ma
n
Al tomar cuadrados tenemos:
16m
2
a
2
n+1
+ m
2
a
2
n
+ 1 + 8ma
n+1
8m
2
a
n
a
n+1
2ma
n
= 1 + 2ma
n
ma
2
n
4(2ma
n+1
+ 1)a
n
+ 8a
n+1
(2ma
n+1
+ 1) = 0
Calculando las races se tiene:
a
n
=
4(2ma
n+1
+1)

4
2
(2ma
n+1
+1)
2
4m8a
n+1
(2ma
n+1
+1)
2m
=
4(2ma
n+1
+1)4

2ma
n+1
+1
2m
=
2(2ma
n+1
+1)2

2ma
n+1
+1
m
.
ARITMTICA Y COMBINATORIA
(I. Sucesiones recurrentes)
P. Jara
70 CAP. III. EJERCICIOS DE REPASO
ma
n
= 2(2ma
n+1
+ 1) 2
_
2ma
n+1
+ 1.
Para averiguar el signo, supongamos que ma
n
= 2(2ma
n+1
+1) +2

2ma
n+1
+ 1, entonces al
introducir este valor en la relacin original se tiene
4ma
n+1
= 2(2ma
n+1
+ 1) + 2

2ma
n+1
+ 1 1 +

2ma
n
+ 1
0 = 1 + 2

2ma
n+1
+ 1 +

2ma
n
+ 1,
lo que es imposible, pues todos los trminos del miembro de la derecha son positivos.
Se tiene entonces ma
n
= 2(2ma
n+1
+ 1) 2

2ma
n+1
+ 1.
Podemos considerar entonces las dos relaciones siguientes:
4ma
n+2
= ma
n+1
1 +

1 + 2ma
n+1
ma
n
= 2(2ma
n+1
+ 1) 2

2ma
n+1
+ 1
al sumar el doble de la primera y la segunda se obtiene:
8ma
n+2
+ ma
n
= 2ma
n+1
2 + 4ma
n+1
+ 2,
de donde resulta:
8a
n+2
= 6a
n+1
a
n
.
Resolvemos esta recurrencia con los valores iniciales a
1
= 2(m + 1) y a
2
=
m+2
2
. La ecuacin
caracterstica 8r
2
= 6r 1 tiene races
1
=
1
2
y
2
=
1
4
, entonces tenemos que resolver el
sistema de ecuaciones lineales
1
2

1
+
1
4

2
= 2(m + 1)
_
1
2
_
2

1
+
_
1
4
_
2

2
=
m+2
2
_
Se obtiene
1
= 4 y
2
= 8m, luego el trmino general es:
a
n
= 4
_
1
2
_
n
+ 8m
_
1
4
_
n
= 4
_
1
2
_
n
+ 8m
_
1
2
_
2n
para n 1.
Problema. 9.29.
Sea k un entero positivo, se dene una sucesin mediante las reglas:
x
1
= k,
x
n+1
= kx
n
+
_
(k
2
1)(x
2
n
1), si n 1.
Demostrar que todos los x
n
son enteros positivos.
[Competicin Krschak. Hungra. 1988]
28 de enero de 2013 Curso 20122013. NOTAS DE TRABAJO, 30
SEC. 9. EJERCICIOS DE REPASO 71
SOLUCIN. Vamos a eliminar radicales; se tiene:
x
n1
kx
n
=
_
(k
2
1)(x
2
n
1)
x
2
n+1
+ k
2
x
2
n
2kx
n
x
n+1
= (k
2
1)(x
2
n
1)
x
2
n
x
n
(2kx
n+1
) + (x
2
n+1
+ k
2
1) = 0
Al resolver esta ecuacin se tiene:
x
n
=
2kx
n+1

4k
2
x
2
n+1
4(x
2
n+1
+k
2
1)
2
= kx
n+1

_
(k
2
1)(x
2
n+1
1)
Supongamos que x
n
= kx
n+1
+
_
(k
2
1)(x
2
n+1
1), como
x
n+1
= kx
n
+
_
(k
2
1)(x
2
n
1)
= k(kx
n+1
+
_
(k
2
1)(x
2
n+1
1)) +
_
(k
2
1)((kx
n+1
+
_
(k
2
1)(x
2
n+1
1))
2
1),
entonces
(1 k
2
)x
n+1
= k
_
(k
2
1)(x
2
n+1
1) +
_
(k
2
1)((kx
n+1
+
_
(k
2
1)(x
2
n+1
1))
2
1)
lo que implica que (1 k
2
)x
n+1
0, pero (1 k
2
) 0, luego x
n+1
< 0, pero como x
1
> 0 la la
sucesin es creciente, llegamos a una contradiccin.
Tenemos entonces x
n
= kx
n+1

_
(k
2
1)(x
2
n+1
1). De aqu obtenemos:
x
n
= kx
n+1

_
(k
2
1)(x
2
n+1
1)
x
n+2
= kx
n+1
+
_
(k
2
1)(x
2
n+1
1.
Al sumar se tiene
x
n+2
+ x
n
= 2kx
n+1
,
de donde x
n+2
= 2kx
n+1
x
n
. La ecuacin caracterstica es: r
2
= 2kr 1, cuyas races son:

1
= k +

k
2
1 y
2
= k

k
2
1. El trmino general es:
x
n
=
1
(k +
_
k
2
1)
n1
+
2
(k
_
k
2
1)
n1
;
al imponer las condiciones iniciales x
1
= k y
x
2
= kx
1
+
_
(k
2
1)(x
2
1
1) = k
2
+
_
(k
2
1)
2
= 2k
2
1,
ARITMTICA Y COMBINATORIA
(I. Sucesiones recurrentes)
P. Jara
72 CAP. III. EJERCICIOS DE REPASO
tenemos que resolver el sistema de ecuaciones lineales

1
+
2
= k
(k +

k
2
1)
1
+ (k

k
2
1)
2
= 2k
2
1
_
La solucin es:
1
=
k+

k
2
1
2
y
2
=
k

k
2
1
2
. Tenemos que el trmino general es:
x
n
=
k+

k
2
1
2
(k +

k
2
1)
n1
+
k

k
2
1
2
(k

k
2
1)
n1
=
1
2
(k +

k
2
1)
n
+
1
2
(k

k
2
1)
n
=
1
2

i,par
_
n
i
_
2k
ni

k
2
1
i
=

i,par
_
n
i
_
k
ni
(k
2
1)
i/2
,
que es un entero positivo.
Problema. 9.30.
Sean m un entero positivo y p un entero arbitrario. Se dene la sucesin {x
n
}
n
mediante
x
1
= 0,
x
n+1
= mx
n
+
_
(m
2
1)x
2
n
+ p
2
, si n 1.
Demostrar que todos los trminos de la sucesin son enteros.
[Gazeta Matematica. Rumania. 1990]
SOLUCIN. Vamos a eliminar radicales:
x
n+1
mx
n
=
_
(m
2
1)x
2
n
+ p
2
x
2
n+1
+ m
2
x
2
n
2mx
n
x
n+1
= (m
2
1)x
2
n
+ p
2
x
2
n
2mx
n+1
x
n
+ x
2
n+1
p
2
= 0
Al resolver esta ecuacin se tiene:
x
n
=
2mx
n+1

4m
2
x
2
n+1
4x
2
n+1
+4p
2
2
=
2mx
n+1
2

x
2
n+1
(m
2
1)+p
2
2
= mx
n+1

_
x
2
n+1
(m
2
1) + p
2
.
La sucesin {x
n
}
n
es creciente, ya que x
1
= 0 y el resto se deduce de la ecuacin de re-
currencia. Vamos a determinar qu signo es el correcto. Si suponemos que x
n
= mx
n+1
+
_
x
2
n+1
(m
2
1) + p
2
, se verica:
x
n+1
= mx
n
+
_
(m
2
1)x
2
n
+ p
2
= m(mx
n+1
+
_
x
2
n+1
(m
2
1) + p
2
) +
_
(m
2
1)(mx
n+1
+
_
x
2
n+1
(m
2
1) + p
2
)
2
+ p
2
28 de enero de 2013 Curso 20122013. NOTAS DE TRABAJO, 30
SEC. 9. EJERCICIOS DE REPASO 73
De aqu se tiene
(1 m
2
)x
n+1
= m(
_
x
2
n+1
(m
2
1) + p
2
) +
_
(m
2
1)(mx
n+1
+
_
x
2
n+1
(m
2
1) + p
2
)
2
+ p
2
Tenemos que (1 m
2
)x
n+1
> 0, como (1 m
2
) 0, se tiene x
n+1
< 0, lo que es una contra-
diccin.
Obtenemos pues x
n
= mx
n+1

_
x
2
n+1
(m
2
1) + p
2
. Con sta y la relacin inicial se tiene:
x
n+2
= mx
n+1
+
_
(m
2
1)x
2
n+1
+ p
2
x
n
= mx
n+1

_
(m
2
1)x
2
n+1
+ p
2
,
sumando se tiene:
x
n+2
+ x
n
= 2mx
n+1
.
Tenemos la ecuacin caracterstica r
2
= 2mr 1, cuyas races son:
1
= m +

m
2
1 y

2
= m

m
2
1. Las condiciones iniciales son: x
1
= 0 y
x
2
= mx
1
+
_
(m
2
1)x
2
1
+ p
2
= p.
Tenemos entonces el sistema de ecuaciones lineales

1
+
2
= 0
(m +

m
2
1)
1
+ (m

m
2
1)
2
= p
_
Las races son:
1
=
p
2

m
2
1
y
2
=
p
2

m
2
1
. Entonces el trmino general es:
x
n
= (m +

m
2
1)
n1

1
+ (m

m
2
1)
n1

2
= (m +

m
2
1)
n1
p
2

m
2
1
+ (m

m
2
1)
n1
p
2

m
2
1
=
p
2

m
2
1

i,impar
2
_
n 1
i
_
m
ni1
(

m
2
1)
i
= p

i,impar
_
n 1
i
_
m
ni1
(

m
2
1)
i1
2
que es un entero.
Problema. 9.31.
Sea p(x) un polinomio de grado n que verica
p(j) = 2
j1
, para j = 1, 2, 3, . . . , n, n + 1.
Determinar p(n + 2).
[Olimpada de Israel 1988].
ARITMTICA Y COMBINATORIA
(I. Sucesiones recurrentes)
P. Jara
74 CAP. III. EJERCICIOS DE REPASO
SOLUCIN. Consideramos la sucesin p(1), p(2), . . . , p(n+1); como p(x) es un polinomio de
grado n, entonces esta sucesin es una progresin aritmtica de orden n, a la cual podemos
calcular el trmino general. Veamos cuales son las diferencias sucesivas. Tenemos p(i) =
p(i + 1) p(i) = 2
i
2
i1
= 2
i1
= p(i). En general se tiene
t
p(i) = p(i), siendo siempre
i < n + 1 t. Cuando t = n slo tenemos una diferencia, y sta es
n
p(1) = 1.
Podemos calcular el trmino general aplicando la teora que hemos desarrollado o bien la
teora ms general de interpolacin de Lagrange.
Otra forma directa, para este caso, es la siguiente:
p(n + 2) = p(n + 1) + p(n + 1) = p(n + 1) + p(n + 1)
= p(n + 1) + p(n) + p(n) = p(n + 1) + p(n) + p(n)
=
= p(n + 1) + p(n) + p(n 1) + + p(2) + p(2)
= p(n + 1) + p(n) + p(n 1) + + p(2) + p(1) + p(1)
= 2
n
+ 2
n1
+ 2
n2
+ + 2 + 1 + 1
= 2
n+1
1 + 1 = 2
n+1

Problema. 9.32.
Sean {x
n
}
n
y {y
n
}
n
sucesiones de enteros positivos denidas por:
x
0
= 1 = x
1
, x
n+1
= x
n
+ 2x
n1
, si n 1.
y
0
= 1, y
1
= 7, y
n+1
= 2y
n
+ 3y
n1
, si n 1.
Demostrar que x
0
= x
1
= y
0
= 1 es el nico trmino que tienen en comn ambas sucesio-
nes.
[Olimpada EEUU. 1973]
SOLUCIN. Calculamos los trminos generales de las sucesiones {x
n
}
n
y {y
n
}
n
:
x
n
=
2
3
2
n
+
1
3
(1)
n
,
y
n
= 2 3
n
(1)
n
.
Supongamos que existen n y m tales que x
n
= y
m
, entonces se tiene:
2
3
2
n
+
1
3
(1)
n
= 2 3
m
(1)
m
2
n+1
+ (1)
n
= 2 3
m+1
3(1)
m
2
n+1
2 3
m+1
= ((1)
n
+ 3(1)m)
2
n
3
m+1
=
1
2
((1)
n
+ 3(1)
m
)
28 de enero de 2013 Curso 20122013. NOTAS DE TRABAJO, 30
SEC. 9. EJERCICIOS DE REPASO 75
Los valores del miembro de la derecha son:
n par m par
1
2
(1 + 3) = 2
n par m impar
1
2
(1 3) = 1
n impar m par
1
2
(1 + 3) = 1
n impar m impar
1
2
(1 +3) = 2
Si 2
n
3
m+1
= 2 y n = 0, entonces 2 | 3
m+1
, lo que es una contradiccin; si n = 0, entonces
1 3
m+1
= 2, y resulta 3
m+1
= 3, esto es, m+1 = 1, o equivalentemente m = 0. (ste da
el caso x
0
= 1 = y
0
)
Si 2
n
3
m+1
= 2, como n = 0, ya que n es impar, resulta 2 | 3
m+1
, lo que es una contradiccin.
Si 2
n
3
m+1
= 1, tenemos que n es par; sea n = 2t, y m es impar, sea m + 1 = 4h; tenemos:
2
2t
= 3
m+1
+ 1 = 3
2h
+ 1 = 9
h
+ 1.
Tomando clases de resto mdulo 4 tenemos 0 1
h
+ 1 (mod 4), lo que es una contradiccin.
Si 2
n
3
m+1
= 1, tenemos que n es impar, sea n = 2t + 1 y m es par, sea m = 2h, entonces:
2
2t+1
= 3
m+1
1 = 3
2h+1
. 1
Si t = 0, entonces 2 = 3
2h+1
1, tomando h = 0 tenemos una solucin. Si t = 0, entonces
8 | 2
2t+1
y tomando clases de resto mdulo 8 resulta 0 3
2h+1
1 9 3 1 3 1 2
(mod 8), lo que es una contradiccin. (ste da el caso x
1
= 1 = y
0
)
Problema. 9.33.
Tenemos un plano con dos puntos pintados de rojo y n puntos pintados de negro. Podemos
pintar segmentos solo entre puntos de distinto color.
(1) Calcular el nmero mnimo de segmentos que son necesarios para que todos los puntos
estn conectados.
(2) De cuntas formas diferentes se puede dibujar este nmero mnimo de segmentos para
que todos los puntos estn conectados.
SOLUCIN. (1). Para que todos los puntos estn conectados, de cada punto negro debe salir
un segmento, y como el otro extremos ha de ir a un punto rojo, tenemos al menos n segmen-
tos. Pero en esta situacin los dos puntos rojos no estn conectados, luego necesitamos un
segmento ms, de forma que una punto negro est unido a los dos puntos rojos. De forma
que el nmero mnimo de segmentos es n + 1.
(2). La forma es tener el nmero mnimo de segmentos para el caso de n puntos negros se
puede construir a partir de una situacin con n 1 puntos. Por ello si a
n
es el nmero de
formas diferentes en que se puede dibujar el nmero mnimo para n puntos negros, y como
a partir de una situacin con n1 puntos se puede pasar a una situacin de n puntos de dos
formas distintas (uniendo el n-simo punto negro con cada uno de los puntos rojos), se tiene
la relacin a
n
= 2an 1. As pues la relacin de recurrencia es: a
n
= 2a
n1
, obteniendo pues
ARITMTICA Y COMBINATORIA
(I. Sucesiones recurrentes)
P. Jara
76 CAP. III. EJERCICIOS DE REPASO
una progresin geomtrica de razn 2. El trmino a
0
no tiene signicado, y el trmino a
1
es
igual a 2. Tenemos entonces la frmula a
n
= 2
n
.
e e
u M
M
M
M
M
M
M
M
M
M
M

e e
u
M
M
M
M
M
M
M
M
M
M

U
U
U
U
U
U
U
U
U
U
U

u
e e
u M
M
M
M
M
M
M
M
M
M
M

U
U
U
U
U
U
U
U
U
U
u u
]
]
]
]
]
]
]
]
]

Problema. 9.34.
Determinar una frmula explcita para el trmino general de la sucesin denida:
a
0
= 1,
a
n
=
a
n1
1+a
n1
, si n 1.
SOLUCIN. Tenemos
a
1
=
1
1 + 1
=
1
2
,
a
2
=
1/2
1 + 1/2
=
1
3
.
Hacemos induccin sobre n; supongamos que se tiene a
n
=
1
n
, entonces se verica:
a
n+1
=
a
n
1 + a
n
=
1/n
1 + 1/n
=
1/n
(n + 1)/n
=
1
n + 1
.

Problema. 9.35.
Calcular la suma
1 2 3 + 2 3 4 + + n(n + 1)(n + 2)
SOLUCIN. Llamamos a
n
= 1 2 3 + 2 3 4 + + n(n + 1)(n + 2) y a
0
= 0, entonces la
sucesin {a
n
}
n
es una progresin aritmtica de orden cuatro.
b
[4]
a
0
a
1
a
2
a
3
a
4
a
5
b
[3]
1 2 3 2 3 4 3 4 5 4 5 6 5 6 7
b
[2]
3 2 3 3 3 4 3 4 5 3 5 6
b
[1]
3 2 3 3 2 4 3 2 5 3 2 5
b
[0]
3 2 3 2 3 2

28 de enero de 2013 Curso 20122013. NOTAS DE TRABAJO, 30


SEC. 9. EJERCICIOS DE REPASO 77
Problema. 9.36.
Se considera para cada entero positivo o nulo n el polgono regular de n + 3 lados, y llama-
mos a
n
al nmero de diagonales de dicho polgono.
Dar una expresin explcita de a
n
en funcin de n y comprobar que la sucesin {a
n
}
n
es una
progresin aritmtica de orden 2.
SOLUCIN. Si numeramos los vrtices de un polgono de n +3 lados a partir del 1 siguiendo
el sentido de las agujas del reloj, vamos a ver cuantas diagonales podemos construir.
Con base el vrtice 1 tenemos exactamente n diagonales, una por cada vrtice distinto de los
vrtices 1, 2 y n + 3.
Con base el vrtice 2 tenemos exactamente n diagonales, una por cada vrtice distinto de los
vrtices 2, 3 y 1.
Con base el vrtice 3 tenemos exactamente n1 diagonales, una por cada vrtice distinto de
los vrtices 3, 4 y 2, a los que hay que aadir el vrtice 1, pues la diagonal (1, 3) ya la hemos
contado.
Es claro que conbase el vrtice 4 tenemos n2 diagonales, y para el vrtice j tenemos nj+2.
Por tanto el nmero total de vrtices es:
n + n + (n 1) + (n 2) + + 2 + 1 = n +
(n + 1)n
2
=
n(n + 3)
2
.
Tenemos pues una progresin aritmtica de orden 2.
Problema. 9.37.
Se considera para cada entero positivo o nulo n el prisma recto de bases los polgonos regu-
lares de n + 3 lados, y llamamos d
n
al nmero de todas las diagonales de dicho prisma.
Dar una expresin explcita de d
n
en funcin de n y comprobar que la sucesin {d
n
}
n
es una
progresin aritmtica de orden 2.
SOLUCIN. Consideramos un prisma recto de base un polgono de n +3 lados. Numeramos
los vrtices del polgono de n+3 lados de la base superior a partir del 1 siguiendo el sentido de
las agujas del reloj, y hacemos la misma numeracin en la base de forma que los vrtices con
el mismo nmero estn unidos por una arista vertical. Veamos cuantas diagonales podemos
construir.
Tenemos todas las diagonales de las bases, en total 2a
n
siguiendo la notacin del proble-
ma (9.36.).
Para cada cara lateral, que es un rectngulo tenemos dos diagonales, en total 2(n + 3) ya que
tenemos n + 3 caras.
Slo nos queda por contar las diagonales interiores al prisma. Del vrtice 1, de la base supe-
rior, podemos construir las diagonales a los vrtices de la base inferior, una por cada vrtice
distinto del vrtice 1, tenemos entonces n diagonales, una para cada uno de los vrtices dis-
tintos de 1, 2 y n + 3. Como esta construccin la podemos hacer para cada vrtice de la base
superior, tenemos por tanto n(n + 3).
ARITMTICA Y COMBINATORIA
(I. Sucesiones recurrentes)
P. Jara
78 CAP. III. EJERCICIOS DE REPASO
Al sumar todos estos nmeros tenemos que el nmero total de diagonales es:
2
n(n + 3)
2
+ 2(n + 3) + n(n + 3) = 2(n + 1)(n + 3).
Si llamamos d
n
a este nmero, resulta que la sucesin {d
n
}
n
es una progresin aritmtica de
orden 2.
Problema. 9.38.
Sea f (1) = 1 y f (1) + f (2) + + f (n) = n
2
f (n). Calcular razonadamente f(2001).
[Gacetilla Matemtica. Problema 107]
SOLUCIN. De entre las muchas formas de resolver este problema vamos a mostrar una en
la que se utilizan progresiones aritmticas.
Tenemos
f (1) + f (2) = 2
2
f (2),
1 = (2
2
1)f (2),
f (2) =
1
3
f (1) + f (2) + f (3) = 3
2
f (3),
1 +
1
3
= (3
2
1)f (3),
f (3) =
1
6
En la misma forma se tiene f (4) =
1
10
, f (5) =
1
15
. Podemos conjeturar que se tiene f (n) =
1

n + 1
2
=
2
(n+1)n
; esto es, las denominadores son los nmeros triangulares: 1, 3, 6, 10, 15, . . . ,
con trmino general
_
n + 1
2
_
comenzando en a
1
.
Vamos a hacer induccin para probar este resultado. Supongamos que el resultado es cierto
para n y vamos a probarlo para n + 1.
f (n + 1) =
f (1)+f (2)++f (n)+f (n+1)
(n+1)
2
1
=
n
2
f (n)
(n+1)
2
1
=
n
2 1

n + 1
2

(n+1)
2
1
=
2n
2
(n+1)n
(n+1)
2
1
=
2n
2
(n+1)n((n+1)
2
1)
=
2n
2
(n+1)n(n+2)n
=
2
(n+2)(n+1)
.
28 de enero de 2013 Curso 20122013. NOTAS DE TRABAJO, 30
SEC. 9. EJERCICIOS DE REPASO 79
Para el valor que se pide, tenemos:
f (2001) =
2
(2001 + 1)2001
=
1
1001 2001
=
1
2003001
.

Problema. 9.39. (Los nmeros de Pell)


Se dene al sucesinde Pell mediante las condiciones iniciales a
0
= 0 y a
1
= 1 y la ecuacin
de recurrencia
a
n
= 2a
n1
+ a
n
, si n 2.
Determinar el trmino general de la sucesin de Pell.
SOLUCIN.

Problema. 9.40. (Nmeros de Padovan)


Se considera la sucesin recurrente denida por:
a
0
= a
1
= a
2
= 1,
a
n
= a
n2
+ a
n3
, si n 3.
Determinar el trmino general de esta sucesin. Los nmeros de la sucesin{a
n
}
n
se llaman
nmeros de Padovan.
Los nmeros de Padovan aparecen en el lugar (2, 2) (coeciente central) de la matriz M
n+2
,
siendo M la matriz
_
_
0 1 0
0 0 1
1 1 0
_
_
.
SOLUCIN.

Problema. 9.41. (Nmeros de Jacobsthal)


Se considera la sucesin {a
n
}
n
denida por:
a
0
= 0, a
1
= 1,
a
n
= a
n1
+ 2a
n2
, si n 2.
Dar una descripcinexplcita del trmino general a
n
.Demostrar que se vericanlas siguien-
tes condiciones:
(1) a
n
+1a
n1
a
2
n
= (1)
n
a
n1
.
(2)

n
i=2
a
i
=
1
2
(a
n+2
3).
SOLUCIN. .
ARITMTICA Y COMBINATORIA
(I. Sucesiones recurrentes)
P. Jara
80 CAP. III. EJERCICIOS DE REPASO
Problema. 9.42.
Se realiza un experimento que consiste en lanzar un dado hasta que aparezcan dos n-
meros pares. Determinar una relacin de recurrencia para el nmero de experimentos que
nalizan en el n-simo lanzamiento o antes.
SOLUCIN. Llamamos a
n1
al nmero de experimentos que nalizan antes del n-simo lan-
zamiento. Si un experimento naliza en el n-simo lanzamiento, puede ser debido a una de
las siguientes posibilidades:
(a). uno de los n 1 lanzamientos anteriores era par; tenemos n 1 posibilidades,
(b). si uno de los lanzamientos era para tenemos para cada uno tres posibles valores; tene-
mos 3 posibilidades,
(c). tenemos exactamente n2 nmeros impares, y en total el nmero de posibilidades para
estos es: 3
n2
.
(d). el ltimo lanzamiento es para, tenemos tres posibilidades.
En total se tiene entonces
a
n
= a
n1
+ (n 1) 3 3
n2
3 = a
n1
+ (n 1) 3
n
.
con valores iniciales a
0
= 0, a
1
= 0
Se trata ahora de dar un valor explcito para a
n
. Como es una recurrencia lineal no homog-
nea, tenemos que determinar una solucin particular y una solucin general de la recurren-
cia homognea. Comencemos por sta ltima.
La ecuacin caracterstica es: r = 1, luego una solucin general es: a
n
=
1
.
Como la parte no homognea es: (n 1)3
n
, y sta proviene de la ecuacin caracterstica (r
3)(r 3), entonces una solucin particular se obtiene al considerar la ecuacin caracterstica
(r 1)(r 3)
2
, y eliminar de la solucin la parte correspondiente a (r 1); se tiene entonces
(
2
+
3
n)3
n
; al introducir este valor en la recurrencia se tiene:
(
2
+
3
n)3
n
(
2
+
3
(n 1))3
n1
(n 1) 3
n
= 0
(
2
+
3
n)3 (
2
+
3
(n 1)) (n 1) 3 = 0
3
2
+ 3
3
n
2

3
n +
3
3n + 3 = 0
3
2

2
+
3
+ 3 + (3
3

3
3)n = 0
De aqu se obtiene
_
2
2
+
3
+ 3 = 0
2
3
3 = 0
_
cuya solucin es:
2
=
9
4
y
3
=
3
2
.
La solucin general de la recurrencia no homognea es: (
9
4
+
3
2
n)3
n
+
1
. Al imponer las
condiciones iniciales se tiene:
(
9
4
+
3
2
0)3
0
+
1
= 0,
(
9
4
+
3
2
1)3
1
+
1
= 0,
28 de enero de 2013 Curso 20122013. NOTAS DE TRABAJO, 30
SEC. 9. EJERCICIOS DE REPASO 81
luego
1
=
9
4
, y el trmino general es: a
n
= (
9
4
+
3
2
n)3
n
+
9
4
.
Problema. 9.43.
Encontrar el trmino general de la sucesin {a
n
}
n
denida por la recurrencia
a
n
= a
n2
, si n 2, a
0
= 1, a
1
= 0.
SOLUCIN. La ecuacin caracterstica es: r
2
= 1, cuya races son
1
= i y
2
= i, entonces
una solucin general es: a
n
=
1
i
n
+
2
(i)
n
, al imponer las condiciones iniciales se tiene:

1
+
2
= 1

1
i +
2
(i) = 0
_
de donde resulta:
1
= y
2
=. El trmino general es: a
n
= (
1
2
i
n
+
1
2
(i)
n
).
Problema. 9.44.
Se considera la sucesin {a
n
}
n
vericando la recurrencia
a
1
= 1,
a
n
=
n
n1
a
n1
+ n
3
, si n 2.
Determinar el trmino general.
SOLUCIN. Denimos sucesiones auxiliares {b
n
}
n
y {c
n
}
n
mediante:
b
n
=
n!
(n1
! = n, si n 1,
c
1
= 1, c
n
= c
n1
+
n
3
b
n
= c
n1
+ n
2
, si n 2.
Si suponemos que a
r
= b
r
c
r
para r < n, por induccin tenemos:
b
n
c
n
= n(c
n1
+ n
2
) = nc
n1
+ n
3
=
n
n 1
(n 1)c
n1
+ n
3
=
n
n 1
a
n1
+ n
3
= a
n
.
El trmino general de {c
n
}
n
es fcil, ya que es la suma de los n primeros cuadrados: c
n
=
2n
3
+3n
2
+n
6
, y por tanto
a
n
= b
n
c
n
= n
2n
3
+ 3n
2
+ n
6
=
2n
4
+ 3n
3
+ n
2
6
=
n
2
(n + 1)(2n + 1)
6
.

Problema. 9.45.
Se considera la sucesin {a
n
}
n
vericando la recurrencia
a
0
= 1,
a
n
= na
n1
+ (1)
n
, si n 1.
Determinar el trmino general.
ARITMTICA Y COMBINATORIA
(I. Sucesiones recurrentes)
P. Jara
82 CAP. III. EJERCICIOS DE REPASO
SOLUCIN. Denimos sucesiones auxiliares {b
n
}
n
y {c
n
}
n
mediante:
b
n
= n!,
c
0
= 1, c
n
= c
n1
+
(1)
n
b
n
= c
n1
+
(1)
n
n!
, si n 1.
Se verica a
n
= b
n
c
n
. Como el trmino general de {c
n
}
n
es
1
2!

1
3!
+ +
(1)
n
n!
,
resulta que el trmino general de a
n
es:
n!(
1
2!

1
3!
+ +
(1)
n
n!
).

28 de enero de 2013 Curso 20122013. NOTAS DE TRABAJO, 30


Captulo IV
Ejercicios avanzados
10. Ejercicios avanzados
Ejercicios avanzados
Problema. 10.1.
Se considera la sucesin {S
n
}
n
denida para cada entero positivo o nulo n mediante:
S
n
= 1 + 3 + 5 + + (2n 1)
Determinar el trmino general de esta sucesin.
SOLUCIN. Es claro que si n = 0, entonces S
0
= 1 + (1) = 0 y S
1
= 1, S
2
= 1 + 3 = 4, etc.
Adems se verica la siguiente relacin de recurrencia
S
n
= S
n1
+ (2n 1).
Como consecuencia la ecuacin caracterstica es: r = 1, pero como no es una recurrencia
homognea, el trmino no homogneo es: 2n 1. Una solucin particular ser de la forma
an
2
+ bn + c; vamos a ver cuales son los coecientes:
a
n
+ bn + c = a(n 1)
2
+ b(n 1) + c + (2n 1),
2an a + b 2n + 1 = 0
2(a 1)n + (a + b + 1) = 0,
luego
_
a 1 = 0
a b 1 = 0
_
, esto es, a = 1, b = 0 y c es indeterminado. Una solucin particular
es: n
2
. La solucin de la recurrencia homognea S
n
= S
n1
es 1, luego la solucin general es:

1
+ n
2
,
83
84 CAP. IV. EJERCICIOS AVANZADOS
sujeta a las condiciones iniciales S
0
= 0, S
1
= 1, luego tenemos:

1
+ 0
2
= 0,
de donde se tiene
1
= 0 y el trmino general es S
n
= n
2
.
Nota. Podramos haber considerado tambin la solucin de
1
+1
2
= 1, obteniendo el mismo
resultado.
Nota. Tambin podramos haber supuesto al inicio que el trmino general es S
n
= n
2
, algo
que se deduce de una simple observacin, y probar el resultado por induccin sobre n.
Problema. 10.2.
Determinar el trmino general de la sucesin {a
n
}
n
denida por la recurrencia
a
n
= 3a
n1
+ 5 7
n
, si n 1 y a
0
= 2.
SOLUCIN. Tenemos a
n
=
1
4
(35 7
n
27 3
n
).
Problema. 10.3.
Determinar el trmino general de la sucesin {a
n
}
n
denida por la recurrencia
a
n
= 3a
n1
+ 5 3
n
, si n 1 y a
0
= 2.
SOLUCIN. Tenemos a
n
= (2 + 5n)3
n
.
Problema. 10.4.
Determinar el trmino general de la sucesin {a
n
}
n
denida por la recurrencia
a
n
= 4a
n1
4a
n2
+ n
2
, si n 2 y a
0
= 0, a
1
= 2.
SOLUCIN. Tenemos a
n
=
55
54
n(2)
n
+
1
27
(3n
2
4n).
Problema. 10.5.
Determinar el trmino general de la sucesin {a
n
}
n
denida por la recurrencia
a
n
= 5a
n1
8a
n2
+ 4a
n3
, si n 3 y a
0
= 1, a
1
= 2, a
2
= 3.
SOLUCIN.

Problema. 10.6.
Determinar el trmino general de la sucesin {a
n
}
n
denida por la recurrencia
a
n
a
n1
= 4[(a
n1
a
n2
) (a
n2
an 3)], si n 2 y a
0
= 0, a
1
= 1, a
2
= 3.
SOLUCIN. a
n
= 2
n
1
28 de enero de 2013 Curso 20122013. NOTAS DE TRABAJO, 30
SEC. 10. EJERCICIOS AVANZADOS 85
Problema. 10.7.
Los dos primeros trminos de una sucesin son, respectivamente, 1 y 2. Si cada trmino es
la media aritmtica del anterior con la media aritmtica de los dos adyacentes (anterior y
posterior), determinar:
(1) Una forma explcita del trmino general de la sucesin.
(2) Probar mediante induccin la validez del resultado obtenido.
(3) Un procedimiento para calcular el trmino cuadragsimo realizando a lo ms 10 opera-
ciones.
SOLUCIN.

Problema. 10.8.
Determinar el trmino general de la sucesin {a
n
}
n
denida por la recurrencia
a
n
= 6a
n1
9a
n2
, si n 2 y a
0
= 1, a
1
= 6.
SOLUCIN. a
n
= (n + 1)3
n

Problema. 10.9.
Determinar el trmino general de la sucesin {a
n
}
n
denida por la recurrencia
a
n
= 6a
n1
9a
n2
+ 4n, si n 2 y a
0
= 1, a
1
= 6.
SOLUCIN. a
n
= (8n 6)3
n1
+ n + 3.
Problema. 10.10.
Determinar a y b, siendo a el nmero de enteros positivos menores o iguales que 100 que no
son divisores ni por 3 ni por 7 ni por 11 y b es el nmero de enteros positivos, en el mismo
rango, que son divisibles por 2 y por 9.
Dar la forma explcita del trmino general de la sucesin denida por la recurrencia
a
n
= aa
n1
(130b+)a
n2
, si n 2, a
0
= 0, A
1
= 10.
Siendo a y b los nmeros obtenidos en el apartado anterior. Probar que cada trmino de la
sucesin es un mltiplo de 10.
SOLUCIN. a = 52, b = 5 a
n
= 31
n
21
n
= (31 21)(31
n1
+31
n2
21 +31
n3
21
2
+ +
31
2
21
n3
+ 31 21
n2
+ 21
n1
)
Problema. 10.11.
Nos regalan tres sellos y decidimos comenzar una coleccin. El ao siguiente compramos
8 nuevos sellos, de forma que tenemos ya 11 sellos en nuestra coleccin. Si cada ao com-
pramos un nmero de nuevos sellos igual al doble de los que compramos el ao anterior,
al cabo de cuntos aos habremos superado los 100.000 sellos?
ARITMTICA Y COMBINATORIA
(I. Sucesiones recurrentes)
P. Jara
86 CAP. IV. EJERCICIOS AVANZADOS
SOLUCIN. a
n
= 2
n+2
5. El nmero de aos es 13.
Problema. 10.12.
Se trazan en el plano n rectas de forma que cada una de ellas corta a todas las dems y no
existen tres que se corten en un mismo punto.
Dar una frmula explcita para a
n
, el nmero de regiones en que estas n rectas dividen al
plano?
Dar una frmula para b
n
el nmero de regiones no acotadas que aparecen en la congura-
cin anterior.
SOLUCIN.

Problema. 10.13.
Existe un pas de cuyo nombre no puedo ahora acordarme cuya moneda ocial
es el euro, y que tiene los siguientes valores faciales para sus monedas y billetes:
Monedas 9 y 19 euros
Billetes 9, 19, 125 y 232 euros
Puede cambiarse algn billete de ms de 100 euros en monedas?
En caso armativo, de cuntas formas puede hacerse?
Se ha propuesto por el consejo de ministros de este pas realizar una emisin de nuevos
billetes hasta completar una serie de billetes con cien valores diferentes. El ministro de ha-
cienda ha observado que los actuales billetes cumplen la recurrencia siguiente:
a
n
= 2a
n1
a
n2
+ 329n 815, si n 3, a
1
= 9, a
2
= 19.
y ha decidido que los nuevos billetes tambin cumplan con esta relacin. Cul es el valor
del ltimo billete de la nueva serie?
SOLUCIN. a
n
=
1
4
[(1)
n
(97n 290) + 329n 486] a
100
=
1
4
[(1)
100
(97 100 290) + 329
100 486] = 10456 euros.
Problema. 10.14.
Determinar el trmino general de la sucesin {a
n
}
n
denida por la recurrencia
a
n
= 4a
n1
5a
n2
+ 2a
n3
2, si n 3 y a
0
= 2, a
1
= 5, a
2
= 11.
SOLUCIN. a
n
= 2
n
+ n
2
+ n + 1
Problema. 10.15.
Ana ha abierto una cuenta en un banco y ha realizado una imposicin por valor de 1.000
euros. Resulta que el banco le da a Ana un inters anual del 6 %, y que le hace ingresos
mensuales del inters. Qu cantidad tiene Ana al cabo de un ao?
28 de enero de 2013 Curso 20122013. NOTAS DE TRABAJO, 30
SEC. 10. EJERCICIOS AVANZADOS 87
SOLUCIN. Si llamamos a
n
a la cantidad de dinero de Ana en el mes n, resulta que a
0
= 1,000
y a
n
= a
n1
+0, 005a
n1
= 1, 005a
n1
, ya que el inters del 6 %anual se traduce en un inters
del 0,5 % mensual. S tiene entonces a
n
= 1, 005
n
a
0
.
Problema. 10.16.
Este es el cuento de la lechera para hacerse tremendamente rico. Primero ahorramos todo
lo que podamos hasta que juntemos un capital respetable, y del que podamos prescindir en
caso de necesidad, por ejemplo supongamos que en un ao podemos ahorrar 1.000 euros.
El segundo ao seguimos ahorrando y comenzamos a hacer negocios, compraventa de
productos y acciones, etc., que menos que juntar al menos 4.000 euros. Ya con esta respe-
table cantidad podemos dedicarnos a ms actividades econmicas, de forma que podemos
cada ao comprar bienes por cuatro veces nuestro capital del ao anterior y venderlo por
tres veces el capital de ese ao, parece una situacin posible verdad?
Determinar una frmula que nos d el capital al nal de n aos.
Al cabo de cuanto tiempo tendremos al menos un milln de euros de capital?
Qu cantidad tendremos al cabo de 15 aos?
SOLUCIN.

Problema. 10.17.
Se tiene la siguiente recurrencia:
a
0
= 0, a
1
= 9, a
2
= 1, a
3
= 21,
a
n
= 5a
n1
6a
n2
4a
n3
+ 8a
n4
, si n 4.
Dar una descripcin explcita del trmino general.
SOLUCIN. a
n
= (n
2
n 3)2
n
+ 3(1)
n
.
Problema. 10.18.
Se tiene la siguiente recurrencia:
a
0
= 0, a
1
= 1,
a
n
= a
n1
+ 6a
n2
+ n, si n 2.
Dar una descripcin explcita del trmino general.
SOLUCIN. a
n
=
1
36
(5 6(n + 1) + 9 3
n
8 (2)
n
).
ARITMTICA Y COMBINATORIA
(I. Sucesiones recurrentes)
P. Jara
88 CAP. IV. EJERCICIOS AVANZADOS
Problema. 10.19.
Se tiene la siguiente recurrencia:
a
0
= 1,
a
n
= 2a
n1
+ 4
n
, si n 1.
Dar una descripcin explcita del trmino general.
SOLUCIN.

Problema. 10.20.
Se considera la sucesin denida por:
a
0
= 0, a
n+1
=
_
6 + a
n
, si n 0.
(1) Prueba que {a
n
}n es una sucesin montona creciente.
(2) Prueba que {a
n
}
n
est acotada superiormente por 3.
(3) Calcula el lmite de {a
n
}
n
.
SOLUCIN.
(1) Tenemos a
1
=

6 > a
0
, Supongamos que para todos t n se verica a
t1
< a
t
, sumando
6 se tiene:
6 + a
t1
< 6 + a
t
,
calculando la raz cuadrada se tiene:
a
t
=
_
6 + a
t1
<
_
6 + a
t
= a
t+1
,
por lo tanto la sucesin es creciente.
(2) Supongamos que a
t
< 3 para cada t n, entonces 6 + a
t
< 6 + 3 = 9, y tomando raz
cuadrada resulta
a
t+1
=
_
6 + a
t
<

9 = 3.
(3) Como a
n+1
=

6 + a
n
, tomando lmite, si ste existe, se tiene l =

6 + l, luego l
2
= 6 +l,
de donde resulta que l es raz de l
2
l 6. luego l = 2 3. Por lo tanto el lmite es 3.

Problema. 10.21.
Determina el nmero a
n
de permutaciones del conjunto {1, 2, . . . , n} que verican |(i)
i| 1 para i = 1, 2, . . . , n.
28 de enero de 2013 Curso 20122013. NOTAS DE TRABAJO, 30
SEC. 10. EJERCICIOS AVANZADOS 89
SOLUCIN. Consideramos que n permanece jo, entonces el nmero de las permutaciones
pedidas es el nmero de permutaciones de este tipo del conjunto {1, 2, . . . , n1}, por lo tanto
son a
n1
Ahora consideramos que n no es jo, en este caso se tiene (n) = n 1, y tambin se verica
(n1) = n. Resulta que el nmero de permutaciones pedidas es en ese caso el del conjunto
{1, 2, . . . , n 2}, por lo tanto son a
n2
.
Los valores iniciales son: a
0
= 1, a
1
= 1 y a
2
= 2, por lo tanto la sucesin es la de Fibonacci
{f
n
}
n
pero corrida una posicin, esto es, a
n
= f
n+1
. Tenemos pues a
n
=

5
5

n+1

n+1
).
Problema. 10.22.
Determina el nmero b
n
de permutaciones del conjunto circular {1, 2, . . . , n} que veri-
can |(i) i| 1. (En este caso suponemos que 1 = n + 1 cuando sea necesario.)
SOLUCIN. Analizamos los distinto casos segn n.
1. n queda jo; tenemos exactamente a
n1
permutaciones, siendo {a
n
}
n
la sucesin del pro-
blema (10.21.).
2. (n) = n 1. Puede ocurrir que (n 1) = n, en este caso tenemos exactamente a
n1
permutaciones. Por el contrario, si (n 1) = n tiene que ser (1) = n, lo que obliga a que
(2) = 1, (3) = 2, . . . , (n 1) = n 2, que es una permutaciones cclica. Por lo tanto el
nmero total de permutaciones en este caso es: a
n2
+ 1.
3. Si (n) = 1. Puede ocurrir que (1) = n, y tenemos a
n2
permutaciones o por el contrario
que (1) = n, eneste caso tenemos la permutacincircular: (1) = 2, (2) = 3, . . . , pi(n1) =
n. El nmero total de permutaciones es: a
n2
+ 2.
Tenemos entonces b
n
= a
n1
+ 2a
n2
+ 2. Vamos a calcular este valor. Sabemos que a
n
=
a
n1
+a
n2
, luego se tiene: b
n
= a
n
+a
n2
+2. Por otro lado a
n
=

5
5
(
n+1

n+1
), entonces:
b
n
=

5
5
(
n+1

n+1
+
n1

n1
) + 2 =

5
5
(
n1
(
2
+ 1)
n1
(
2
+ 1)) + 2
=
n1

5(
2
+1)
5

n1

5(
2
+1)
5
+ 2 =
n1

n1
() + 2
=
n
+
n
+ 2.

ARITMTICA Y COMBINATORIA
(I. Sucesiones recurrentes)
P. Jara
90 CAP. IV. EJERCICIOS AVANZADOS
28 de enero de 2013 Curso 20122013. NOTAS DE TRABAJO, 30
Captulo V
Ejemplos
11. Ejemplos
Ejemplos
Ejemplo. 11.1.
Se considera nrectas enel plano de forma que se cortendos a dos y no hay tres coincidentes.
Llamamos a
n
al nmero de regiones en que se divide el plano cuando tenemos n rectas, y
llamamos b
n
al nmero de regiones no acotadas. Dar frmulas para a
n
y b
n
.
SOLUCIN. Si tenemos una sola recta, entonces a
1
= 2 = b
1
. Para el caso n = 2 resulta
a
2
= 4 = b
2
. Para n = 3 ya tenemos regiones no acotadas. En efecto, la nueva recta corta a
cada una de las existentes y hace aparece nuevas regiones, exactamente tres, una ms que
las rectas previamente existentes. Para ver esto basta con considerar los puntos de corte de
la nueva recta con las rectas existentes, en este caso dos, que dividen a la nueva recta en tres
segmentos, una acotado y dos no acotados; cada uno de estos segmentos no acotados es
una frontera de regiones no acotadas, y el segmento acotado determina una regin acotada
y otra no acotada. As pues en este caso resulta a
3
= 4 + 3 = 7, y b
3
= 4 + 2 = 6. Para
n > 3 tenemos a
n
= a
n1
+ n, ya que la nueva recta queda dividida en n segmentos, por
los puntos de corte con las rectas existentes; y cada segmento divide regiones previas en dos,
luego tenemos n nuevas regiones. Para contar las regiones no acotadas, basta ver que los
segmentos no acotados dividen regiones no acotadas en regiones no acotadas, mientras que
un segmento acotado divide una regin acotada en dos regiones acotadas, y una regin no
acotada en dos, una acotada y la otra no, luego tenemos la recurrencia b
n
= b
n1
+ 2.
Caso de la sucesin a
n
. La frmula de recurrencia es a
n
= a
n1
+ n, luego la recurrencia
homognea es a
n
= a
n1
; la solucin homognea es la sucesin contante igual a 1. La solu-
cin de la parte no homognea es
2
n
2
+
1
n +
0
. Al comprobar la frmula de recurrencia
91
92 CAP. V. EJEMPLOS
tenemos:

2
n
2
+
1
n +
0
=
2
(n 1)
2
+
1
(n 1) +
0
+ n
0 = 2
2
n +
2

1
+ n

2
=
1
2
=
1
La solucin general es: +
1
2
(n
2
+ n); al imponer las condiciones iniciales tenemos:
+ 1 = 2
+
1
2
(4 + 2) = 4
+
1
2
(9 + 3) = 7
_
_
_
La solucin es: = 1, luego el trmino general a
n
es: a
n
= 1 +
1
2
n(n + 1).
Caso de la sucesin b
n
. La frmula de recurrencia es b
n
= b
n1
+ 2, luego la recurrencia ho-
mognea es b
n
= b
n1
; la solucin homognea es la sucesin contante igual a 1. La solucin
de la parte no homognea es
1
n +
0
. Al comprobar la frmula de recurrencia tenemos:

1
n +
0
=
1
(n 1) +
0
+ 2
0 =
1
+ 2

1
= 2
La solucin general es: + 2n; al imponer las condiciones iniciales tenemos:
+ 2 = 2
+ 2(2) = 4
+ 2(3) = 6
_
_
_
La solucin es: = 0, luego el trmino general b
n
es: b
n
= 2n.
Ejemplo. 11.2.
Se considera un conjunto de 2
n
, n 1, nmeros reales. Cuntas comparaciones son nece-
sarias hacer, entre pares de estos nmeros, para determinar el mximo y el mnimo de este
conjunto?
SOLUCIN. Llamamos a
n
el nmero de comparaciones necesarias en el caso de n. Tene-
mos a
1
= 1. Para n 2, para calcular a
n
consideramos el conjunto S = {x
1
, x
2
, . . . , x
2
n}. Si
descomponemos S = {x
1
, x
2
, . . . , x
2
n1} {x
2
n1
+1
, x
2
n1
+2
, . . . , x
2
n} y aplicamos el paso pre-
vio, necesitamos entonces 2a
n1
comparaciones. Finalmente comparamos los mximos y los
mnimos de cada conjunto, por lo que necesitamos hacer dos comparaciones ms. Entonces
a
n
= 2a
n1
+ 2 = 2(a
n1
+ 1), si n 1.
La solucin general de la resolucin homognea es 2
n
, y la solucin de la pate no homog-
nea es la sucesin contante, luego una solucin genrica ser . Comprobamos la frmula de
28 de enero de 2013 Curso 20122013. NOTAS DE TRABAJO, 30
SEC. 11. EJEMPLOS 93
recurrencia para esta solucin genrica:
= 2 + 2
= 2
La solucin general es: 2
n
2, y al imponer las condiciones iniciales tenemos:
2 2 = 1
Entonces =
3
2
. La solucin es: a
n
=
3
2
2
n
2 = 3 2
n1
2, si n 1.
Ejemplo. 11.3.
Pablo pide un anticipo de A euros que debe pagar en t plazos. Si llamamos i al inters en
cada uno de los plazos, qu cantidad, contante, P debe pagar Pablo al nal de cada plazo?
SOLUCIN. Llamamos a
n
a la cantidad de dinero que debe al nal del plazo n-simo. Tene-
mos la siguiente relacin de recurrencia: a
n+1
= a
n
+ ia
n
P, con las condiciones a
0
= A,
a
t
= 0 y 0 n t 1.
Consideramos la relacin de recurrencia homognea: a
n+1
= (1 + i)a
n
; una solucin general
es (1 + i)
n
.
La parte no homognea es P, una solucin genrica es ; el imponer la frmula de recu-
rrencia tenemos:
= (1 + i) P
=
P
i
La solucin general es: (1 + i)
n
+
P
i
; si imponemos las condiciones iniciales resulta:
(1 + i)
0
+
P
i
= A
(1 + i)
t
+
P
i
= 0
_
= A
P
i
(1 + i)
t
+
P
i
= 0
_
_
A
P
i
_
(1 + i)
t
+
P
i
= 0
P
i
_
1 (1 + i)
t
_
= A(1 + i)
t
P =
A(1 + i)
t
i
1 (1 + i)
t
=
A(1 + i)
t
i
(1 + i)
t
1
=
Ai
1
1
(1+i)
t

Ejemplo. 11.4.
Resolver la relacin de recurrencia a
n
= 3a
n1
+ 5 3
n
con la condicin inicial a
0
= 2.
SOLUCIN. Tenemos a
n
= 3a
n1
+f (n), y consideramos la relacin de recurrencia homog-
nea a
n
= 3a
n1
; una solucin general es: 3
n
.
ARITMTICA Y COMBINATORIA
(I. Sucesiones recurrentes)
P. Jara
94 CAP. V. EJEMPLOS
Consideramos tambin la solucin del trmino no homogneo, eneste caso n3
n
. Al sustituir
en la frmula de recurrencia se tiene
n3
n
= 3(n 1)3
n1
+ 5 3
n
,
simplicando se obtiene = 5. Una solucin particular es: 5n3
n
.
La solucin general es: 3
n
+ 5n3
n
; al imponer las condiciones iniciales resulta:
= 2.
La solucin es: a
n
= 2 3
n
+ 5n3
n
= (2 + 5n) 3
n
.
Ejemplo. 11.5.
Resolver la relacin de recurrencia a
n
= 3a
n1
+ 5 7
n
con la condicin inicial a
0
= 2.
SOLUCIN. Tenemos a
n
= 3a
n1
+f (n), y consideramos la relacin de recurrencia homog-
nea a
n
= 3a
n1
; una solucin general es: 3
n
.
Consideramos tambin la solucin del trmino no homogneo, en este caso 7
n
. Al sustituir
en la frmula de recurrencia se tiene
7
n
= 37
n1
+ 5 7
n
,
simplicando se obtiene =
35
4
. Una solucin particular es:
35
4
7
n
=
5
4
7
n+1
.
La solucin general es: 3
n
+
5
4
7
n+1
; al imponer las condiciones iniciales resulta:
=
27
4
.
La solucin es: a
n
=
27
4
3
n
+
5
4
7
n+1
=
5
4
7
n+1

1
4
3
n+3
.
Ejemplo. 11.6.
Resolver la relacin de recurrencia a
n
= a
n1
+ 3n
2
con la condicin inicial a
0
= 7.
SOLUCIN. Tenemos a
n
= a
n1
+ f (n), y consideramos las siguientes relaciones:
a
1
= a
0
+ f (1)
a
2
= a
1
+ f (2) = a
0
+ f (1) + f (2)
.
.
.
a
n
= a
0
+ f (1) + + f (n)
Se tiene entonces a
n
= a
0
+

n
i=1
3i
2
= a
0
+ 3

n
i=1
i
2
= 7 + 3
n(n+1)(2n+1)
2
.
Ejemplo. 11.7.
Sea a
n
el nmero de sucesiones de los dgitos 0, 1 y 2 en las que no hay dos ceros consecu-
tivos. Determinar una frmula para a
n
.
28 de enero de 2013 Curso 20122013. NOTAS DE TRABAJO, 30
SEC. 11. EJEMPLOS 95
SOLUCIN. Para n = 1 tenemos a
1
= 3. Para n = 2 tenemos a
2
= 8, ya que la posibles
sucesiones son: 0,1; 0,2; 1,0; 1,1; 1,2; 2,0; 2,1; 2,2.
La forma de construir una sucesin de longitud n a partir de una de longitud n 1 es agre-
gando un 1 y 2; de esta forma tenemos todas las posibles sucesiones que acaban en 1 2.
Si una sucesin de longitud n acaba en 0, entonces los n 2 primeros trminos pueden ser
arbitrarios, pero el trmino n 1 es o bien 1 o bien 0; esto signica que el valor es a
n
es:
a
n
= 2a
n1
+ 2a
n2
La ecuacin caracterstica es: r
2
= 2r + 2; las soluciones de esta ecuacin son: = 1 +

3 y
= 1

3.
La solucin general es:

1
(1 +

3)
n
+
2
(1

3)
n
.
Al imponer las condiciones iniciales resulta:

1
(1 +

3) +
2
(1

3) = 3

1
(1 +

3)
2
+
2
(1

3)
2
= 8
_

1
(1 +

3) +
2
(1

3) = 3

1
(4 + 2

3) +
2
(4 2

3) = 8
_
Al multiplicar la primera ecuacin por 2 y restarla a la segunda se obtiene: 2(
1
+
2
) = 2,
esto es,
1
+
2
= 1. Al sustituir este valor en la primera ecuacin resulta:
1
(1 +

3) + (1

1
)(1

3) = 3, esto es, (2
1
1)

3 + 1 = 3; y de aqu
1
=
3+2

3
3
y
2
=
32

3
6
.
El resultado nal es:
a
n
=
3 + 2

3
3
(1 +

3)
n
+
3 2

3
6
(1

3)
n
.

Ejemplo. 11.8.
Sea a
n
el nmero de sucesiones de los dgitos 0, 1 y 2 en las que no hay dos uno dos ceros
consecutivos. Dar una frmula para a
n
.
Ver el problema (9.5.).
SOLUCIN. Para n = 1 tenemos a
1
= 3. Para n = 2 tenemos a
2
= 7, ya que las sucesiones
que podemos construir son: 0,1; 0,2;1,0; 1,2;2,0; 2,1; 2,2. Para n = 3 tenemos a
3
= 17, ya que
las sucesiones que podemos construir son: 0,1,0; 0,1,2; 0,2,0; 0,2,1; 0,2,2;
1,0,1; 1,0,2; 1,2,0; 1,2,1; 1,2,2;
2,0,1; 2,0,2; 2,1,0; 2,1,2; 2,2,0; 2,2,1; 2,2,2.
A partir de una sucesin se longitud n 1 podemos construir otra de longitud n agregando
un 2; si el lugar n1 es distinto de 2, entonces podemos construir una secesin de longitud n
agregando 0 1, segn corresponda; si el lugar n1 es igual a 2, entonces podemos construir
ARITMTICA Y COMBINATORIA
(I. Sucesiones recurrentes)
P. Jara
96 CAP. V. EJEMPLOS
dos de longitud n, agregando 0 y 1; el nmero de sucesiones vlidas de longitud n 1 con un
dos en el lugar n 1 es exactamente a
n2
, entonces tenemos la siguiente frmula para a
n
.
a
n
= a
n1
+ a
n1
+ a
n2
= 2a
n1
+ a
n2
.
Vamos a comprobarlo: a
3
= 2a
2
+ a
1
= 2 7 + 3 = 17. Otra comprobacin se puede hacer al
calcular a
4
; ser a
4
= 2a
3
+ a
2
= 2 17 + 7 = 41; en efecto, tenemos
0,1,0,2; 0,1,2,2; 0,2,0,2; 0,2,1,2; 0,2,2,2;
1,0,1,2; 1,0,2,2; 1,2,0,2; 1,2,1,2; 1,2,2,2;
2,0,1,2; 2,0,2,2; 2,1,0,2; 2,1,2,2; 2,2,0,2; 2,2,1,2; 2,2,2,2;
0,1,0,1; 0,1,2,0; 0,2,0,1; 0,2,1,0; 0,2,2,0;
1,0,1,0; 1,0,2,0; 1,2,0,1; 1,2,1,0; 1,2,2,0;
2,0,1,0; 2,0,2,1; 2,1,0,1; 2,1,2,0; 2,2,0,1; 2,2,1,0; 2,2,2,0;
0,1,2,1; 0,2,2,1;
1,0,2,1; 1,2,2,1;
2,0,2,1; 2,1,2,1; 2,2,2,1.
Tenemos entonces que la ecuacin caracterstica es:
r
2
= 2r + 1,
cuyas races son: = 1 +

2 y = 1

2. Una solucin general es:

1
(1 +

2)
n
+
2
(1

2)
n
;
al imponer las condiciones iniciales resulta:

1
(1 +

2) +
2
(1

2) = 3

1
(1 +

2)
2
+
2
(1

2)
2
= 7
_

1
(1 +

2) +
2
(1

2) = 3

1
(3 + 2

2) +
2
(3 2

2)
2
= 7
_
Multiplicando la primera ecuacin por 2 y restando a la segunda se tiene
1
+
2
= 1. Sus-
tituyendo este valor en la primera ecuacin resulta: (2
1
1)

2 + 1 = 3, luego
1
=
1+

2
2
y
entonces
2
=
1

2
2
.
Vamos a comprobar que realmente esta es la solucin. En efecto,
a
1
=
1+

2+1
2
(1 +

2) +
1

2
2
(1

2) =
3+2

2
2
+
3

2
2
= 3.
a
2
=
1+

2
2
(1 +

2)
2
+
1

2
2
(1

2)
2
=
(1+

2)
3
2
+
(1

2)
3
2
= 7.
Ejemplo. 11.9.
Sea a
n
el nmero de sucesiones de 1 y 2 de forma que su suma sea n. Dar una frmula para
a
n
.
28 de enero de 2013 Curso 20122013. NOTAS DE TRABAJO, 30
SEC. 11. EJEMPLOS 97
SOLUCIN. Para n = 1 tenemos a
1
= 1. Para n = 2 tenemos a
2
= 2, ya que existen dos
sucesiones: 1,1; 2. Para n = 3 tenemos a
3
= 3, ya que existen tres sucesiones: 1,1,1; 1,2; 2,1.
De cada sucesin que suma n1 podemos construir una sucesin que suma n agregando un
1, y de cada sucesin que suma n 2 podemos construir dos sucesiones que suman n, agre-
gando un 2 agregando 1, 1; este ltimo caso ya ha sido considerado, por lo que la frmula
para a
n
es la siguiente:
a
n
= a
n1
+ a
n2
.
La ecuacin caracterstica es: r
2
= r + 1. Las races son: =
1+

5
2
y =
1

5
2
, y una solucin
general es:

1
(
1 +

5
2
)
n
+
2
(
1

5
2
)
n
.
Al imponer las condiciones iniciales resulta

1
(
1+

5
2
) +
2
(
1

5
2
) = 1

1
(
1+

5
2
)
2
+
2
(
1

5
2
)
2
= 2
_
Las soluciones son:
1
=
1+

5
2

5
y
2
=
1

5
2

5
, por lo que el resultado nal es:
a
n
=
1

5
_
1 +

5
2
_
n+1

5
_
1

5
2
_
n+1
.
Nota. Ver la sucesin de Fibonacci.
Ejemplo. 11.10.
Tenemos motocicletas y coches. Determinar el nmero de formas de acomodar motocicle-
tas y coches en una hilera de n huecos si cada motocicletas necesita un hueco y cada coche
necesita dos. (Las motocicletas y los coches son indistinguibles entre s.)
SOLUCIN. Llamamos a
n
al nmero de formas posibles para una hilera de n huecos. Si te-
nemos una ordenacin de n huecos y al nal tenemos una motocicleta, los n 1 huecos
restantes se pueden ordenar de a
n1
formas, y si tenemos al nal un coche, los n 2 huecos
restantes se pueden ordenar de a
n2
formas, por lo tanto el valor de a
n
es:
a
n
= a
n1
+ a
n2
.
Los valores iniciales son a
1
= 1 y a
2
= 2, por lo tanto la resolucin del problema es similar al
Ejercicio (11.9.).
ARITMTICA Y COMBINATORIA
(I. Sucesiones recurrentes)
P. Jara
98 CAP. V. EJEMPLOS
Ejemplo. 11.11.
Si un caso de una enfermedad infecciosa se descubre en un colegio, se p
n
la probabilidad
de que al menos un caso se descubra en la semana nsima. Se tiene la siguiente evidencia:
p
n
= p
n1

1
4
p
n2
, si n 2.
Con las condiciones iniciales p
0
= 0 y p
1
= 1. Cundo se tendr una probabilidad inferior
a
1
100
?
SOLUCIN. La ecuacin caracterstica es: r
2
= r
1
4
, y sus races son: =
1
2
con multiplicidad
dos.
Las soluciones bsicas son a
n
=
_
1
2
_
n
y b
n
= n
_
1
2
_
n
. La solucin general es:

1
_
1
2
_
n
+
2
n
_
1
2
_
n
.
Al imponer las condiciones iniciales tenemos:

1
_
1
2
_
0
+
2
0
_
1
2
_
0
= 0

1
_
1
2
_
+
2
_
1
2
_
= 1
_

1
= 0

1
_
1
2
_
+
2
_
1
2
_
= 1
_
Luego
1
= 0 y
1
= 2. La solucin de la sucesin recurrente es:
p
n
= 2n
_
1
2
_
n
.
Para que este valor sea menor que
1
100
tiene que ser 2n
_
1
2
_
n
<
1
100
, luego 100n
_
1
2
_
n1
< 1, esto
es, 100n < 2
n1
; 25n < 2
n3
. Cuando n 11 tenemos 25n > 2
n3
(275 > 2
8
= 2
4
2
4
= 256),
y para n = 12 resulta 300 < 2
9
= 512.
La solucin es para n 12 ya que la funcin 2
n3
25n es una funcin estrictamente cre-
ciente.
28 de enero de 2013 Curso 20122013. NOTAS DE TRABAJO, 30
SEC. 11. EJEMPLOS 99
Ejemplo. 11.12.
Se considera b R
+
y el determinante d
n
de la matriz
A
n
=
_
_
_
_
_
_
_
_
_
_
_
_
_
b b 0 0 0
b b b 0 0
0 b b b 0
0 0 b b b
.
.
.
.
.
.
.
.
.
.
.
.
.
.
.
.
.
.

b b 0
b b b
0 b b
_
_
_
_
_
_
_
_
_
_
_
_
_
la diagonal, la diagonal superior y la diagonal inferior tienen todas sus entradas iguales a b.
Calcular el valor de d
n
.
SOLUCIN. Tenemos d
1
= b, d
2
=

bb
bb

= 0 y d
3
=

bb0
bbb
0 bb

= b
3
.
En general A
n+1
=
_
_
_
_
_
A
n
0
.
.
.
b
0 b b
_
_
_
_
_
, luego d
n+1
=| A
n
| b | B
n
| b, siendo B
n
=
_
_
_
_
_
A
n1
0
.
.
.
b
0 0 b
_
_
_
_
_
,
entonces | B
n
|=| A
n1
| b.
Resulta entonces d
n+1
= |A
n
|b |A
n1
|b
2
= d
n
b d
n1
b
2
.
La ecuacin caracterstica es: r
2
= rb b
2
. Las races son: = b
_
1+

3
2
_
y = b
_
1

3
2
_
.
La solucin general es:

1
b
n
_
1 +

3
2
_
n
+
2
b
n
_
1

3
2
_
n
.
Al imponer las condiciones iniciales tenemos:

1
b
_
1+

3
2
_
+
2
_
1

3
2
_
= b

1
b
2
_
1+

3
2
_
2
+
2
b
2
_
1

3
2
_
2
= 0
_
_
_

1
_
1+

3
2
_
+
2
_
1

3
2
_
= b

1
_
1+

3
2
_
2
+
2
_
1

3
2
_
2
= 0
_
_
_
Cuya solucin es:
1
=
1+

3
2

3
y
2
=
1

3
2

3
.
ARITMTICA Y COMBINATORIA
(I. Sucesiones recurrentes)
P. Jara
100 CAP. V. EJEMPLOS
El trmino general es:
d
n
=
1+

3
2

3
b
n
_
1+

3
2
_
n

3
2

3
b
n
_
1

3
2
_
n
=
b
n

3
_
_
1+

3
2
_
n+1

_
1

3
2
_
n+1
_
=
b
n
(i)

3
_
_
1+i

3
2
_
n+1

_
1i

3
2
_
n+1
_
Tenemos
1+i

3
2
= cos + sin con cos =
1
2
y sin =

3
2
, luego =

3
(= 60
o
).
Entonces
_
1+i

3
2
_
n+1
= (cos

3
+i sin

3
)
n+1
= cos
(n+1)
3
+i sin
(n+1)
3
y i
_
1+i

3
2
_
n+1
= i cos
(n+1)
3

sin
(n+1)
3
.
De la misma forma resulta i
_
1i

3
2
_
n+1
= i cos
(n+1)
3
+ sin
(n+1)
3
.
Como consecuencia d
n
=
b
n

3
2 sin
(n+1)
3
.
Ejemplo. 11.13.
Resolver la sucesin recurrente
a
n
= 2(a
n1
a
n2
), n 2
con las condiciones iniciales a
0
= 1, a
1
= 2.
SOLUCIN. La ecuacin caracterstica es r
2
= 2(r 1). Las races son = 1 + i, = 1 i.
Las soluciones bsicas son: b
n
= (1 + i)
n
y c
n
= (1 i)
n
.
La solucin general es:
1
(1 + i)
n
+
2
(1 i)
n
. Al imponer las condiciones iniciales tenemos:

1
+
2
= 1

1
(1 + i) +
2
(1 i) = 2
_
La solucin es:
2
=
1+i
2
y
1
=
1i
2
.
La solucin es: a
n
=
1
2
((1 i)(1 + i)
n
+ (1 + i)((1 i)
n
)) = (1 + i)
n1
+ (1 i)
n1
.
Tenemos que 1 + i =

2(cos

4
+ sin

4
), luego (1 + i)
n
=

2
n
(cos
n
4
+ sin
n
4
). Como conse-
cuencia resulta
a
n
= (1 + i)
n1
+ (1 i)
n1
=

2
n1
(cos
(n1)
4
+ i sin
n
4
) +

2
n1
(cos
(n1)
4
i sin
n
4
)
=

2
n+1
cos
(n1)
4
.

28 de enero de 2013 Curso 20122013. NOTAS DE TRABAJO, 30


SEC. 11. EJEMPLOS 101
Ejemplo. 11.14.
Resolver la sucesin recurrente
2a
n+3
= a
n+2
+ 2a
n+1
a
n
, n 0,
con las condiciones iniciales a
0
= 0, a
1
= 1, a
2
=.
SOLUCIN. La ecuacin caracterstica es: 2r
3
= r
2
+ 2r 1. Las races son =
1
2
, = 1,
= 1.
Las soluciones bsicas son: b
n
= 1, c
n
= (1)
n
y d
n
=
_
1
2
_
n
. La solucin general es:
1
+

2
(1)
n
+
3
_
1
2
_
n
.
Al imponer las condiciones iniciales tenemos:

1
+
2
+
3
= 0

2
+
1
2

3
= 1

1
+
2
+
1
4

3
= 2
_
_
_
De aqu se obtiene
1
=
5
2
,
2
1
6
,
3
=
8
3
.
La solucin es: a
n
=
5
2
+
(1)
n
6

8
3
_
1
2
_
n
.
Ejemplo. 11.15.
Encontrar una relacin de recurrencia para el nmero de sucesiones binarias de longitud n
que no tienen dos ceros consecutivos.
SOLUCIN. Llamamos a
n
al nmero en cuestin. Es claro que a
0
= 1, a
1
= 2, a
2
= 3 y a
3
= 5,
pues las sucesiones de longitud tres son: 0,1,0; 0,1,1; 1,0,1; 1,1,0; 1,1,1.
En general dada una sucesin de n trminos, si acaba en 1, entonces los n 1 primeros tr-
minos son arbitrarios, tenemos entonces a
n1
sucesiones. Si acaba en 0, entonces el trmino
que ocupa el lugar n1 es un 1, y los primeros n2 trminos son arbitrarios, tenemos enton-
ces a
n2
. La frmula de recurrencia es: a
n
= a
n1
+ a
n2
, con la condiciones iniciales a
0
= 1
y a
1
= 2.
Nota. Ver la sucesin de Fibonacci.
ARITMTICA Y COMBINATORIA
(I. Sucesiones recurrentes)
P. Jara
102 CAP. V. EJEMPLOS
Ejemplo. 11.16.
Se considera un reactor al que en el momento cero se le inyecta un neutrn de alta energa.
En cada periodo de tiempo, por ejemplo un microsegundo (10
6
segundos) se produce el
siguiente proceso:
(1). Un neutrn se alta energa interacta con un ncleo (de material de fusin, por ejemplo
uranio) y en este proceso para el siguiente periodo se producen dos nuevos neutrones de
alta energa y uno de baja energa.
(2). Un neutrn de baja energa interacta con un ncleo, y en este proceso se produce un
electrn de alta energa y uno de baja energa.
Si suponemos una situacin ideal en la que cada neutrn libre interacta con un ncleo en
cada periodo, y si llamamos a
n
al nmero de neutrones de alta energa y b
n
al nmero de
neutrones de baja energa en el periodo nsimo. Determinar una frmula general para a
n
y b
n
.
SOLUCIN. Los valores iniciales son a
0
= 1, b
0
= 0 y la regla de formacin es:
a
n+1
= 2a
n
+ b
n
b
n+1
= a
n
+ b
n
_
Si llamamos f (x) =

a
n
x
n
y g(x) =

b
n
x
n
, entonces tenemos:
a
n+1
x
n+1
= 2a
n
x
n+1
+ b
n
x
n+1
b
n+1
x
n+1
= a
n
x
n+1
+ b
n
x
n+1
_

a
n+1
x
n+1
= 2x

a
n
x
n
+ x

b
n
x
n

b
n+1
x
n+1
= x

a
n
x
n
+ x

b
n
x
n
_
f (x) a
0
= 2xf (x) + xg(x)
g(x) b
0
= xf (x) + xg(x)
_
f (x)(1 2x) = xg(x) + a
0
xf (x) = g(x)(1 x) b
0
_
f (x)(1 2x)x = x
2
g(x) + xa
0
f (x)(1 2x)x = g(x)(1 x)(1 2x) b
0
(1 2x)
_
0 = g(x)(x
2
(1 x)(1 2x)) + xa
0
+ (1 2x)b
0
g(x) =
x
x
2
3x+1
f (x) =
g(x)(1x)
x
=
1x
x
2
3x+1
_
x
2
3x + 1 = 0, entonces x =
3

5
2
=
_
=
3+

5
2
=
3

5
2
Valor de g(x).
x
x
2
3x + 1
=
A
x
+
B
x
=
( x)A + ( x)B
( x)( x)
28 de enero de 2013 Curso 20122013. NOTAS DE TRABAJO, 30
SEC. 11. EJEMPLOS 103
x = ( x)A + ( x)B = x(A + B) + B + A
A + B = 1
B + A = 0
_
B + (1 B) = 0; B( ) = ;
B =

=
(3

5)/2

5
=
5+3

5
10
A = 1 B =
53

5
10
_
g(x) =
5 + 3

5
10
1
x
+
5 + 3

5
10
1
x
.
Valor de f (x).
1 x
x
2
3x + 1
=
A
x
+
B
x
=
( x)A + ( x)B
( x)( x)
1 x = ( x)A + ( x)B = x(A + B) + B + A
A + B = 1
B + A = 1
_
B =
1

=
1(3

5)/2

5
=
5

5
10
A = 1 B =
5+

5
10
_
f (x) =
5 +

5
10
1
x
+
5

5
10
1
x
.
Tenemos entonces
a
n
=
5+

5
10

(n+1)
+
5

5
10

(n+1)
=
5+

5
10

n+1
+
5

5
10

n+1
=
5+

5
10
_
3

5
2
_
n+1
+
5

5
10
_
3+

5
2
_
n+1
b
n
=
5+3

5
10

(n+1)
+
5+3

5
10

(n+1)
=
5+3

5
10

n+1
+
5+3

5
10

n+1
=
5+3

5
10
_
3

5
2
_
n+1
+
5+3

5
10
_
3+

5
2
_
n+1
Nota. Se tiene:
1
= y
1
x
=
1

1
1
x

=
1

_
1 +
x

+
_
x

_
2
+
_

Ejemplo. 11.17.
Dado un conjunto de n elementos distintos y r 0, llamamos a(n, r) al nmero de formas
en que se pueden elegir r elementos (incluso con repeticin) de este conjunto de n elemen-
tos. Determinar una frmula para a(n, r).
SOLUCIN. Tenemos la frmula a(m, r) = a(n 1, r) + a(n, r 1), ya que jado un elemen-
to b
1
del conjunto {b
1
, . . . , b
n
}, el valor de a(n, r) se calcular teniendo en cuenta que b
1
no
ARITMTICA Y COMBINATORIA
(I. Sucesiones recurrentes)
P. Jara
104 CAP. V. EJEMPLOS
intervenga en cuyo caso es a(n 1, r), o bien b
1
interviene, lego el valor es a(n, r 1), y en
conjunto tenemos
a(n, r) = a(n 1, r) + a(n 1, r).
Construimos las funciones generatrices
f
n
(x) =

r=0
a(n, r)x
r
,
con a(n, 0) = 1 y a(o, r) = 0 para cada n, r > 0.
La relacin anterior signica, para n 1,
f
n
(x) =

a(n, r)x
r
=

a(n 1, r)x
r
+

a(n, r 1)x
r
= f
n1
(x) + xf
n
(x)
f
n
(x)(1 x) = f
n1
(x)
f
n
(x) =
f
n1
(x)
1 x
.
Por induccin tenemos f
n
(x) =
1
(1x)
n
= (1 x)
n
, y resulta
f
n
(x) =

r
_
n
r
_
(1)
r
x
r
=

r
_
n + r 1
r
_
x
r
,
luego la solucin es: a(n, r) =
_
n+r1
r
_
.
Ejemplo. 11.18.
Se considera la recurrencia no homognea
a
n
= 3a
n1
+ n,
con la condicin inicial a
0
= 1. Determinar una frmula para a
n
.
SOLUCIN. Escribimos todos los casos desde n = 1, multiplicamos por x
n
y hacemos la
suma:

i=1
a
i
x
i
= 3

i=1
a
i1
x
i
+

i=1
ix
i
.
Denimos la funcin f (x) =

i=1
a
i
x
i
. Se verica:
f (x) a
0
= 3xf (x) +

i=1
ix
i
La funcin generatriz

i=0
ix
i
=

i=1
ix
i
es la funcin generatriz de
d
dx
1
1x
=
1
(1x)
2
=
1 + 2x + 3x
3
+ , luego
x
(1x)
2
=

i=1
ix
i
, y obtenemos:
f (x) a
0
= 3xf (x) +
x
(1 x)
2
,
28 de enero de 2013 Curso 20122013. NOTAS DE TRABAJO, 30
SEC. 11. EJEMPLOS 105
de donde
f (x) =
x/(1 x)
2
+ a
0
1 3x
=
x
(1 x)
2
(1 3x)
+
1
1 3x
Para descomponer en fracciones simples.
x
(1 x)
2
(1 3x)
=
A
1 x
+
B
(1 x)
2
+
C
1 3x
=
A(1 x)(1 3x) + B(1 3x) + C(1 x)
2
(1 x)
2
(1 3x)
x = A(1 x)(1 3x) + B(1 3x) + C(1 x)
2
= (3A + C)x
2
(4A + 3B + 2C)x + (A + B + C)
3A + C = 0
4A + 3B + 2C = 1
A + B + C = 0
_
_
_
de donde A =
1
4
, B =
1
2
y C =
3
4
. Tenemos entonces f (x) =
1/4
1x

1/2
(1x)
2
+
3/4
13x
+
1
13x
=

1/4
1x

1/2
(1x)
2
+
7/4
13x
. El coeciente de x
n
es:

1
4
1
1
2
_
2 + n 1
n
_
(1)
2n
+
7
4
3
n
=
1
4

1
2
_
n + 1
n
_
+
7
4
3
n
=
1
4

n
2
+
7
4
3
n
,
ya que
1
13x
= 1 + 3x + (3x)
2
+ (3x)
3
+ .
Nota. Hacer este ejercicio si utilizar funciones generatrices!
Ejemplo. 11.19.
Determinar cuntos subconjuntos de cuatro elementos del conjunto {1, 2, 3, . . . , 14, 15}
existen que no contienen dos enteros consecutivos.
SOLUCIN. Dado un subconjunto vlido, sea {a, b, c, d}, con a < b < c < d, tenemos que
a 1, b a, c b, d c, 15 d
son enteros y su suma es 14, siendo todos menores que 14.
Entonces cada subconjunto admisible est determinado por un quinteto a
1
, a
2
, a
3
, a
4
y a
5
,
vericando:
a
1
+ a
2
+ a
3
+ a
4
+ a
5
= 14,
0 a
1
, a
2
, a
3
, a
4
, a
5
,
2 a
2
, a
3
, a
4
_
_
_
La funcin que dene esta situacin es:
f (x) = (1 + x + x
2
+ x
3
+ )
2
(x
2
+ x
3
+ x
4
+ )
3
= x
6
(1 + x + x
2
+ x
3
+ ) = x
6
1
(1 x)
5
.
Tenemos que calcular el coeciente de x
14
en f (x), o equivalentemente el coeciente de x
8
en
1
(1x)
5
. Como
1
(1 x)
5
= (1 x)
5
=

r=0
_
5
r
_
(x)
r
=

r=0
_
5 + r 1
r
_
x
r
,
ARITMTICA Y COMBINATORIA
(I. Sucesiones recurrentes)
P. Jara
106 CAP. V. EJEMPLOS
entonces tenemos que calcular
_
5 + 8 1
6
_
=
_
12
6
_
=
_
12
4
_
=
12 11 10 9
4 3 2
=
11 10 9
2
= 99 5 = 495.

Ejemplo. 11.20.
Determinar el coeciente de x
8
en
1
(x3)(x2)
2
.
SOLUCIN. Descomponemos
1
(x3)(x2)
2
en fracciones simples:
1
(x 3)(x 2)
2
=
A
(x 3)
+
B
(x 2)
+
C
(x 2)
2
=
A(x 2)
2
+ B(x 2)(x 3) + C(x 3)
(x 3)(x 2)
2
1 = (A + B)x
2
+ (4A 5B + C)x + (4A + 6B 3C),
A + B = 0
4A 5B + C = 0
4A + 6B 3C = 1
_
_
_
La solucin es: A = 1, B = 1 y C = 1. Tenemos entonces:
1
(x3)(x2)
2
=
1
(x3)

1
(x2)

1
(x2)
2
=
1
3
1
1
x
3
+
1
2
1
1
x
2

1
4
1
(1
x
2
)
2
=
1
3

r
_
x
3
_
r
+
1
2

r
_
x
2
_
r

1
4
_
_
2
0
_
+
_
2
1
_ _
x
2
_
+
_
2
2
_ _
x
2
_
2
_
El coeciente de x
8
es:

1
3
1
3
8
+
1
2
1
2
8

1
4
_
2
8
_
1
2
8
=
1
3
9
+
1
2
9

1
4
_
2+81
8
_
1
2
8
=
1
3
9
+
1
2
9

1
2
10
_
9
8
_
=
1
3
9
+
1
2
9

1
2
10
9
=
1
3
9
+
29
2
10
=
1
3
9

7
2
10

Ejemplo. 11.21.
Comprobar que
_
2n
n
_
=

n
i=0
_
n
i
_
2
.
28 de enero de 2013 Curso 20122013. NOTAS DE TRABAJO, 30
SEC. 11. EJEMPLOS 107
SOLUCIN. Tenemos (1 + x)
2n
= ((1 + x)
n
)
2
=
__
n
0
_
+
_
n
1
_
+ +
_
n
n1
_
+
_
n
n
__
2
, entonces
_
2n
n
_
=
_
n
0
__
n
n
_
+
_
n
1
__
n
n1
_
+ +
_
n
n1
__
n
1
_
+
_
n
n
__
n
0
_
=

n
i=0
_
n
i
__
n
ni
_
=

n
i=0
_
n
i
__
n
i
_
=

n
i=0
_
n
i
_
2
.

Ejemplo. 11.22.
De cuntas formas puede unmaestrorepartir 24 libros entre cuatroalumnos si cada alumno
debe tener al menos tres y no ms de ocho.
SOLUCIN. Para cada alumno la distribucin de libros es x
3
+ x
4
+ x
5
+ x
6
+ x
7
+ x
8
y como
tenemos cuatro alumnos, la distribucin es: f (x) = (x
3
+ x
4
+ x
5
+ x
6
+ x
7
+ x
8
)
4
. Vamos a
calcular el coeciente de x
24
en f (x).
f (x) = (x
3
+ x
4
+ x
5
+ x
6
+ x
7
+ x
8
)
4
= x
12
(1 + x + x
2
+ x
3
+ x
4
+ x
5
)
4
= x
12
_
1 x
6
1 x
_
4
.
Se trata entonces de determinar el coeciente de x
12
en la expansin de
_
1x
6
1x
_
4
.
_
1x
6
1x
_
4
= (1 x
6
)
4 1
(1x)
4
=
_
1
_
4
1
_
x
6
+
_
4
2
_
x
12

_
4
3
_
x
18
+ x
24
_ _
_
4
0
_
+
_
4
1
_
(x) +
_
4
2
_
(x)
2
+
_
Este coeciente es:
1
_
4
12
_
(1)
12

_
4
1
__
4
6
_
(1)
6
+
_
4
2
__
4
0
_
=
_
4+121
12
_

_
4
1
__
4+61
6
_
+
_
4
2
__
4+01
0
_
=
_
15
12
_

_
4
1
__
9
6
_
+
_
4
2
__
3
0
_
=
_
15
3
_
4
_
9
3
_
+ 6
_
3
0
_
=
151413
32
4
987
32
+ 6 3 = 560 336 + 6 = 230.

Ejemplo. 11.23.
Determinar el coeciente de x
15
en la expansin de f (x) = (x
2
+ x
3
+ x
4
+ )
4
.
SOLUCIN. Tenemos x
2
+ x
3
+ x
4
+ = x
2
(1 + x + x
2
+ ) =
x
2
1x
.
ARITMTICA Y COMBINATORIA
(I. Sucesiones recurrentes)
P. Jara
108 CAP. V. EJEMPLOS
Tenemos f (x) =
x
8
(1x)
4
, y por tanto basta determinar el coeciente de x
7
en la expansin de
1
(1x)
4
.
1
(1 x)
4
= (1 x)
4
=

i=0
_
4
i
_
(x)
i
=

i=0
_
4 + i 1
i
_
x
r
.
Luego el coeciente es:
_
4+71
7
_
=
_
10
7
_
= 120.

Ejemplo. 11.24.
De cuntas formas se puede seleccionar r objetos de un conjunto de n objetos distintos
(est permitida la repeticin).
SOLUCIN. Si solotenemos unobjetolas diferentes formas de seleccionar 0, 1, 2, 3, . . . objetos
estn dadas por la serie geomtrica 1 + x + x
2
+ x
3
+
Si en vez de uno tenemos n objetos, las diferentes formas estarn dadas por la funcin
f (x) = (1 + x + x
2
+ x
3
+ )
n
.
Se trata de determinar el coeciente de x
r
en f (x).
f (x) =
1
(1 x)
n
= (1 x)
n
=

i=0
_
n
r
_
(x)
r
=

i=0
_
n + r 1
r
_
x
r
,
luego la solucin es:
_
n+r1
r
_
.
Ejercicio. 11.25.
Se considera la sucesin {a
n
}
n
denida por a
0
= 6, a
1
= 22, a
2
= 38 y a
n+3
= a
n+2
+
8a
n+1
12a
n
para n 0. Determina el trmino general de la sucesin.
SOLUCIN. a
n
= 2(3)
n+1
+ n2
n+1

Ejercicio. 11.26.
En un taller de reparaciones de coches un operario se dice que est en el nivel n si le faltan n
etapas para llegar a reparar el vehculo en el que est trabajando. Sabemos que desde cada
nivel n hay dos formas de llegar al nivel n 1 y tres de llegar al nivel n 2. Si llamamos a
n
al nmero de formas en que se puede llegar a reparar un vehculo estando en el nivel n y si
a
0
= 1. Da una frmula para calcular a
n
si a
1
= 2.
SOLUCIN. Observa que se tiene la relacin a
n
= 2a
n1
+ 3a
n+2
. Por tanto {a
n
}
n
es una
sucesin recurrente con polinomio caracterstico r
2
2r 3. Las races de este polinomio son
1 y 3, entonces una solucin general es: a
n
=
1
(1)
n
+
2
3
n
. Al imponer las condiciones
iniciales se tiene:
a
0
= 1 =
1
+
2
,
a
1
= 2 =
1
+ 3
2
_
28 de enero de 2013 Curso 20122013. NOTAS DE TRABAJO, 30
SEC. 11. EJEMPLOS 109
La solucin es
1
=
1
4
y
2
=
3
4
. El trmino general de la sucesin pedida es: a
n
=
1
4
(1)
n
+
o
frac343
n
=
(1)
n
4
+
3
n+1
4
.
Ejercicio. 11.27.
Se supone que la facturacin de una empresa es cada ao la media entre la del ao anterior
y la del ao siguiente. Si las ventas en el ao 2000 fueron a
0
= a y el ao 2001 fueron a
1
= b.
Calcula las ventas en el ao 2000 + n.
SOLUCIN. Si llamamos a
n
a las ventas en el ao 2000 + n se verica:
a
n
=
a
n1
+ a
n+1
2
,
a
n+1
= 2a
n
a
n1
.
Se trata de una sucesin recurrente cuyo polinomio caracterstico es r
2
2r +1 que tiene raz
1 con multiplicidad dos. La expresin de la solucin general es: a
n
=
1
1
n
+
2
n1
n
=
1
+n
2
.
Al imponer las condiciones iniciales se tiene:
a
0
= a =
1
,
a
1
= b =
1
+
2
_
La solucin es:
1
= a,
2
= b a. El trmino general de la sucesin es:
a
n
= a + n(b a) = a(1 n) + bn.

Ejercicio. 11.28.
Un sistema luminoso emite tres tipos de seales, una de las cuales dura un segundo y las
otras dos dos segundos cada una. Halla el nmero de seales diferentes que se pueden emi-
tir en n segundos suponiendo que las seales se emiten de forma continua.
SOLUCIN. Llamamos a
n
al nmero de seales que se pueden emitir en n segundos. Ob-
servar que a
1
= 1 y a
2
= 3. Adems se tiene a
n+2
= a
n+1
+ 2a
n
para n 0. Tenemos pues
una sucesin recurrente con polinomio caracterstica r
2
r 2, cuyas races son: 1 y 2. La
solucin general es: a
n
=
1
(1)
n
+
2
2
n
, que al imponer las condiciones iniciales resulta:
a
1
= 1 =
1
+ 2
2
,
a
2
= 3 =
1
+ 4
2
_
La solucin es:
1
=
1
3
y
2
=
2
3
. El trmino general de la sucesin es: a
n
=
1
3
(1)
n
+
2
3
2
n
=
(1)
n+1
3
+
2
n+1
3
.
ARITMTICA Y COMBINATORIA
(I. Sucesiones recurrentes)
P. Jara
110 CAP. V. EJEMPLOS
28 de enero de 2013 Curso 20122013. NOTAS DE TRABAJO, 30
Captulo VI
Miscelnea
12. Nmeros naturales. El principio de induccin
Se considera N el conjunto de los nmeros naturales, esto es,
N = {0, 1, 2, 3, 4, 5, . . .}
Una propiedad P de nmeros naturales determina un subconjunto X
P
de N, el formado por
todos los nmeros naturales que verican la propiedad P. Veamos un ejemplo, sea P la pro-
piedad ser mltiplo de 2, en este caso se tenemos
X
P
= {0, 2, 4, 6, . . .}.
Este conjunto se representa por 2N. Si P es la propiedad ser un nmero impar, entonces
X
P
= {1, 3, 5, 7, . . .}.
Este conjunto es el complemento de 2N, esto es, N \ 2N.
Existe una frmula para la suma de los primeros nmeros naturales hasta n :
0 + 1 + + (n 1) + n =
n(n + 1)
2
.
Esta frmula podemos comprobarla para algunos nmeros. Veamos algunos ejemplos:
n 0 + 1 + + (n 2) + (n 1)
(n1)n
2
0 0 0
1 1 1
2 3 3
3 6 6
4 10 10
Pero no basta esta comprobacin para armar que la frmula es correcta, por esto hacemos
el siguiente razonamiento. Llamamos X al subconjunto de Nformado por todos los nmeros
111
112 CAP. VI. MISCELNEA
naturales para los que la frmula es cierta. Ya hemos comprobado que 0, 1, 2, 3, 4 X. Como
queremos ver que la frmula es cierta para todos los nmeros naturales debemos comprobar
que X = N. Para probar nos apoyamos en que los nmeros naturales verican la propiedad
siguiente: Si un subconjunto S de N contiene a 0 y al siguiente de cada elemento de S, enton-
ces S = N. Es el llamado Principio de Induccin.
Nos vamos a basar enesta propiedad para ver que la frmula es cierta para todos los nmeros
naturales. En efecto, ya hemos comprobado que 0 X, y supongamos que x X, esto es, que
se verica
0 + 1 + 2 + + x =
x(x + 1)
2
.
Vamos a comprobar que tambin x + 1 X y tendremos X = N. Este es inmediato:
0 + 1 + 2 + + x + (x + 1) =
x(x + 1)
2
+ (x + 1) = (x + 1)
_
x
2
+ 1
_
=
(x + 1)(x + 2)
2
.
Obtenemos por tanto
0 + 1 + 2 + + n =
n(n + 1)
2
, para cada n N.
Este mtodo se conoce como el mtodo de induccin y es muy til a la hora de probar pro-
piedades de nmeros naturales.
Existen muchas otras frmulas de este tipo. Por ejemplo:
0 + 2 + 4 + + 2n = n(n + 1), para cada n N.
En este caso no es necesario hacer induccin, basta sacar el 2 factor comn a todos los su-
mandos.
Otra frmula til es:
1 + 3 + 5 + + (2n 1) = n
2
, para cada n N.
En este caso basta considerar el siguiente desarrollo:
1+3+5+ +(2n1) = (0+1+2+ +2n)(0+2+4+ +2n) =
2n(2n + 1)
2
n(n+1) = n
2
.
Ejercicio. 12.1.
Dados enteros positivos a
0
, a
1
, . . . , a
100
vericando:
_

_
a
1
> a
0
,
a
2
= 3a
1
2a
0
,
. . .
a
100
= 3a
99
2a
98
.
_

_
Probar que a
100
> 2
99
.
[The problems of the All-Soviet-Unionmathematical competitions 1961-1986. Problem015]
28 de enero de 2013 Curso 20122013. NOTAS DE TRABAJO, 30
SEC. 12. NMEROS NATURALES. EL PRINCIPIO DE INDUCCIN 113
SOLUCIN. [Ver tambin el Ejercicio (9.1.)] De la relacin a
1
> a
0
sabemos que existe h
N \ {0} tal que a
1
= a
0
+ h.
De la relacin a
2
= 3a
1
2a
0
se obtiene:
a
2
= 3(a
0
+ h) 2a
0
= a
0
+ 3h.
De esta forma tambin obtenemos:
a
3
= 3a
2
2a
1
= 3(a
0
+ 3h) 2(a
0
+ h) = a
0
+ 7h
Es decir, para cada ndice n se tiene una expresin:
a
n
= a
0
+ f
n
h,
siendo f
0
= 0, f
1
= 1, f
2
= 3, f
3
= 7, . . . .
La sucesin {f
n
}
n
sigue una ley de formacin, por ejemplo f
n
= 2
n
1. Esta ley que se com-
prueba para los valores 0, 1, 2, 3, queremos probar que es cierta para todo nmero natural.
Vamos a probar este hecho por induccin sobre n. Llamamos X al subconjunto de N forma-
do por los nmeros naturales x tales que f
x
= 2
x
1. Ya hemos comprobado que 0, 1 X;
supongamos que 0, 1, 2, . . . , x X, siendo x 2, y vamos a probar que tambin x +1 X. en
efecto, se tiene:
a
x+1
= 3a
x
2a
x1
= 3(a
0
+ (2
x
1)h) 2(a
0
+ (2
x1
1)h)
= a
0
+ (3(2
x
1) 2(2
x1
1))h = a
0
+ (3 2
x
2 2
x1
1)h
= a
0
+ (2
x+1
1)h.
Como consecuencia f
x+1
= 2
x+1
1 y tambin x + 1 X. Entonces X = N.
Al estudiar el caso de a
100
tenemos: a
100
= a
0
+ (2
100
1)h, y como a
0
0 y h 1, resulta
que a
100
> 2
99
.
ARITMTICA Y COMBINATORIA
(I. Sucesiones recurrentes)
P. Jara
114 CAP. VI. MISCELNEA
13. Progresiones aritmticas
Una sucesin de nmeros reales {a
n
}
n
= {a
0
, a
1
, a
2
, . . .} se llama una progresin aritmtica
si cada trmino a
n
se obtiene del anterior sumando un nmero jo d. El nmero d se llama
la diferencia de la progresin. Esto es, si a
n+1
= a
n
+ d para cada n 0. Observar que la
progresin aritmtica est dada por el primer trmino a
0
y la diferencia, ya que para cada
ndice n se tiene a
n
= a
0
+nd. (La demostracin de este hecho se debe hacer por induccin.)
La descripcin a
n
= a
0
+ nd, para cada n N se conoce como el trmino general de la
progresin.
La suma de los primeros trminos de una progresin aritmtica se calcula de forma sencilla
aplicando lo ya conocido sobre la suma de los primeros nmeros naturales.
a
0
+ a
1
+ + a
n
= a
0
+ (a
0
+ d) + + (a
0
+ nd) = na
0
+ (0 + 1 + + n)
= na
0
+
n(n+1)
2
=
2na
0
+n(n+1)
2
=
(a
0
+a
n
)(n+1)
2
.
Observar que si queremos sumar trminos consecutivos de la progresin {a
n
}
n
se tiene:
a
r
+ a
r+1
+ + a
r+s
=
(a
0
+ a
r+s
)(r + s + 1)
2

(a
0
+ a
r1
)r
2
=
(a
r
+ a
r+s
)(s + 1)
2
.
Esto es la suma del primero ms el ltimo por el nmero de trminos divido por 2.
Vamos a intentar sumar los cuadrados de los primeros nmeros naturales:
0 + 1 + 2
2
+ + n
2
Si llamamos s
n
a este valor, tenemos una sucesin {s
n
}
n
. Como no es una progresin aritm-
tica, tal y como anteriormente hemos denido, no sabemos como hacer esta suma. Vamos a
hacer algunas operaciones con los nmeros s
n
. Los escribimos en una columna y a la derecha
de cada uno escribimos la diferencia con el siguiente; esto nos forma una segunda columna,
formada por los cuadrados de los nmeros naturales. Repetimos el proceso para obtener una
tercera columna que, ahora s, es una progresin aritmtica. Si a esta columna le hacemos lo
mismo obtenemos una cuarta columna en la que todos los trminos con constantes. De-
cimos que la sucesin {s
n
}
n
es una progresin aritmtica de tercer grado, mientras que la
sucesin {n
2
}
n
es una progresin aritmtica de segundo grado.
n s
n
s
n

2
s
n

3
s
n
0 0 1 3 2
1 1 4 5 2
2 5 9 7 2
3 14 16 9
4 30 25
5 55
En general una sucesin {a
n
}
n
forma una progresin aritmtica de grado t si al someterla a
este proceso la columna t tiene todos sus trminos iguales.
28 de enero de 2013 Curso 20122013. NOTAS DE TRABAJO, 30
SEC. 13. PROGRESIONES ARITMTICAS 115
Podemos probar que el trmino general, a
n
, de una progresin aritmtica de grado t se ob-
tiene como
a
n
=
n

i=0
_
n
i
_

i
a
0
.
En donde hemos utilizado la siguiente notacin:

0
a
0
es el trmino a
0
,

1
a
0
es el trmino que ocupa el primer lugar en la segunda columna, y en general
i
A
0
es el elemento que ocupa el primer lugar en la columna i + 1.
En el caso de la suma de cuadrados de los primeros nmeros naturales se tiene:
s
n
=

n
i=0
_
n
i
_

i
s
0
=
_
n
0
_
s
0
+
_
n
1
_

1
s
0
+
_
n
2
_

2
s
0
+
_
n
3
_

3
s
0
= 1 0 + n 1 +
n(n1)
2
3 +
n(n1)(n2)
6
2 = n +
n(n1)
2
+
3n(n1)(n2)
3
=
n
3
+n
2
+n
6
=
n(n+1)(2n+1)
6
Ejercicio. 13.1.
Determina la frmula para la suma de los cubos de primeros nmeros naturales.
SOLUCIN. Llamamos {s
n
}
n
a la sucesin denida s
n
= 0 + 1 + 2
3
+ + n
3
. Hacemos la
construccin en este caso:
n s
n
s
n

2
s
n

3
s
n

4
s
n
0 0 1 7 12 6
1 1 8 19 18 6
2 9 27 37 24 6
3 36 64 61 30
4 100 125 91
5 225 216
6 441
El trmino general es:
s
n
=

n
i=0
_
n
i
_

i
s
0
=
_
n
0
_
s
0
+
_
n
1
_

1
s
0
+
_
n
2
_

2
s
0
+
_
n
3
_

3
s
0
+
_
n
4
_

4
s
0
= 1 0 + n 1 +
n(n1)
2
7 +
n(n1)(n2)
6
12 +
n(n1)(n2)(n3)
24
6
= n +
7n(n1)
2
+ 2n(n 1)(n 2) +
n(n1)(n2)(n3)
4
=
_
n(n+1)
2
_
2
.

ARITMTICA Y COMBINATORIA
(I. Sucesiones recurrentes)
P. Jara
116 CAP. VI. MISCELNEA
Ejercicio. 13.2.
Demuestra que se verica
0 + 1 + 2
4
+ + n
4
=
n(n + 1)(2n + 1)(3n
2
+ 3n 1)
30
.
28 de enero de 2013 Curso 20122013. NOTAS DE TRABAJO, 30
SEC. 14. PROGRESIONES GEOMTRICAS 117
14. Progresiones geomtricas
Una progresin geomtrica es una sucesin {a
n
}
n
vericando a
n+1
= a
n
r para cada n
0. Observa que una progresin geomtrica est determinada por los valores de a
0
y de r.
Llamamos a r la razn de la progresin. El caso en que r = 0 nos da una sucesin trivial, por
lo en general no lo vamos a considerar. Observar que se tiene a
n
= a
0
r
n
para cada n 0.
Para progresiones geomtricas es de inters calcular la suma de los primeros trminos.
a
0
+ a
1
+ + a
n
= a
0
+ a
0
r + + a
0
r
n
= a
0
(1 + r + + r
n
)
= a
0
x
n+1
1
x1
.
ARITMTICA Y COMBINATORIA
(I. Sucesiones recurrentes)
P. Jara
118 CAP. VI. MISCELNEA
15. Sucesiones recurrentes
Otro tipo de sucesiones de inters son las sucesiones denidas por recurrencia. Una sucesin
{a
n
}
n
est denida por recurrencia si a partir de un ndice cada trmino se puede calcular,
mediante una frmula, a partir de los trminos anteriores. Los ejemplos que vamos a estudiar
son aquellos que son semejantes a la sucesin de Fibonacci. Recordar que esta sucesin est
denida mediante
a
0
= 0, a
1
= 1, a
n+2
= a
n+1
+ a
n
, para todo n 0.
Consideremos por tanto el caso general, esto es, una sucesin denida por:
a
0
, a
1
, a
n+2
= c
1
a
n+1
+ c
2
a
n
, para todo n 0.
Siendo c
1
, c
2
R. Como ya hemos comentamos todas las sucesiones que verican esta rela-
cinformanunespacio vectorial, por lo tanto para estudiarlas todas vamos a dar una base del
mismo. No viene al caso, pero conviene sealar que la dimensin del espacio vectorial para
la sucesin de Fibonacci es igual a dos, y por tanto una base est formada por dos elementos
(dos sucesiones).
El mtodo de clculo consiste en ver qu progresiones geomtricas verican la relacin. Con-
sideremos una progresin geomtrica a
n
= a
0
r
n
(podemos suponer que a
0
= 0 y r = 0, ya
que en otro caso estaramos ante una sucesin trivial). Se tiene entonces las siguientes rela-
ciones que se pueden simplicar en la forma obvia.
a
0
r
n+2
= c
1
a
0
r
n+1
+ c
2
r
n
, r
2
= c
1
r + c
2
(ecuacin caracterstica).
Por tanto r es raz del polinomio X
2
c
1
X c
2
(polinomio caracterstico). Si es una raz de
este polinomio (puede ser un nmero complejo), entonces la sucesin {1, ,
2
, . . .} verica
la relacin a
n+2
= c
1
a
n+1
+ c
2
a
n
.
Si el polinomio tiene dos races distintas, y , entonces tenemos dos sucesiones distintas
{1, ,
2
, . . .} y {1, ,
2
, . . .} que satisfacen la relacin, y por tanto stas forman una base del
espacio vectorial.
Si es una raz doble del polinomio, como es tambin raz de la derivada del polinomio, se
tiene:
2 c
1
= 0.
Multiplicando por se tiene:
2
2
c
1
= 0.
Eneste caso la sucesin{0, , 2
2
, 3
3
, . . .} verica la relacina
n+2
= c
1
a
n+1
+c
2
a
n
. Enefecto,
si n 0 tenemos
c
1
(n + 1)
n+1
+ c
2
n
n
= nc
1

n+1
+ nc
2

n
+ c
1

n+1
= n
n+2
+ 2
2

n
= (n + 2)
n+2
.
La base est formada por {1, ,
2
, . . .} y {0, , 2
2
, 3
3
, . . .}.
28 de enero de 2013 Curso 20122013. NOTAS DE TRABAJO, 30
SEC. 15. SUCESIONES RECURRENTES 119
En este caso cualquier sucesin genrica que verica la relacin ser una combinacin de
estas dos sucesiones, esto es, es de la forma d
1

n
+d
2
n
n
, siendo d
1
, d
2
R. Ahora los valores
iniciales a
0
y a
1
nos permiten calcular los coecientes d
1
y d
2
.
Ejemplo. 15.1.
Determinar el trmino general de una sucesin {a
n
}
n
que verica las condiciones:
a
0
= 1, a
1
= 6, a
n+2
= 4a
n+1
4a
n
, para todo n 0.
SOLUCIN. En este caso c
1
= 4 y c
2
= 4 y el polinomio caracterstico es: X
2
4X + 4. Este
polinomio tiene una nica raz, = 2, con multiplicidad dos. Por lo tanto las sucesiones que
forman la base son:
{1, ,
2
, . . .} = {1, 2, 2
2
, . . .} y
{0, , 2
2
, 3
3
, . . .} = {0, 2, 8, 24, . . .}.
Una sucesin genrica tiene trmino general igual a:
d
1

n
+ d
2
n
n
.
Al vericar las condiciones iniciales resulta:
d
1

0
+ d
2
0
0
= 1
d
1

1
+ d
2
1
1
= 6
_
d
1
= 1
2d
1
+ 2d
2
= 6
_
La nica solucin es: d
1
= 1, d
2
= 2. la sucesin {a
n
}
n
tiene trmino general igual a
a
n
=
n
+ 2n
n
= (1 + 2n)
n
= (1 + 2n)2
n
.
Comprobar que esta sucesin verica las condiciones!
Vamos a hacer un ejemplo del caso en que las races del polinomio caracterstico son distin-
tas.
Ejemplo. 15.2.
Determinar el trmino general de una sucesin {a
n
}
n
que verica las condiciones:
a
0
= 1, a
1
= 5, a
n+2
= a
n+1
+ 2a
n
, para todo n 0.
SOLUCIN. En este caso c
1
= 1 y c
2
= 2 y el polinomio caracterstico es: X
2
X 2. Este
polinomio tiene dos races distintas, = 1, = 2. Por lo tanto las sucesiones que forman la
base son:
{1, ,
2
, . . .} = {1, 1, 1, 1, . . .} y
{1, ,
2
,
3
, . . .} = {1, 2, 2
2
, 2
3
, . . .}.
ARITMTICA Y COMBINATORIA
(I. Sucesiones recurrentes)
P. Jara
120 CAP. VI. MISCELNEA
Una sucesin genrica tiene trmino general igual a:
d
1

n
+ d
2

n
.
Al vericar las condiciones iniciales resulta:
d
1

0
+ d
2

0
= 1
d
1

1
+ d
2

1
= 5
_
d
1
+ d
2
= 1
(1)d
1
+ 2d
2
= 5
_
La nica solucin es: d
1
= 1, d
2
= 2. la sucesin {a
n
}
n
tiene trmino general igual a
a
n
=
n
+ 2
n
= (1)
n
+ 2 2
n
= (1)
n+1
+ 2
n+1
.
Comprobar que esta sucesin verica las condiciones!
28 de enero de 2013 Curso 20122013. NOTAS DE TRABAJO, 30
SEC. 16. EJERCICIOS RESUELTOS. SELECCIN 121
16. Ejercicios resueltos. Seleccin
Ejercicio (9.4.)
Ejercicio (9.5.)
La parte (1) del siguiente ejercicio no la hemos estudiado.
Ejercicio (9.6.)
Ejercicio (9.8.)
Ejercicio (9.13.)
Ejercicio (11.25.)
Ejercicio (11.26.)
Ejercicio (11.27.)
Ejercicio (11.28.)
ARITMTICA Y COMBINATORIA
(I. Sucesiones recurrentes)
P. Jara
122 CAP. VI. MISCELNEA
17. Recurrencia en combinatoria
El nmero de subconjuntos de r elementos de un conjunto de n elementos se representa por
_
n
r
_
, y se calcula como
_
n
r
_
=
n(n 1) (n r + 1)
r(r 1) 1
=
n!
r!(n r)!
.
Consecuencia de la denicin son las siguientes propiedades:
Lema. 17.1.
Para n y r nmeros enteros se tiene:
(1)
_
n
0
_
= 1 =
_
n
n
_
.
(2) Si n > r, entonces
_
n
r
_
=
_
n
nr
_
.
(3)
_
n
r
_
= 0 si r > n.
Adems se tiene la siguiente propiedadque permitir dar unmtodorecurrente para el clcu-
lo de
_
n
r
_
.
Proposicin. 17.2.
Para cada n, r N, se tiene
_
n + 1
r
_
=
_
n
r
_
+
_
n
r 1
_
.
DEMOSTRACIN. Dado un conjunto X = {x
0
, x
1
, . . . , x
n
} con n + 1 elementos y r N, si
r > n + 1, entones en la expresin del enunciado todos los elementos son iguales a cero y se
tiene la relacin. Si r = n+1, entonces
_
n
r
_
= 0 y se tiene la igualdad 1 = 1, que evidentemente
es cierta. Si r < n + 1 al formar un subconjunto de X con r elementos, si este subconjunto
no contiene a x
n
, entonces podemos formarlo de
_
n
r
_
formas distintas, y si contiene a x
n
,
entonces el subconjunto formado por los r 1 elementos restantes puede construirse de
_
n
r1
_
formas distintas. en consecuencia se tiene
_
n + 1
r
_
=
_
n
r
_
+
_
n
r 1
_
.
para cualesquiera n, r N.
28 de enero de 2013 Curso 20122013. NOTAS DE TRABAJO, 30
SEC. 17. RECURRENCIA EN COMBINATORIA 123
Fruto de este resultado se tiene el llamado tringulo de Tartaglia:
_
0
0
_
_
1
0
_ _
1
1
_
_
2
0
_ _
2
1
_ _
2
2
_
_
3
0
_ _
3
1
_ _
3
2
_ _
3
3
_
_
4
0
_ _
4
1
_ _
4
2
_

_
4
3
_ _
4
4
_
_
5
0
_ _
5
1
_ _
5
2
_ _
5
3
_ _
5
4
_ _
5
5
_
1
1 1
1 2 1
1 3 3 1
1 4 6 4 1
1 5 10 10 5 1
1 6 15 20 15 6 1
1 7 21 35 35 21 7 1
1 8 28 56 70 56 28 8 1
1 9 36 84 126 126 84 36 9 1
1 10 45 120 210 252 210 120 45 10 1
La primera aplicacin del tringulo de Tartaglia es el desarrollo del binomio, ya que la la
nsima tiene los coecientes del desarrollo del binomio (a + b)
n
.
(a + b)
n
=
_
n
0
_
a
n
+
_
n
1
_
a
n1
b +
_
n
2
_
a
n2
b
2
+ +
_
n
n 1
_
ab
n1
+
_
n
n
_
b
n
.
Podemos considerar lo anterior como un desarrollo en serie de la siguiente forma:
(1 + x)
n
= 1 + nx +
n(n 1)x
2
2!
+
n(n 1)(n 2)x
3
3!
+
Si n = 1, 2, 3, . . ., tenemos un valor exacto de esta expansin. En cambio si tomamos n = 1/2
el valor que se obtiene:
(1 + x)
1
2
= 1 +
1
2
x +
1
2
1
2
x
2
2!
+
1
2
1
2
3
2
x
3
3!
+ ,
que da una aproximacin al valor (1 + x)
1
2
. Para calcular

2 procedemos como sigue:


(1 + 1)
1
2
= 1 +
1
2

1
8
+
1
16
+
En este caso no tenemos problemas, y la razn es que la serie 1+nx+
n(n1)x
2
2!
+
n(n1)(n2)x
3
3!
+
converge cuando x 1. por esta razn no podemos aplicarla para aproximar por ejemplo
ARITMTICA Y COMBINATORIA
(I. Sucesiones recurrentes)
P. Jara
124 CAP. VI. MISCELNEA

35, pues el desarrollo de la serie (1+34)


1
2
no converge. Sin embargo, si se toma un cuadrado
perfecto anterior a 35, en este caso 25, podemos escribir

35 = (25 + 10)
1
2
= 5(1 +
10
25
)
1
2
=
Para el clculo de la raz cbica procedemos de la misma forma. En este caso tenemos que
calcular el desarrollo de
(1 + x)
1
3
= 1 +
1
3
x +
1
3
2
3
x
2
2!
+
1
3
2
3
5
3
x
3
3!
+
Para el calculo de la raz cbica de 35,
3

35 procedemos como sigue:


3

35 = (27 + 8)
1
3
= 3(1 +
8
27
)
1
3
=
en donde ahora hemos considerado un cubo menor que 35.
Problema. 17.3.
Determinar el nmero de soluciones enteras positivas de la ecuacin
x
1
+ + x
m
= n,
en donde n, m N.
La solucin a este problema nos da tambin el nmero de combinaciones con repeticin de
m elementos tomados de n en n.
SOLUCIN. Llamamos S
n,m
al nmero de soluciones distintas de la ecuacin anterior, en
donde n, m N. Observa que:
(1) S
0,m
= 0 para cada m N.
(2) S
n,0
= 0 para cada n N \ {0}.
(3) S
n,1
= 1 para cada n N \ {0}.
Adems, dados n, m N, el nmero de soluciones de x
1
+ + x
m+1
= n + 1 con x
m+1
= 1
es exactamente S
n,m
, salvo en el caso en que n = 0 = m que vale 1. Y si se tiene una solucin
con x
m+1
> 1, se tiene una solucin de la ecuacin y
1
+ + y
m+1
= n tomando y
i
= x
i
si i = 1, 2, . . . , m e y
m+1
= x
m+1
1, y viceversa; por lo tanto el nmero de soluciones con
x
m+1
> 1 es S
n,m+1
. En consecuencia se tiene
S
n+1,m+1
= S
n,m
+ S
n,m+1
.
28 de enero de 2013 Curso 20122013. NOTAS DE TRABAJO, 30
SEC. 17. RECURRENCIA EN COMBINATORIA 125
en donde n, m N no son ambos iguales a 0, y S
1,1
= 1. Todos estos valores podemos escri-
birlos en la siguiente tabla:
S
0,0
S
0,1
S
0,2
S
0,3

S
1,0
S
1,1
S
1,2
S
1,3

S
2,0
S
2,1
S
2,2
S
2,3
S
2,4

S
3,0
S
3,1
S
3,2
S
3,3
S
3,4

S
4,0
S
4,1
S
4,2
S
4,3
S
4,4
S
4,5

S
5,0
S
5,1
S
5,2
S
5,3
S
5,4
S
5,5

0 0 0 0
0 1 0 0
0 1 1 0 0
0 1 2 1 0
0 1 3 3 1 0
0 1 4 6 4 1
Los puntos en color azul tienen el valor cero, y los puntos an color rojo o negro reproducen
el tringulo de Tartaglia.
En efecto, vamos a probar que S
n+1,m+1
=
_
n
m
_
para todo n, m N. Hacemos induccin sobre
n y m. Si n = 0 = m, entonces se tiene S
1,1
= 1 =
_
0
0
_
, y el resultado es cierto.
El resultado es cierto para todos los pares (n, 0), ya que S
n+1,1
= 1 =
_
n
0
_
. Vamos a suponer
que el resultado es cierto para todos los pares (n, m) siendo n t, y sea cualquiera el valor
de m. Tenemos entonces S
n+1,m+1
= S
n,m
+ S
n,m+1
=
_
n1
m1
_
+
_
n1
m
_
=
_
n
m
_
. Por lo tanto el
resultado es cierto para todos los pares (n, m) N
2
.
En consecuencia el nmero de soluciones enteras positivas de la ecuacin x
1
+ + x
m
= n
es igual a
S
n,m
=
_
n 1
m1
_
,
si n, m 1 y S
n,m
= 0 si n = 0 o m = 0.
Problema. 17.4.
Determinar el nmero de soluciones enteras no negativas de la ecuacin
x
1
+ + x
m
= n,
en donde n, m N.
SOLUCIN. Llamamos S
n,m
al nmero de soluciones distintas de la ecuacin anterior, con
n, m N. Se tienen los siguientes valores:
(1) S
n,0
= 0 para cada n N.
(2) S
0,m
= 1 para cada m N \ {0}.
ARITMTICA Y COMBINATORIA
(I. Sucesiones recurrentes)
P. Jara
126 CAP. VI. MISCELNEA
Dados n, m N consideramos la ecuacin x
1
+ + x
m+1
= n + 1. El nmero de soluciones
con x
m+1
= 0 es
n+1,m
, salvo en el caso en que m = 0 que vale 1. Y si se tiene una solucin
con x
m+1
= 0, entonces tenemos una solucin de la ecuacin y
1
+ + y
m+1
= n tomando
y
i
= x
i
si i = 1, . . . , m e y
m+1
= x
m+1
1, y viceversa; por lo tanto el nmero de soluciones
con x
m+1
= 0 es S
n,m+1
. En consecuencia se tiene
S
n+1,m+1
= S
n,m
+ S
n,m+1
,
para cualesquiera n N y m N \ {0}, y S
n,1
= 1 para todo n N.
Agrupamos estos valores en una tabla
S
0,0
S
0,1
S
0,2
S
0,3
S
0,4

S
1,0
S
1,1
S
1,2
S
1,3
S
1,4

S
2,0
S
2,1
S
2,2
S
2,3
S
2,4

S
3,0
S
3,1
S
3,2
S
3,3
S
3,4

S
4,0
S
4,1
S
4,2
S
4,3
S
4,4

0 1 1 1 1
0 1 2 3 4
0 1 3 6 10
0 1 4 10 20
0 1 5 15 35
Se trata de calcular el valor exacto de cada S
n,m
. Observa que una posible hiptesis es:
S
n,m
=
_
n + m1
m1
_
=
_
n + m1
n
_
,
si m 1, y S
n,0
= 0 en otro caso. Consideramos el caso de S
n,m
con m = 0. Vamos a calcular
S
0,m
; sabemos que S
0,m
= 1 cuando m = 0, y se tiene
_
m1
m1
_
= 1; por lo tanto el resultado
es cierto para estos valores. Para cualquier valor de n se tiene S
n,1
= 1 =
_
n
n
_
, y por tanto el
resultado es cierto para estos valores.
Supongamos que el resultado es cierto para todos los pare (t, s), s = 0, con t + s menor que
un cierto h > 0; vamos a ver que es cierto para todos los pares (n, m), m = 0. Se tiene S
t+1,s
=
S
t,s
+ S
t+1,s1
=
_
t+s1
s1
_
+
_
t+s1
s2
_
=
_
t+s
s1
_
, y de la misma forma S
t,s+1
= S
t1,s+1
+ S
t,s
=
_
t+s1
s
_
+
_
t+s1
s1
_
=
_
t+s
s
_
.
Como consecuencia se tiene
S
n,m
=
_
n + m1
m1
_
,
para n N, m N \ {0} y S
n,0
= 0 para n N.
Igualdades con nmeros combinatorios.
Lema. 17.5.
_
n+1
k+1
_
=

n
j=0
_
j
k
_
.
28 de enero de 2013 Curso 20122013. NOTAS DE TRABAJO, 30
SEC. 17. RECURRENCIA EN COMBINATORIA 127
Esta igualdad expresa que en el tringulo de Pascal, el que aparece ms abajo, cada elemento
se consigue como la suma de los elementos de la columna anterior.
_
0
0
_ _
1
1
_ _
2
2
_ _
3
3
_ _
4
4
_

_
1
0
_ _
2
1
_ _
3
2
_ _
4
3
_ _
5
4
_

_
2
0
_ _
3
1
_ _
4
2
_ _
5
3
_ _
6
4
_

_
3
0
_ _
4
1
_ _
5
2
_ _
6
3
_ _
7
4
_

_
4
0
_ _
5
1
_ _
6
2
_

_
7
3
_ _
8
4
_

_
5
0
_ _
6
1
_ _
7
2
_ _
8
3
_ _
9
4
_

_
6
0
_ _
7
1
_ _
8
2
_ _
9
3
_ _
10
4
_

1 1 1 1 1 1 1 1 1 1
1 2 3 4 5 6 7 8 9 10
1 3 6 10 15 21 28 36 45 55
1 4 10 20 35 56 84 120 165 220
1 5 15 35 70 126 210 330 495 715
1 6 21 56 126 252 462 792 1287 2002
1 7 28 84 210 462 924 1716 3003 5005
1 8 36 120 330 792 1716 3432 6435 11440
1 9 45 165 495 1287 3003 6435 12870 24310
1 10 55 220 715 2002 5005 11440 24310 48620
DEMOSTRACIN. Se parte de la formula de la adicin, Proposicin (17.2.), y se procede por
recurrencia:
_
n+1
k+1
_
=
_
n
k+1
_
+
_
n
k
_
=
_
n1
k+1
_
+
_
n1
k
_
+
_
n
k
_
=
_
n2
k+1
_
+
_
n2
k
_
+
_
n1
k
_
+
_
n
k
_
=
_
0
k+1
_
+
_
0
k
_
+
_
1
k
_
+ +
_
n1
k
_
+
_
n
k
_
=
_
0
k
_
+
_
1
k
_
+ +
_
n1
k
_
+
_
n
k
_

De forma semejante, trabajando ahora por las en vez de columnas, se tiene la frmula:
Lema. 17.6.
_
n+k+1
k
_
=

k
j=0
_
n+j
j
_
.
_
0
0
_ _
1
1
_ _
2
2
_ _
3
3
_ _
4
4
_

_
1
0
_ _
2
1
_ _
3
2
_ _
4
3
_ _
5
4
_

_
2
0
_ _
3
1
_ _
4
2
_ _
5
3
_ _
6
4
_

_
3
0
_ _
4
1
_ _
5
2
_ _
6
3
_ _
7
4
_

_
4
0
_ _
5
1
_ _
6
2
_ _
7
3
_

_
8
4
_

_
5
0
_ _
6
1
_ _
7
2
_ _
8
3
_ _
9
4
_

_
6
0
_ _
7
1
_ _
8
2
_ _
9
3
_ _
10
4
_

ARITMTICA Y COMBINATORIA
(I. Sucesiones recurrentes)
P. Jara
128 CAP. VI. MISCELNEA
Otras identidades de inters son:
Lema. 17.7.
_
n
m
__
m
k
_
=
_
n
k
__
nk
mk
_
.
DEMOSTRACIN.
_
n
m
_ _
m
k
_
=
n!
m!(nm)!
m!
k!(mk)!
=
n!
k!(nk)!
(nk)!
(nm)!(mk)!
=
_
n
k
_ _
nk
mk
_
.

Lema. 17.8.

k=0
_
r
m+k
__
s
nk
_
=
_
r+s
m+n
_
.
DEMOSTRACIN. Comparar los desarrollos de (X +Y)
r
(X +Y)
s
y (X +Y)
r+s
; los trminos de
la igualdad del enunciado son los coecientes de X
r+s
Y
r+s(m+n)
en cada uno de los desarro-
llos.
Tringulo de Sierpinski
El tringulo de Sierpinski. Consideramos el tringulo de Tartaglia y reducimos cada entra-
da mdulo 2; de esta forma si el valor de una entrada es par aparecer un 0 y si es impar
aparecer un 1.
1
1 1
1 0 1
1 1 1 1
1 0 0 0 1
1 1 0 0 1 1
1 0 1 0 1 0 1
1 1 1 1 1 1 1 1
1 0 0 0 0 0 0 0 1
1 1 0 0 0 0 0 0 1 1
1 0 1 0 0 0 0 0 1 0 1
1 1 1 1 0 0 0 0 1 1 1 1
1 0 0 0 1 0 0 0 1 0 0 0 1
1 1 0 0 1 1 0 0 1 1 0 0 1 1
1 0 1 0 1 0 1 0 1 0 1 0 1 0 1
1 1 1 1 1 1 1 1 1 1 1 1 1 1 1 1
28 de enero de 2013 Curso 20122013. NOTAS DE TRABAJO, 30
SEC. 17. RECURRENCIA EN COMBINATORIA 129
















Problema. 17.9.
Se considera t N \ {0}. Determina la mayor potencia de 2 que divide a 2
t
!
SOLUCIN. Primero comprobamos que
2 | 2
1
, 2
3
| 2
2
!, 2
7
| 2
3
!
Vamos a probar que la mayor potencia de 2 que divide a 2
t
! es 2
2
t
1
. Tomamos como hiptesis
de induccin que para cada s t la mayor potencia de 2 que divide a 2
s
! es 2
2
s
1
. Vamos a
estudiar 2
t+1
! Consideramos la fraccin
2
t+1
!
2
t
!
= (2
t
+ 1)(2
t
+ 2) (2
t
+ 2
t
).
Prescindiendo de los impares tenemos que estudiar la lista
2
t
+ 2, 2
t
+ 4, . . . , 2
t
+ 2
t
.
Salvo el caso de 2
t
+ 2
t
= 2
t+1
que es divisible por 2
t+1
, todos los dems son divisibles como
mximo por 2
t
, y en cada caso para estudiar la potencia de 2 que divide a 2
t
+x basta estudiar
la divisibilidad de x; observa que en el caso de 2
t+1
tendremos que aadir un factor 2 extra al
nal.
Estudiamos la lista
2, 4, . . . , 2
t
esta lista tiene 2
t1
elementos. Dividimos por 2 cada elemento y obtenemos
1, 2, . . . , 2
t1
.
ARITMTICA Y COMBINATORIA
(I. Sucesiones recurrentes)
P. Jara
130 CAP. VI. MISCELNEA
El nmero de potencias de 2 que dividen a estos elementos es el exponente de la mayor po-
tencia de 2 que divide a 2
t1
!, y por hiptesis es 2
t1
1. As pues la mayor potencia de 2 que
divide a
2
t+1
!
2
t
!
es 2
2
t1
2
2
t1
1
2 = 2
2
t
.
Reuniendo los resultados anteriores se tiene que la mayor potencia de 2 que divide a 2
t+1
! es
2
2
t
1
2
2
t
= 2
2
t+1
1
.

Problema. 17.10.
Se considera t N \ {0}. Demuestra que
_
2
t
1
i
_
es impar para cada 0 i 2
t
1.
SOLUCIN. Tenemos
_
2
t
1
i
_
=
(2
t
1)(2
t
2) (2
t
i)
i(i 1) 2
Podemos prescindir de los factores impares, y si 2h i es el mayor par menor que i, entonces
tendremos que estudiar la fraccin
(2
t
2)(2
t
4) (2
t
2h)
2h 2
Como solo nos interesan las potencias de 2, las potencias que dividen al numerador y al de-
nominador son las mismas, por tanto
_
2
t
1
i
_
es impar.
Problema. 17.11.
Se considera el tringulo de Tartaglia. Determina cuanto nmeros pares e impares hay en la
lnea 51 (corresponde a
_
51
i
_
).
SOLUCIN. En el tringulo de Tartaglia reducido mdulo 2 la la 2
t
est formada solo por
unos, ya que todos elementos de la misma en el tringulo de Tartaglia son impares. En la la
2
t
+ 1 se inician dos nuevos tringulos, en las posiciones primera y ltima, con 2
t
las que
son iguales al tringulo que se inicia en la posicin inicial. Como consecuencia el nmero de
elementos impares en la la 2
t
+ h es el doble que el de la la h, siendo 1 h 2
t
.
Como consecuencia, como 51 = 32 + 19, el nmero de elementos impares es el doble que el
nmero de elementos de la la 19. Ahora, como 19 = 16+3, el nmero de elementos impares
en la la 19 es el doble que el de la la 3. Como la la 3 tiene 2 elementos impares, la la 19
tiene 2 2 y la la 51 tiene 2 2 2 = 8.
28 de enero de 2013 Curso 20122013. NOTAS DE TRABAJO, 30
SEC. 18. AMPLIACIN DE NMEROS COMBINATORIOS 131
18. Ampliacin de nmeros combinatorios
Nmeros de Stirling de segunda especie
El nmero
_
n
m
_
indica el nmero de formas en que un conjunto de n elementos puede
descomponerse en particiones de m conjuntos se llama un nmero de Stirling se segunda
especie. Por ejemplo
_
4
2
_
indica el nmero de particiones de un conjunto de 4 elementos
que constan de exactamente dos conjuntos. Si X = {a, b, c, d}, las posibles particiones son:
{a, b, c} {d}; {a, b} {c, b}
{a, b, d} {c}; {a, c} {b, d}
{a, c, d} {b}; {a, d} {b, c}
{b, c, d} {a};
y por lo tanto
_
4
2
_
= 7.
Vamos a determinar una forma recursiva para los nmeros de Stirling de segunda especie.
Para calcular
_
n + 1
m + 1
_
consideramos el conjunto X = {x
0
, . . . , x
n
}, a partir del conjunto
{x
0
, . . . , x
n1
} construimos
_
n
m + 1
_
, ahora el elemento x
n
podemos colocarlo en cada uno
de los conjuntos que forman cada particin, obteniendo as particiones distintas con m ele-
mentos. Faltan las particiones en m + 1 conjuntos, uno de los cuales es {x
n
}, pero stas sa-
bemos que son exactamente
_
n
m
_
. Como consecuencia el nmero de Stirling de segunda
especie verica la relacin:
_
n + 1
m + 1
_
= (m + 1)
_
n
m + 1
_
+
_
n
m
_
.
Los nmeros de Stirling de segunda especie podemos agruparlos en la siguiente tabla:
ARITMTICA Y COMBINATORIA
(I. Sucesiones recurrentes)
P. Jara
132 CAP. VI. MISCELNEA
_
0
0
_
= 1
_
1
0
_
= 0
_
1
1
_
= 1
_
2
0
_
= 0
_
2
1
_
= 1
_
2
2
_
= 1
_
3
0
_
= 0
_
3
1
_
= 1
_
3
2
_
= 3
_
3
3
_
= 1
_
4
0
_
= 0
_
4
1
_
= 1
_
4
2
_
= 7
_
4
3
_
= 6
_
4
4
_
= 1
_
5
0
_
= 0
_
5
1
_
= 1
_
5
2
_
= 15
_
5
3
_
= 25
_
5
4
_
= 10
_
5
5
_
= 1
_
6
0
_
= 0
_
6
1
_
= 1
_
6
2
_
= 31
_
6
3
_
= 90
_
6
4
_
= 65
_
6
5
_
= 15
_
6
6
_
= 1
_
7
0
_
= 0
_
7
1
_
= 1
_
7
2
_
= 63
_
7
3
_
= 301
_
7
4
_
= 350
_
7
5
_
= 140
_
7
6
_
= 21
Nmeros de Stirling de primera especie
El nmero de Stirling de primera especie
_
n
m
_
es el nmero de formas en que podemos des-
componer un subconjunto de n elementos en m conjuntos mutuamente disjuntos conside-
rando las ordenaciones cclicas de cada uno de estos conjuntos. Por ejemplo, si consideramos
X = {a, c, b, d}, entonces tenemos las siguientes descomposiciones:
_
4
1
_
= 6.
[a, b, c, d] [a, b, c, d] [a, c, b, d] [a, d, c, b] [a, c, d, b] [a, d, b, c].
_
4
2
_
= 11.
[a, b, c], [d] [a, c, b], [d] [a, b, d], [c] [a, d, b], [c]
[a, c, d], [b] [a, d, c], [b] [b, c, d], [d] [b, d, c], [a]
[a, b], [c, d] [a, c], [b, d] [a, d], [b, c].
_
4
3
_
= 6.
[a, b], [c], [d] [a, c], [b], [d] [a, d], [b], [c] [b, c], [a], [d] [b, d], [a], [c] [c, d], [a], [b].
28 de enero de 2013 Curso 20122013. NOTAS DE TRABAJO, 30
SEC. 18. AMPLIACIN DE NMEROS COMBINATORIOS 133
Nmeros trinomiales
Los nmeros combinatorios
_
n
m
_
aparecen el desarrollar el binomio (a+b)
n
segn la Frmula
de Newton. Si se desarrolla el trinomio (a + b + c)
n
se obtiene una expresin similar:
(a + b + c)
n
=
n

i=0
n

j=0,i+jn
_
n
i, j
_
a
i
b
j
c
nij
Los nmeros
_
n
i,j
_
se llaman nmeros trinomiales.
Problema. 18.1.
Determina el valor de
_
n
i,j
_
.
Por comodidad el trinomio
_
n
i,j
_
lo representamos por
_
n
i,j,k
_
, siendo i +j +k = n. Observa que
ahora la expresin del trinomio es:
_
n
i, j, k
_
=
n!
i!j!k!
.
El nmero
_
n
i,j,k
_
representa el nmero de particiones ordenadas de unconjunto de nelemen-
tos en tres conjuntos con i, j y k elementos.
Por ejemplo, si X = {a, b, c}, entonces
_
3
1,1,1
_
=
3!
1!1!1!
= 6, que corresponde a las particiones:
{{a}, {b}, {c}} , {{a}, {c}, {b}} , {{b}, {a}, {c}} ,
{{b}, {c}, {a}} , {{c}, {a}, {b}} , {{c}, {b}, {a}} .
En cambio
_
3
2,1,0
_
=
3!
2!1!
= 3 corresponde a las particiones:
{{a, b}, {c}, } , {{a, c}, {b}, } , {{b, c}, {a}, } ,
y
_
3
1,0,2
_
= 3 corresponde a las particiones:
{{c}, , {a, b}} , {{b}, , {a, c}} , {{a}, , {b, c}} .
De forma similar el tringulo de Tartaglia, se puede construir, en este caso una pirmide, en
la que aparezcan todos los trinomios. el primer nivel de la pirmide es:
_
0
0,0,0
_
El segundo nivel es:
_
1
1,0,0
_ _
1
0,0,1
_
_
1
0,1,0
_
ARITMTICA Y COMBINATORIA
(I. Sucesiones recurrentes)
P. Jara
134 CAP. VI. MISCELNEA
El tercer nivel es:
_
2
2,0,0
_ _
2
1,0,1
_ _
2
0,0,2
_
_
2
1,1,0
_ _
2
0,1,1
_
_
2
0,2,0
_
El cuarto nivel es:
_
3
3,0,0
_ _
3
2,0,1
_ _
3
1,0,2
_ _
3
0,0,3
_
_
3
2,1,0
_ _
3
1,1,1
_ _
3
0,1,2
_
_
3
1,2,0
_ _
3
0,2,1
_
_
3
0,3,0
_
El quinto nivel es: _
4
4,0,0
_ _
4
3,0,1
_ _
4
2,0,2
_ _
4
1,0,3
_ _
4
0,0,4
_
_
4
3,1,0
_ _
4
2,1,1
_ _
4
1,1,2
_ _
4
0,1,3
_
_
4
2,2,0
_ _
4
1,2,1
_ _
4
0,2,2
_
_
4
1,3,0
_ _
4
0,3,1
_
_
4
0,4,0
_
El sexto nivel es:
_
5
5,0,0
_ _
5
4,0,1
_ _
5
3,0,2
_ _
5
2,0,3
_ _
5
1,0,4
_ _
5
0,0,5
_
_
5
4,1,0
_ _
5
3,1,1
_ _
5
2,1,2
_ _
5
1,1,3
_ _
5
0,1,4
_
_
5
3,2,0
_ _
5
2,2,1
_ _
5
1,2,2
_ _
5
0,2,3
_
_
5
2,3,0
_ _
5
1,3,1
_ _
5
0,3,2
_
_
5
1,4,0
_ _
5
0,4,1
_
_
5
0,5,0
_
Observa que la regla de formacin es: Los puntos en azul se completan con la ley
_
n + 1
i, j, k
_
=
_
n
i 1, j, k
_
+
_
n
i, j 1, k
_
en sus tres versiones (observa que as se pasa de un nivel al siguiente), y los puntos en rojo
se completan con la ley
_
n + 1
i, j, k
_
=
_
n + 1
i + 1, j 1, k
_
+
_
n
i 1, j + 1, k
_
.
Problema. 18.2.
Prueba las leyes enunciadas en el prrafo anterior.
Ejercicio. 18.3.
Completa el sptimo nivel.
28 de enero de 2013 Curso 20122013. NOTAS DE TRABAJO, 30
Bibliografa
[1] S. Lang, Algebra 3rd. ed., Springer, 2002.
ndice alfabtico
combinaciones con repeticin, 124
condiciones iniciales, 20
diferencia, 6, 114
diferencia de orden k > 1, 7
diferencia de orden uno, 7
ecuacin caracterstica, 22
ecuacin de recurrencia, 20
Espiral de Fibonacci, 42
Frmula de Binet, 23
funcin generatriz, 35
Identidad de Cassini, 62
Identidad de Catalan, 62
Identidad de dOcagne, 62
Identidad de GelinCesro, 62
Nmeros de Jacobsthal, 79
Nmeros de Lucas, 61
Nmeros de Padovan, 79
Nmeros de Pell, 79
Nmero de Stirling de primera especie, 132
Nmero de Stirling se segunda especie, 131
Nmero trinomial, 133
Nmeros de Perrin, 44
Principio de Induccin, 112
progresin aritmtica de grado k 0, 7
progresiones aritmticas, 6
progresiones geomtricas, 13
razn, 13
Serie de Fibonacci, 38
Serie de Maclaurin, 38
Sucesin de Pell, 79
Sucesin de Fibonacci, 20
sucesin recurrente, 20
sucesiones recurrentes no homogneas, 31
suma parcial, 6
trmino general, 6
Tringulo de Sierpinski, 128
Tringulo de Tartaglia, 123
137

Anda mungkin juga menyukai